You are on page 1of 351

4

Use Code NIMISHA for maximum Discount

For any query call or whatsapp 9953592800


5

Preface

Dear students,
English Language plays a very significant role in banking and other
competitive exams. This section is also a major deciding factor in your
success in every leg of your examination. The English section can be
tremendously scoring if one handles this section with all his dedication,
attention and care.

One of the key factors in attaining success in any field is the age old
concept of “practice makes a man perfect.” Although some people might
have an innate knack for a particular subject or skill, the truth is mastery
takes time, repetition and pushing oneself out of his/her comfort zone –
there is no shortcut!
Now, in the case of major banking and insurance exams, adequate and
appropriate practice materials aren't easily available. The contents you'll
rely on for the sake of practising must be exam-specific and relevant. So,
I've come up with ''Vijeta''(that translates to 'success finder') - a free-to-all
biweekly magazine for the English language that targets all the major
banking and insurance exams throughout the country.
Every question in this magazine has been framed with utmost care and
attention keeping in mind that millions of you will be highly benefitted.
All the different patterns of questions from this section that appear in the
exams have been furnished in a very organized manner.

You'll only need to carry on practising consistently, as steady and


consistent practice yields steady and consistent results.

Your invaluable suggestions are heartily welcome, as we believe in


relentless improvement.
Use Code NIMISHA for maximum Discount

For any query call or whatsapp 9953592800


6

ABOUT NIMISHA BANSAL

Nimisha Bansal is one of India's most eminent educators of English


language for Banking, Insurance and other government
examinations with over 300k followers on Unacademy. Her motto is to
make English an easy and accessible language for everyone. She also
has a very popular YouTube channel named 'Nimisha Bansal' with a
subscription base of more than half-a-million.

She is not only a highly-competent pedagog, but also a wonderful


motivator. She always promotes enthusiasm for giving your best.
She is also a prominent author of some very successful books for
the English language for various competitive exams. Some of her
top-selling books are Vocab Prodigy, 120 Rules of Grammar,
Descriptive Book and Perfection Book.
These books have been highly praised and are immensely beneficial
for the aspirants.
Thousands of students from her batches like Perfection Batch,
Achievers' series and Editorial Analysis have been selected in
various exams like SBI PO and Clerk, IBPS/RRB PO and Clerk and
RBI Assistant.
Nimisha Bansal also carries out many free classes on a regular
basis.
She has dedicated her life to the field of education and has coached lakhs
of students countrywide. She takes free classes regularly on both
Unacademy and her YouTube channel and has garnered millions of
views on her videos. With a willing heart to provide the best education to
her students, Nimisha Bansal has proudly helped thousands of students
achieve their dream jobs every year. Many of her students are currently
working in various public sector banks and government organizations.

Use Code NIMISHA for maximum Discount

For any query call or whatsapp 9953592800


7

She believes education should not be limited to the youth but to anyone
who shares the love of learning.

A Note to my champions

Dear students,

Getting a government job isn't a child's play nowadays. One must go


through a stiff competitive situation, recurrent stresses and lack of
confidence on several occasions. It's not at all 'a bed of roses'.
But success is no accident. It is consistent hard work, perseverance,
learning, studying and sacrifice.

So, how can goals be accomplished?


What are the main qualities you need to inculcate to be successful?
First, you need to keep in mind that there are four most important
mantras for success.
Patience, Perseverance, Practice and Planning.
It is important to be patient and keep going to reach your goals. Once we
learn the importance of patience, we can stop worrying when things take
longer than we would like.
Progress occurs by trying, understanding what works and what does not
work, course-correcting, and trying again.
Success comes to those who continue to make efforts to achieve
something in their lives, which is what perseverance is all about.
Planning can be defined as “thinking in advance what is to be done,
when it is to be done, how it is to be done and by whom it should be
done”. In simple words we can say, planning bridges the gap between
where we are standing today and where we want to reach.

Use Code NIMISHA for maximum Discount

For any query call or whatsapp 9953592800


8

Second, target every exam as it's the easiest exam. It's not your
overconfidence. It's your self-assurance or confidence. Confidence is
believing in yourself, feeling comfortable in your true self, knowing you
have everything required to succeed.
By this positive mindset, you'll not be overwhelmed by the fear of losing.

Third, enjoy the journey. It's a healthy way to maintain balance between
doing and being, between producing results and going through the
required processes. Don't become too burdened with all your anxieties,
doubts and uncertainties.
There will be stress, doubts, fears but you will have to constantly remind
yourself that you can do it and you will do it.
Fourth, it is so important to realize the magnitude of introspection.
The ability to examine, explore and get intimate with your inner conscious
and feelings allows you to take responsibility for your growth.
By looking into and reviewing your own thoughts and the actions attached
to them, you get to know yourself - your strengths, your weaknesses.
Once you have understood these aspects, you can start working on them.

We are wired to move toward things that make us feel good and away
from things that make us feel uncomfortable. Our brains tag effort as bad
because it’s hard work.
But hard actions can have tremendous benefits — ones that may not be
visible for some time.
We can accomplish hard things by practicing the habits of a growth
mindset.
Challenges and problems are important parts of life that give you
experiences; make you learn and help you become stronger. Problems
make us grow and shape us. The biggest problem people have is that
they hope for a life without problems.

Use Code NIMISHA for maximum Discount

For any query call or whatsapp 9953592800


9

Therefore, preparing yourself for the hardest challenges will certainly help
you achieve success more efficiently.

Finally, the temptation to give up is a common one, and nobody is


exempt. Failure isn't something many of us can handle gracefully. And
even though we know it's a common human condition, we're somehow
always surprised when it happens to us.
But, when you want to give up, remind yourself that loss of hope is
temporary. It might just compel you to pick yourself up and move forward.
Use failures as your motivations. Failure isn’t the end of a journey, instead
it is just an experience you have while on your way to success.

Wishing you the best for every step in your journey. Go and conquer your
dreams.

I am always there for you, and I know that my champions can never give
up

With love and lots of blessings


Nimisha Bansal

Use Code NIMISHA for maximum Discount

For any query call or whatsapp 9953592800


10

TOPPER’S TALK

Ishika Saini (SBI PO 2021-2022)

1. Congratulations on your success. I'm very happy for you and I’m
proud of you. What was the reaction of your parents and you when
you saw the result?

ISHIKA SAINI : Thank You so much mam! It was really unexpected,


there were tears of happiness in my eyes. When I saw the result I couldn't
believe it was real. The feeling cannot be expressed. My parents are
really proud of me. I cleared it in the 1st attempt. I had been preparing
since October, 2020 but I joined your perfection batch in May.

2. Tell me more about your goals, strategies and how did you
prepare?

ISHIKA SAINI : I mainly focused on the things I had command on like


English then I tried to clear my mathematics concepts and reasoning.
The biggest factor that helped me were the mocks tests, I improved my
speed of solving questions by giving mock tests.

3. How important and instrumental have Nimisha ma'am's various


batches been in your preparation for the banking exams? How will
you consider her guidance and contributions?

Use Code NIMISHA for maximum Discount

For any query call or whatsapp 9953592800


11

A: I've been to many English classes on many platforms so far. But the
kind of ambience she creates in her classes has been unmatchable. Her
flagship batch - Perfection Batch has been enormously helpful for me and
thousands like me, who always used to think that English is the hardest
section. When I started attending her classes, I realized that solving only
the easy and simple questions would not yield any positive result. The
level of her questions in the Perfection Batch is so advanced that I
experienced a different dimension of learning. I acquired a great deal of
knowledge. My reading skill was improved to a great extent.
I'm also a regular attendee of her other free classes and editorial analysis
classes. These also have big contributions to my preparation.
Apart from her wonderful teaching ability, her warm gestures and constant
motivation are the things, I'm forever indebted to.
4. Many students ask me about their doubts about the examinations,
like is reading enough for English?

ISHIKA SAINI : I don't think only reading can improve the language, you
have to solve questions to improve the same. Reading editorials does
help but only reading them isn't enough. I really enjoyed attending the
perfection batch and it helped me a lot and improved my weak links.

5. How was your interview? Were you nervous enough to face the
interviewers?

ISHIKA SAINI : I was nervous till I was outside the door waiting for my
turn. As soon as I got inside, I thought the interviewers were also human.
They won’t do anything if I answer wrong, so my nervousness flew away.
My interview was mostly situation based; it wasn't very much about
banking. I remember they asked me about the difference between SBI
and PNB. In which bank do you have your account? Most questions were
from my introduction and hobbies.

6. How did you manage your time? What was your timetable?
Use Code NIMISHA for maximum Discount

For any query call or whatsapp 9953592800


12

ISHIKA SAINI : My target was to study 7 - 8 hours a day. I used to give


most of my time to English, like 4 -5 hours, as I knew that English was the
most scoring subject. When my reasoning and mathematics concepts got
clear, I was totally dependent upon mock tests. I did not take any extra
classes , I learned the concepts and practised them. My general
awareness was very weak. I used to study it on youtube. I believe that
English is a game changing subject because my marks increased
because of it and descriptive writing.

7. What would you like to say to fellow aspirants?

ISHIKA SAINI : My aim was to clear this exam. So I completed my


graduation and got dedicated to preparing for the exam. Just make a
mindset that this is the easiest exam and I can do it. Just train your mind
and when you will be in the exam hall solving the paper you will feel the
energy. Just stay positive. Don't ever lose hope and work whole-heartedly
towards your goals.

Use Code NIMISHA for maximum Discount

For any query call or whatsapp 9953592800


13

INDEX

S.N0. CHAPTER NAME PAGE NO


1 PARAJUMBLE 14- 69
2 FILLERS 70- 106
3 CONNECTOR 107- 121
4 CLOZE TEST 122- 166
5 WORD REARRANGEMENT 167-225
6 PHRASE BASED QUESTIONS 226-242
7 READING COMPREHENSION 243-270
8 PARAGRAPH BASED QUESTION 271-282
9 MISCELLANEOUS PATTERN 283-350

Use Code NIMISHA for maximum Discount

For any query call or whatsapp 9953592800


14

CHAPTER 1 - PARAJUMBLE
Direction(1-10): Below are some statements, given in a random order. When these
statements are sequenced properly, they'll form a coherent and meaningful
paragraph.
There are various questions based on each of the statements. Read every statement
carefully and answer the questions that follow.

The statement in bold is fixed and the SIXTH statement.

(a) The Centre had set the goal of spending 2.5% of the GDP in healthcare in its 2017
national health policy but the actual proportion of expenditure increased from 1.15% in 2013-
14 to 1.28% in 2018-19; the share of out-of-pocket(OoP) healthcare spending went down
from 64.2 per cent in 2013-14 to 48.2 in 2018-19.

(b) One of the principal findings of the report is that government spending as a proportion of
the country’s gross domestic product has fallen to 1.28% from 1.35%; the total health
spending — expenditure by government and non-government agents — declined from 3.9%
of the GDP to 3.2%.

(c) Moreover, it must be noted that the NHA data come after a significant lag and that the
pandemic may have led to an increase in OoP expenditure.

(d) Could it be that people who need to seek help for healthcare are unwilling to do so
because of the general economic distress?

(e) This is extremely __________ in a country where 55 million, according to WHO, were
pushed into __________ to meet health expenses.

(f) The methodology followed to calculate OoP expenditure must be shared and
deliberated upon.

(g) Neither the Centre nor the states must lose sight of the fact that healthcare infrastructure
continues to be uneven while the disease burden is high — 61 per cent of total deaths in
India can be attributed to lifestyle or non-communicable diseases; Universal, accessible
healthcare is a priority.

Use Code NIMISHA for maximum Discount

For any query call or whatsapp 9953592800


15

(h) Healthcare spending in India continues to be a matter of concern; according to the latest
National Health Accounts report, India’s per capita healthcare expenditure at constant prices
has remained near-stagnant in spite of increases in government spending and decreases in
household expenditure over a six-year period.

(a) The Centre had set the goal of spending 2.5% of the GDP in healthcare in its 2017
national health policy but the actual proportion of expenditure increased from 1.15% in 2013-
14 to 1.28% in 2018-19; the share of out-of-pocket(OoP) healthcare spending went down
from 64.2 per cent in 2013-14 to 48.2 in 2018-19.

1. Which of the following possible statements can be inferred from the above
statement?

a. There has possibly been an increase in utilisation and reduction in cost of services in
government health facilities.
b. The very consumption of healthcare and other social services has plummeted.
c. India's population’s access to medicines is not restricted.

A. Only a and b B. Only a and c C. Only b and c D. All a, b and c E. None of a, b


and c

(b) One of the principal findings of the report is that government spending as a proportion of
the country’s gross domestic product has fallen to 1.28% from 1.35%; the total health
spending — expenditure by government and non-government agents — declined from 3.9%
of the GDP to 3.2%.

2. Which of the following statements regarding the above context may create
apprehensions?

a. The chronic under-funding of the healthcare sector affects the poorest.


b. An efficient and equitable health system cannot be developed because there's no
adequate financing.
c. The government expenditure on health should be increased to 2.5% of GDP by 2025.

Use Code NIMISHA for maximum Discount

For any query call or whatsapp 9953592800


16

A. Only a and b B. Only a and c C. Only b and c D. All a, b and c E. None of a, b


and c

(c) Moreover, it must be noted that the NHA data come after a significant lag and that the
pandemic may have led to an increase in OoP expenditure.

3. If you have to make a statement that will elaborate more on the above context,
which of the following statements can be chosen?

a. COVID-19 infection resulted in significant economic implications to patients as well as


a considerable financial burden to the general population for preventive measures.
b. Due to the pandemic, the share of medical spending falling to patients started being
on an upward trajectory.
c. During COVID-19, patients put off non-essential care and regulatory waivers that
eliminated cost-sharing for virtual visits.

A. Only a and b B. Only a and c C. Only b and c D. All a, b and c E. None of a, b


and c

(d) Could it be that people who need to seek help for healthcare are unwilling to do so
because of the general economic distress?

4. Which of the following phrases can most appropriately be derived from the above
statement?

a. Being unable to meet financial obligations


b. Not have the stomach to do something
c. A lump in one's throat

A. Only a and b B. Only a and c C. Only b and c D. All a, b and c E. None of a, b


and c

5. Which of the following combinations is the combination of THREE CONSECUTIVE


statements after the correct rearrangement?

Use Code NIMISHA for maximum Discount

For any query call or whatsapp 9953592800


17

A. b a d
B. e d f
C. a e c
D. d c g
E. None of the above options

(e) This is extremely __________ in a country where 55 million, according to WHO, were
pushed into __________ to meet health expenses.

6. Which of the following pairs of words can most appropriately fit in the blanks in the
above statement?

a. worrying, penury
b. perturbing, lucre
c. concerning, indigence

A. Only a and b B. Only a and c C. Only b and c D. All a, b and c E. None of a, b


and c

(f) The methodology followed to calculate OoP expenditure must be shared and deliberated
upon.

7. Which of the following components associated with an individual or a household


may contribute to the methodology followed to calculate OoP expenditure?

a. Complete information on costs of medicine and doctors.


b. Total health care expenditure and income.
c. The association between state net domestic product per capita and component share
of each health care service.

A. Only a and b B. Only a and c C. Only b and c D. All a, b and c E. None of a, b


and c

Use Code NIMISHA for maximum Discount

For any query call or whatsapp 9953592800


18

(g) Neither the Centre nor the states must lose sight of the fact that healthcare infrastructure
continues to be uneven while the disease burden is high — 61 per cent of total deaths in
India can be attributed to lifestyle or non-communicable diseases; Universal, accessible
healthcare is a priority.

8. Which of the following statements can be derived from the above statement?

a. Limited national progress has been made in the fight against non-communicable
diseases.
b. More than 50% of deaths in India are now attributed to diseases like heart disorders,
cancer and diabetes.
c. People from all spheres must have the ability to obtain healthcare services such as
prevention, diagnosis, treatment, and management of diseases, illness, disorders,
and other health-impacting conditions in a very accessible manner.

A. Only a and b B. Only a and c C. Only b and c D. All a, b and c E. None of a, b


and c

(h) Healthcare spending in India continues to be a matter of concern; according to the latest
National Health Accounts report, India’s per capita healthcare expenditure at constant prices
has remained near-stagnant in spite of increases in government spending and decreases in
household expenditure over a six-year period.

9. How would you describe the author's tone in the above statement?

a. The author is having kittens


b. The author is being malcontent
c. The author is being frenzied

A. Only a and b B. Only a and c C. Only b and c D. All a, b and c E. None of a, b


and c

10. Which of the following combinations is the combination of THREE CONSECUTIVE


statements after the correct rearrangement?

Use Code NIMISHA for maximum Discount

For any query call or whatsapp 9953592800


19

A. b a d
B. e d f
C. a e c
D. d c g
E. None of the above options

Direction(11-17): Below are some statements, given in a random order. When these
statements are sequenced properly, they'll form a coherent and meaningful
paragraph.
There are two blanks in each of the statements. Read every statement carefully and
answer the questions that follow.

(a) This is an ______ ______ itself.

(b) In the “far from normal” last two and half years, when _______ came the closest to living
in a science fiction, India and Indians under the untiring leadership of Prime Minister
Narendra Modi can ________ claim to have learnt crucial lessons.

(c) That’s no small achievement, particularly amid rapidly ________ realities and constantly
evolving science, and given social _______ and vaccine hesitancy in so many different
countries.

(d) We ________ ________, determination and strength that has earned respect the world
over.

(e) Be it the vaccine manufacturing prowess of India; be it innovations trials in R&D; be it


_________ the potential of public-private partnerships, we can claim to have _________
lessons of decades into two years to succeed on many fronts.

(f) Such a claim can be ______ for the vaccine ______ too.

(a) This is an ______ ______ itself.

Use Code NIMISHA for maximum Discount

For any query call or whatsapp 9953592800


20

11. Which of the following pairs of words can most appropriately fit in the blanks in
the above statement respectively?

a. accomplishment, unto
b. master stroke, for
c. triumph, in

A. Only a and b B. Only a and c C. Only b and c D. All a, b and c E. None of a, b


and c

(b) In the “far from normal” last two and half years, when _______ came the closest to living
in a science fiction, India and Indians under the untiring leadership of Prime Minister
Narendra Modi can ________ claim to have learnt crucial lessons.

12. Which of the following pairs of words can most appropriately fit in the blanks in
the above statement respectively?

a. humankind, collectively
b. humanity, as a group
c. mankind, as a whole

A. Only a and b B. Only a and c C. Only b and c D. All a, b and c E. None of a, b


and c

(c) That’s no small achievement, particularly amid rapidly ________ realities and constantly
evolving science, and given social _______ and vaccine hesitancy in so many different
countries.

13. Which of the following pairs of words can most appropriately fit in the blanks in
the above statement respectively?

a. shifting, discord

Use Code NIMISHA for maximum Discount

For any query call or whatsapp 9953592800


21

b. variable, strife
c. changeless, dissension

A. Only a and b B. Only a and c C. Only b and c D. All a, b and c E. None of a, b


and c

(d) We ________ ________, determination and strength that has earned respect the world
over.

14. Which of the following pairs of words can most appropriately fit in the blanks in
the above statement respectively?

a. demonstrated, resilience
b. exemplified, toughness
c. displayed, frailty

A. Only a and b B. Only a and c C. Only b and c D. All a, b and c E. None of a, b


and c

(e) Be it the vaccine manufacturing prowess of India; be it innovations trials in R&D; be it


_________ the potential of public-private partnerships, we can claim to have _________
lessons of decades into two years to succeed on many fronts.

15. Which of the following pairs of words can most appropriately fit in the blanks in
the above statement respectively?

a. unleashing, put together


b. untethering, tracked together
c. unbridling, accumulated

A. Only a and b B. Only a and c C. Only b and c D. All a, b and c E. None of a, b


and c

Use Code NIMISHA for maximum Discount

For any query call or whatsapp 9953592800


22

(f) Such a claim can be ______ for the vaccine ______ too.

16. Which of the following pairs of words can most appropriately fit in the blanks in
the above statement respectively?

a. made, space
b. produced, extent
c. receded, scope

A. Only a and b B. Only a and c C. Only b and c D. All a, b and c E. None of a, b


and c

(a) This is an accomplishment unto itself.

(b) In the “far from normal” last two and half years, when humanity came the closest to
living in a science fiction, India and Indians under the untiring leadership of Prime Minister
Narendra Modi can collectively claim to have learnt crucial lessons.

(c) That’s no small achievement, particularly amid rapidly shifting realities and constantly
evolving science, and given social discord and vaccine hesitancy in so many different
countries.

(d) We demonstrated resilience, determination and strength that has earned respect the
world over.

(e) Be it the vaccine manufacturing prowess of India; be it innovations trials in R&D; be it


unleashing the potential of public-private partnerships, we can claim to have put together
lessons of decades into two years to succeed on many fronts.

(f) Such a claim can be made for the vaccine space too.

17. Which of the following options is the pair of the TERMINAL statement and the
INTRODUCTORY statement, respectively, after the correct rearrangement?

Use Code NIMISHA for maximum Discount

For any query call or whatsapp 9953592800


23

A. c f
B. a e
C. c b
D. a c
E. None of the above options

Direction(18-24): Below are some statements, given in a random order. When these
statements are sequenced properly, they'll form a coherent and meaningful
paragraph.
There are TWO BLANKS in each of the statements. Read every statement carefully
and answer the questions that follow.

(a) Needless to say, right now, ________ and ________ are worryingly high.

(b) And the rupee-dollar exchange rate _____ a record low to _____ Friday’s trading at Rs
80.99.

(c) The main sources of risk are, first, the unclear ________ of major central banks trying to
_______ inflation and, second, fluid geopolitics.

(d) RBI’s monetary policy committee, which will have its bimonthly meeting next week, has
to _______ with risks coming largely from ________.

(e) Indian financial markets were ______ on Friday, in line ______ major global markets.

(f) The BSE Sensex ______ lower ____ 1.7% at 58,098.92.

(a) Needless to say, right now, ________ and ________ are worryingly high.

18. Which of the following pairs of words can most appropriately fit in the blanks in
the above statement respectively?

a. uncertainty, volatility

Use Code NIMISHA for maximum Discount

For any query call or whatsapp 9953592800


24

b. precariousness, unpredictability
c. credence, changeability

A. Only a and b B. Only a and c C. Only b and c D. All a, b and c E. None of a, b


and c

(b) And the rupee-dollar exchange rate _____ a record low to _____ Friday’s trading at Rs
80.99.

19. Which of the following pairs of words can most appropriately fit in the blanks in
the above statement respectively?

a. hit, end
b. struck, break off
c. knocked, conclude

A. Only a and b B. Only a and c C. Only b and c D. All a, b and c E. None of a, b


and c

(c) The main sources of risk are, first, the unclear ________ of major central banks trying to
_______ inflation and, second, fluid geopolitics.

20. Which of the following pairs of words can most appropriately fit in the blanks in
the above statement respectively?

a. trajectory, pull back


b. approach, withdraw
c. orientation, set forth

A. Only a and b B. Only a and c C. Only b and c D. All a, b and c E. None of a, b


and c

Use Code NIMISHA for maximum Discount

For any query call or whatsapp 9953592800


25

(d) RBI’s monetary policy committee, which will have its bimonthly meeting next week, has
to _______ with risks coming largely from ________.

21. Which of the following pairs of words can most appropriately fit in the blanks in
the above statement respectively?

a. contend, overseas
b. grapple, abroad
c. take, oversea

A. Only a and b B. Only a and c C. Only b and c D. All a, b and c E. None of a, b


and c

(e) Indian financial markets were ______ on Friday, in line ______ major global markets.

22. Which of the following pairs of words can most appropriately fit in the blanks in
the above statement respectively?

a. battered, with
b. thrashed, by
c. thumped, with

A. Only a and b B. Only a and c C. Only b and c D. All a, b and c E. None of a, b


and c

(f) The BSE Sensex ______ lower ____ 1.7% at 58,098.92.

23. Which of the following pairs of words can most appropriately fit in the blanks in
the above statement respectively?

a. closed, by
b. concluded, by
c. winded up, by

Use Code NIMISHA for maximum Discount

For any query call or whatsapp 9953592800


26

A. Only a and b B. Only a and c C. Only b and c D. All a, b and c E. None of a, b


and c

24. Which of the following options is the pair of the TERMINAL statement and the
INTRODUCTORY statement, respectively, after the correct rearrangement?

A. c f
B. a e
C. c d
D. a c
E. None of the above options

Directions (25-37): Five statements are given below, labelled a, b, c, d and e, among
these, four statements are in logical order and form a coherent paragraph/passage.
From the given options, choose the option that does not fit into the theme of the
passage.

Q25.
(a)If there is only one idea that Gandhi should be remembered for and identified with, it is
the idea of empowerment of the other.

(b)Mahatma Gandhi’s death anniversary (January 30) may have just passed, but it is not just
an occasion to celebrate his life and his message once more, or to simply add to the
Gandhian biography, so to speak.

(c)Therefore, the task before us is how we, individually and collectively, can understand and
take forward the Gandhian nobility of spirit in today’s world.

(d)The spirit of democracy requires the inculcation of the spirit of brotherhood... democracy
is not a state in which people act like sheep.

(e)We should think of Gandhi as a noble spirit who continues to be among us and who
contributes to the betterment of our world.

Use Code NIMISHA for maximum Discount

For any query call or whatsapp 9953592800


27

Q26.
(a)There are large variations across States in the way the MSP-led procurement operations
are conducted; but there are also variations in States in terms of the role and functions of
the Agricultural Produce Market Committee.

(b)But it is yet unclear as to how the peaceful protest for the last two months was allowed to
take a violent turn by the farmers’ union leadership as well as the police administration.

(c)The deadlock between protesting farmers and the government on the contentious farm
laws have taken an ugly turn, with incidents of vandalism during the tractor parade by the
protesting farmers on January 26.

(d)Attempts by a section of media and representatives of political parties to use the incidents
of vandalism to vilify and malign the two-month-old peaceful agitation will only add to the
mistrust between the government and the farmer unions.

(e)The responsibility must be fixed for the administrative lapses and action taken against
erring antisocial elements, the events again point out to the growing mistrust and breakdown
of any dialogue between the two sides.

Q27.
(a)The Economic Survey for 2020-21 is an expansive attempt at reviewing the
developments in the Indian economy during the current financial year and providing an
outlook for its near-term prospects.

(b)It also cites an approach that used ‘graded public health measures to transform the short-
term trade-off between lives and livelihoods into a win-win that would save both lives and
livelihoods over the longer term’.
(c)Spread over 700 pages, the survey highlights the policy achievements of the government
in steering the economy through the treacherous shoals of “the most unfathomable global
health emergency experienced in modern history”.

(d)The survey asserts that through the approach India established a globally unique model
of strategic policymaking in containing the COVID-19 pandemic while helping the economy
recover quickly from its deleterious impact.

Use Code NIMISHA for maximum Discount

For any query call or whatsapp 9953592800


28

(e)Depreciation, debt write-offs, tax credits and other measures are among the tax
avoidance benefits inherent in the sector.

Q28.
(a)The S-400 defense system is one of the world’s most sophisticated, and the agreement
underscored Russia’s legacy as one of India’s major defense suppliers over the past several
decades.

(b)But American officials assert the deal violates U.S. law, specifically the “Countering
America’s Adversaries through Sanctions Act” (“CAATSA”).

(c)Dr. Narang’s description of the “complicated relations” between Turkey and the U.S.
underlines certain key differences between the Turkish and Indian cases in terms of
American sanctions.

(d)Enacted in 2017 with overwhelming bipartisan support, the CCATSA legislation imposed
sweeping sanctions against Moscow, while also effectively authorizing secondary sanctions
against any other country engaging in “significant transactions” with Russia’s defense or
intelligence sectors.

(e)In October 2018, India agreed to purchase five Russian S-400 self-propelled surface-to-
air systems for approximately $5.4 billion during a state visit by President Vladimir Putin.

Q29.
(a)In February 2020, he pledged $10 billion—almost 10% of his net worth at the time—to
fight climate change through an initiative called the Bezos Earth Fund.

(b)Bezos made headlines in 2019 with his high-profile divorce from MacKenzie Scott, his
wife of 25 years, in which he agreed to give her a quarter of his Amazon stake.

(c)Bezos started the company that would revolutionize e-commerce as an online bookseller
in his Seattle garage in 1994.

(d) When you look at our financial results, what you’re actually seeing are the long-run
cumulative results of invention.

(e)Some 30 years later Amazon’s businesses span cloud storage, video streaming,
groceries and more.

Use Code NIMISHA for maximum Discount

For any query call or whatsapp 9953592800


29

Q30.
(a)Whether natural or manmade, catastrophes disrupt everyday life.

(b)And in such an emergency, finding a place to live can be an urgent, difficult decision.

(c)As a result, routine chores, such as having medications delivered or trips to the grocery
store, are suddenly impossible.

(d)Electric power may go down, cell towers may not work, water could be contaminated, and
homes might be evacuated.

(e)Our public health system is really not supported for older adults.

Q31.
(a)And she warns against doing business with fraudulent contractors who often swarm
through damaged neighborhoods right after a hurricane.
(b)Even if FEMA gets additional funding for it, the money would mark a small down payment
on the much larger challenge of improving disaster planning and preparedness for older
people.

(c)Day counsels older people to realize it will take time for insurance claims to be processed.

(d)Day is the local AARP chapter president in north Florida and recommends disaster
prevention through attending educational meetings with local emergency management
teams.

(e)She has plenty of advice of her own, too.

Q32.
(a)Poorly trained in digital forensics and, sometimes, compromised, law-enforcement
authorities have been complicit in the lack of action on a number of reported social media
threats, particularly against women.

(b)Mass-scale educational and economic deprivation with deep-seated ethnic, religious,


economic and social fault lines abound in a toxic ecosystem that can be exploited easily.

Use Code NIMISHA for maximum Discount

For any query call or whatsapp 9953592800


30

(c)Forewarnings about social media’s potential to inflict greater social harm in Myanmar
were repeatedly made, including by leaders of the tech industry.

(d)One cannot adopt a one-size-fits-all approach as the challenges in developing countries


are multi-layered compared to those in advanced countries.

(e)This is coupled with the lack of institutional autonomy of civil servants compared to
countries like the US.

Q33.
(a)There is a possibility that the script would unfold differently now.

(b)After 1990, the military’s intervention had plunged democracy into darkness for a while.

(c)Myanmar occupies a critical geo-strategic niche, being the gateway to the restive Indo-
Pacific stretch that is witnessing the Great Game at play.

(d)The coup has thrown the rehabilitation process of the Rohingyas into jeopardy, causing
further consternation in the international community.

(e)It is, therefore, unlikely that the global powers would remain passive about these
developments.

Q34.
(a)Jackson Reffitt assumed his father was planning “something big” in the lead-up to US
President Joe Biden’s oath-taking.

(b)One that flutters in the sky to remind us elders of our miserable failings, telling us that the
choice between right and wrong is easy if we want it to be.

(c)On the eve of India’s Republic Day, they got together to celebrate the Constitution, the
document the nation bequeathed itself 71 years ago.

(d)As the world lurches towards self-defeating extreme intolerance from one day to the next,
these boys and girls have planted their own flag.

Use Code NIMISHA for maximum Discount

For any query call or whatsapp 9953592800


31

(e)In Calcutta, a group of children had their own lesson to impart at school.

Q35.
(a)There is little doubt that when the pandemic is under control, more people will return to
clean and green mass mobility.

(b)While advanced countries have turned their back on the need of poor countries to access
COVID-19 vaccines, India has displayed empathy to their needs.

(c)Although COVID-19 has had the perverse effect of driving people away to the safety of
personal car and two-wheeler bubbles.
(d)Comfortable, safe and affordable commuting has well-recognised multiplier effects for the
economy and more generally for public health.

(e)With a clear focus on expansion of Metro Rail and bus services through Central funding,
Budget 2021 has recognised a core component of urbanisation.

Q36.
(a)A survey found that in 2020 stranded migrants had to pay several times more than normal
fares to go back home — may discourage them from exercising their franchise.

(b)It has been presumed that the migrant workers could get their ration from any fair price
shop on their ration cards.

(c)A large segment of this population is underprivileged.

(d)To this end, data from the Inter-State Migrant Workmen Act, 1979 requires all
establishments hiring inter-state migrants to be registered and used to create a
comprehensive database as well as a legal identity.

(e)The paucity of a clear legal definition of ‘migrant worker’ also gets in the way of identifying
the segment for voting facilities.

Q37.
(a)Treating fever can prolong or worsen illness.

Use Code NIMISHA for maximum Discount

For any query call or whatsapp 9953592800


32

(b)Thus, when you take medication such as acetaminophen or ibuprofen to suppress a


fever, you actually work against the inherent protective benefits nature bestowed.

(c)Yes, a fever reducer would probably make you feel better, relieving symptoms such as
headache, muscle aches and fatigue.

(d) She suggested that doctors warn parents to expect a nighttime rise in a child’s fever and
explain that high fevers from an infection are not damaging.

(e)But you’re not supposed to feel better and you’re supposed to stay under the covers,
keep warm and ride out the infection”, not go out and spread it to others.

Direction (38-42): The question consists of five statements labelled A, B, C, D, and E


which when logically ordered form a coherent passage. Choose the option that
represents the most logical order.

(A)Decades ago, mundane email enabled virtually instantaneous communication around the
world.

(B)The entire internet technology wave, for example, was a communications paradigm
rooted in sending messages.

(C)And the thirty years of consumer internet thus far have represented the generalization of
that paradigm.

(D)Every great technology wave has a paradigm, a very broad and deep movement that is
only barely perceptible in its infancy.

(E)This paradigm typically blossoms into many forms over years, even decades, to come.

Q38. Which among the following will be the fourth sentence of the paragraph after the
rearrangement?
(a) D
(b) C
(c) E
(d) A
(e) B

Use Code NIMISHA for maximum Discount

For any query call or whatsapp 9953592800


33

Q39. Which among the following will be the second sentence of the paragraph after the
rearrangement?
(a) B
(b) C
(c) E
(d) D
(e) A

Q40. Which among the following will be the fifth sentence of the paragraph after the
rearrangement?
(a) A
(b) C
(c) D
(d) B
(e) E

Q41. Which among the following will be the FIRST sentence of the paragraph after the
rearrangement?
(a) B
(b) D
(c) C
(d) E
(e) A

Q42. Which among the following will be the third sentence of the paragraph after the
rearrangement?
(a) A
(b) D
(c) E
(d) B
(e) C
Direction (43-47): The question consists of five statements labelled A, B, C, D, and E
which when logically ordered form a coherent passage. Choose the option that
represents the most logical order.

Use Code NIMISHA for maximum Discount

For any query call or whatsapp 9953592800


34

(A) It allows them to make reference to deep-layer technologies like the neural networks that
are building the Artificial Intelligence (AI) brains of the immediate future.

(B)Users today are often more focused on which app, which device, which online web
service and which user interface option they’re going to have to play with.

(C) They are ignoring the provenance, progeny and posterity of the data streams that feed
all of the upper-tier technology layers that they actually touch every day.

(D)The unfortunate side effect of data being so prevalent, populous and all-pervading in
modern IT systems is that data has sometimes become a throwaway term in some senses.

(E)Technology evangelists love data and talking about data makes them sound smart and
considered.

Q43. Which among the following will be the fourth sentence of the paragraph after the
rearrangement?
(a) D
(b) C
(c) E
(d) A
(e) B

Q44. Which among the following will be the second sentence of the paragraph after the
rearrangement?
(a) B
(b) C
(c) E
(d) D
(e) A

Q45. Which among the following will be the fifth sentence of the paragraph after the
rearrangement?
(a) A
(b) C
(c) D

Use Code NIMISHA for maximum Discount

For any query call or whatsapp 9953592800


35

(d) B
(e) E

Q46. Which among the following will be the FIRST sentence of the paragraph after the
rearrangement?
(a) B
(b) D
(c) C
(d) E
(e) A

Q47. Which among the following will be the third sentence of the paragraph after the
rearrangement?
(a) F
(b) D
(c) E
(d) B
(e) C

Direction (48-52): The question consists of five statements labelled A, B, C, D, and E


which when logically ordered form a coherent passage. Choose the option that
represents the most logical order.

(A) A recent report from Checkpoint Security notes that ransomware attacks against
healthcare orgs have jumped about 45% since early November.

(B)That followed an alarming 71% spike in October.

(C)In the spring of last year a number of criminal hacker crews pledged to leave hospitals,
nursing homes and other healthcare entities alone until the Covid-19 pandemic passed.

(D) Checkpoint notes that healthcare entities were actually the number one target of
ransomware attacks that month.

(E)At least one ransomware gang saw that as an opportunity.

Q48. Which among the following will be the fourth sentence of the paragraph after the
rearrangement?

Use Code NIMISHA for maximum Discount

For any query call or whatsapp 9953592800


36

(a) D
(b) C
(c) E
(d) A
(e) B

Q49. Which among the following will be the second sentence of the paragraph after the
rearrangement?
(a) B
(b) C
(c) E
(d) D
(e) A

Q50. Which among the following will be the fifth sentence of the paragraph after the
rearrangement?
(a) A
(b) C
(c) D
(d) B
(e) E

Q51. Which among the following will be the FIRST sentence of the paragraph after the
rearrangement?
(a) B
(b) C
(c) D
(d) E
(e) A

Q52. Which among the following will be the third sentence of the paragraph after the
rearrangement?
(a) A
(b) D
(c) E

Use Code NIMISHA for maximum Discount

For any query call or whatsapp 9953592800


37

(d) B
(e) C

Direction (53-57): The question consists of five statements labelled A, B, C, D, and E


which when logically ordered form a coherent passage. Choose the option that
represents the most logical order.

(A) The Biden plan proposes the reinstatement of the Pease Limitation on itemized
deductions.

(B) In order for this plan to become a reality, it still needs to pass Congress and most likely
will have extensive amendments.

(C) Additionally, the tax plan seems to reduce the effective tax benefits of these deductions
from 39.6% to 28% for those earning more than $400,000 of adjusted gross income.

(D) President-elect Joe Biden's policies and tax plans are becoming increasingly more
relevant as his time in office begins.

(E) The Biden plan was published as a component of the platform for the Democratic Party.

Q53. Which among the following will be the fourth sentence of the paragraph after the
rearrangement?
(a) D
(b) C
(c) E
(d) A
(e) B

Q54. Which among the following will be the second sentence of the paragraph after the
rearrangement?
(a) B
(b) C
(c) E
(d) D
(e) A

Use Code NIMISHA for maximum Discount

For any query call or whatsapp 9953592800


38

Q55. Which among the following will be the fifth sentence of the paragraph after the
rearrangement?
(a) A
(b) C
(c) D
(d) B
(e) E

Q56. Which among the following will be the FIRST sentence of the paragraph after the
rearrangement?
(a) B
(b) D
(c) C
(d) E
(e) A

Q57. Which among the following will be the third sentence of the paragraph after the
rearrangement?
(a) A
(b) D
(c) E
(d) B
(e) C

Parajumble based on Phrase Derivation

Direction(58-63): Below are some statements, given in a random order. When these
statements are sequenced properly, they'll form a coherent and meaningful
paragraph.

There are various questions based on each of the statements. Read every statement
carefully and answer the questions that follow.

(a) In fact, Russia-ally Belarus today ordered joint deployment of troops with Russian forces
near Ukraine.

Use Code NIMISHA for maximum Discount

For any query call or whatsapp 9953592800


39

(b) Although Ukraine hasn’t officially claimed responsibility for the bombing of the bridge,
Russia’s response marks a further escalation in the war.

(c) In what were clearly revenge strikes, Russian forces targeted multiple Ukrainian cities
today with a barrage of cruise missiles during peak working hours.

(d) If Belarus forces actively join the war against Ukraine, it will certainly take the conflict to a
whole new level.

(e) This comes after the destruction of a section of the Russia-made Crimea bridge over the
weekend that Vladimir Putin described as an act of terrorism.

(a) In fact, Russia-ally Belarus today ordered joint deployment of troops with Russian forces
near Ukraine.

58. Which of the following phrases can contextually be derived from the above
statement?

a. pooling resources
b. giving the command to
c. laying down

A. Only a and b B. Only a and c C. Only b and c D. All a, b and c E. None of a, b


and c

(b) Although Ukraine hasn’t officially claimed responsibility for the bombing of the bridge,
Russia’s response marks a further escalation in the war.

59. Which of the following phrases can contextually be derived from the above
statement?

a. an increase in the intensity


b. not bringing into the open
c. looking the other way

Use Code NIMISHA for maximum Discount

For any query call or whatsapp 9953592800


40

A. Only a and b B. Only a and c C. Only b and c D. All a, b and c E. None of a, b


and c

(c) In what were clearly revenge strikes, Russian forces targeted multiple Ukrainian cities
today with a barrage of cruise missiles during peak working hours.

60. Which of the following phrases can contextually be derived from the above
statement?

a. beginning hostilities
b. clear-cut
c. wheeling and dealing

A. Only a and b B. Only a and c C. Only b and c D. All a, b and c E. None of a, b


and c

(d) If Belarus forces actively join the war against Ukraine, it will certainly take the conflict to a
whole new level.

61. Which of the following phrases can contextually be derived from the above
statement?

a. in the offing
b. taking in hand
c. sea change

A. Only a and b B. Only a and c C. Only b and c D. All a, b and c E. None of a, b


and c

(e) This comes after the destruction of a section of the Russia-made Crimea bridge over the
weekend that Vladimir Putin described as an act of terrorism.

Use Code NIMISHA for maximum Discount

For any query call or whatsapp 9953592800


41

62. Which of the following phrases can contextually be derived from the above
statement?

a. after-effects
b. pulling down
c. freedom from interference

A. Only a and b B. Only a and c C. Only b and c D. All a, b and c E. None of a, b


and c

63. Which of the following options is the pair of the TERMINAL statement and the
INTRODUCTORY statement, respectively, after the correct rearrangement?

A. c d
B. a e
C. d c
D. a c
E. None of the above options

Parajumble based on Word Swap

Direction(64-71): Below are some statements, given in a random order. When these
statements are sequenced properly, they'll form a coherent and meaningful
paragraph.

There are various questions based on each of the statements. Read every statement
carefully and answer the questions that follow.

(a) Is it the government’s (m)mortality that India’s children with stunted


(n)undernourishment make up approximately one-third the number in the whole world, or
that the Global Hunger Index (2021) places India 101st among 116 countries by virtue of
children’s (o)maturation, stunted growth and (p)consideration?

Use Code NIMISHA for maximum Discount

For any query call or whatsapp 9953592800


42

(b) India was never markedly kind to its children; but society’s (m)veneer seems to have
been overtaken by the (n)dispensation, which appears to be uninterested in maintaining
even a (o)vexation of (p)callosity.

(c) It announced a (m)stratagem of 9.6 per cent in the material (n)predicament of school
midday meals up to Class VIII; this is after two years of unprecedented (o)swelling for
teachers tasked with implementing the (p)valuation.

(d) As it is, the midday meal scheme, although one of the most fruitful approaches in linking
(m)delinquency and education, is plagued by problems ranging from (n)seating — stealing
children’s food — to caste (o)sustenance, the latter relating to the caste of cooks on the
one hand to children’s caste on the other, with teachers discriminating between them in
(p)partisanship and the portions served.

(e) Yet the increase is so stingy as to be a (m)repast, according to a (n)value in West


Bengal; its (o)preceptor is less than a rupee for a child’s (p)mockery, coming to Rs 5.45 for
a student in the primary class and Rs 8.17 for one above it.

(f) Apparently, children’s (m)noshes have nothing to do with (n)outlays in the (o)market —
of lentils or vegetables or eggs; they can eat whatever is available within the given
(p)wherewithal.

(g) The programme, therefore, demands additional (m)credo, even in terms of (n)well-
being, because the right to food and the right to (o)contemplation are integrally linked to
the right to (p)enlightenment, and they make up, together, the right to a happy and
productive life.

(a) Is it the government’s (m)mortality that India’s children with stunted


(n)undernourishment make up approximately one-third the number in the whole world, or
that the Global Hunger Index (2021) places India 101st among 116 countries by virtue of
children’s (o)maturation, stunted growth and (p)consideration?

64. Which of the following arrangements of the words can be chosen as the correct
arrangement?

A. p o n m
B. o n m p

Use Code NIMISHA for maximum Discount

For any query call or whatsapp 9953592800


43

C. p n o m
D. m p o n
E. No replacement required

(b) India was never markedly kind to its children; but society’s (m)veneer seems to have
been overtaken by the (n)dispensation, which appears to be uninterested in maintaining
even a (o)vexation of (p)callosity.

65. Which of the following arrangements of the words can be chosen as the correct
arrangement?

A. p o n m
B. o n m p
C. p n m o
D. m p o n
E. No replacement required

(c) It announced a (m)stratagem of 9.6 per cent in the material (n)predicament of school
midday meals up to Class VIII; this is after two years of unprecedented (o)swelling for
teachers tasked with implementing the (p)valuation.

66. Which of the following arrangements of the words can be chosen as the correct
arrangement?

A. p o n m
B. o n m p
C. p n m o
D. o p n m
E. No replacement required

(d) As it is, the midday meal scheme, although one of the most fruitful approaches in linking
(m)delinquency and education, is plagued by problems ranging from (n)seating — stealing
children’s food — to caste (o)sustenance, the latter relating to the caste of cooks on the

Use Code NIMISHA for maximum Discount

For any query call or whatsapp 9953592800


44

one hand to children’s caste on the other, with teachers discriminating between them in
(p)partisanship and the portions served.

67. Which of the following arrangements of the words can be chosen as the correct
arrangement?

A. o m p n
B. o n m p
C. p n m o
D. n p o m
E. No replacement required

(e) Yet the increase is so stingy as to be a (m)repast, according to a (n)value in West


Bengal; its (o)preceptor is less than a rupee for a child’s (p)mockery, coming to Rs 5.45 for
a student in the primary class and Rs 8.17 for one above it.

68. Which of the following arrangements of the words can be chosen as the correct
arrangement?

A. o m p n
B. o n m p
C. p o n m
D. n p o m
E. No replacement required

(f) Apparently, children’s (m)noshes have nothing to do with (n)outlays in the (o)market —
of lentils or vegetables or eggs; they can eat whatever is available within the given
(p)wherewithal.

69. Which of the following arrangements of the words can be chosen as the correct
arrangement?

A. o m p n
B. o n m p

Use Code NIMISHA for maximum Discount

For any query call or whatsapp 9953592800


45

C. p o n m
D. n p o m
E. No replacement required

(g) The programme, therefore, demands additional (m)credo, even in terms of (n)well-
being, because the right to food and the right to (o)contemplation are integrally linked to
the right to (p)enlightenment, and they make up, together, the right to a happy and
productive life.

70. Which of the following arrangements of the words can be chosen as the correct
arrangement?

A. o m p n
B. o n m p
C. p o n m
D. n p o m
E. No replacement required

(a) Is it the government’s consideration that India’s children with stunted maturation make
up approximately one-third the number in the whole world, or that the Global Hunger Index
(2021) places India 101st among 116 countries by virtue of children’s undernourishment,
stunted growth and mortality?

(b) India was never markedly kind to its children; but society’s callosity seems to have been
overtaken by the dispensation, which appears to be uninterested in maintaining even a
veneer of vexation.

(c) It announced a swelling of 9.6 per cent in the material valuation of school midday
meals up to Class VIII; this is after two years of unprecedented predicament for teachers
tasked with implementing the stratagem.

(d) As it is, the midday meal scheme, although one of the most fruitful approaches in linking
sustenance and education, is plagued by problems ranging from delinquency — stealing
children’s food — to caste partisanship, the latter relating to the caste of cooks on the one

Use Code NIMISHA for maximum Discount

For any query call or whatsapp 9953592800


46

hand to children’s caste on the other, with teachers discriminating between them in seating
and the portions served.

(e) Yet the increase is so stingy as to be a mockery, according to a preceptor in West


Bengal; its value is less than a rupee for a child’s repast, coming to Rs 5.45 for a student in
the primary class and Rs 8.17 for one above it.

(f) Apparently, children’s noshes have nothing to do with outlays in the market — of lentils
or vegetables or eggs; they can eat whatever is available within the given wherewithal.

(g) The programme, therefore, demands additional contemplation, even in terms of credo,
because the right to food and the right to enlightenment are integrally linked to the right to
well-being, and they make up, together, the right to a happy and productive life.

71. Which of the following combinations is the combination of FOUR CONSECUTIVE


STATEMENTS after the correct rearrangement?

A. b a c d
B. e f a d
C. a d g f
D. c e f d
E. None of the above options

Parajumble based on Double Fillers

Direction(72-77): Below are some statements, given in a random order. When these
statements are sequenced properly, they'll form a coherent and meaningful
paragraph.

There are various questions based on each of the statements. Read every statement
carefully and answer the questions that follow.

(a) Android has about 96% market _______ in India among mobile phones, which are the
most _______ access points to the internet.

(b) It’s a _______ in curbing _________ behavior in the digital economy.

Use Code NIMISHA for maximum Discount

For any query call or whatsapp 9953592800


47

(c) India’s competition regulator CCI on Thursday passed an order _______ a provisional
penalty of Rs 1,337.7 crore on Google for ________ the dominant position of its Android
mobile phone operating system.

(d) And its ______ comes from the scale of its impact in India and the global ______ too.

(e) CCI also asked Google to __________ a set of 10 remedial _________ that will open up
the mobile phone ecosystem.

(a) Android has about 96% market _______ in India among mobile phones, which are the
most _______ access points to the internet.

72. Which of the following pairs of words can most appropriately fit in the blanks in
the above statement respectively?

a. dividend, prevailing
b. portion, spurned
c. stake, widespread

A. Only a and b B. Only a and c C. Only b and c D. All a, b and c E. None of a, b


and c

(b) It’s a _______ in curbing _________ behavior in the digital economy.

73. Which of the following pairs of words can most appropriately fit in the blanks in
the above statement respectively?

a. milestone, monopolistic
b. milepost, complaisant
c. landmark, accommodative

A. Only a and c B. Only a C. Only b D. All a, b and c E. None of a, b and c

Use Code NIMISHA for maximum Discount

For any query call or whatsapp 9953592800


48

(c) India’s competition regulator CCI on Thursday passed an order _______ a provisional
penalty of Rs 1,337.7 crore on Google for ________ the dominant position of its Android
mobile phone operating system.

74. Which of the following pairs of words can most appropriately fit in the blanks in
the above statement respectively?

a. levying, abusing
b. exacting, mishandling
c. imposing, misemploying

A. Only a and b B. Only a and c C. Only b and c D. All a, b and c E. None of a, b


and c

(d) And its ______ comes from the scale of its impact in India and the global ______ too.

75. Which of the following pairs of words can most appropriately fit in the blanks in
the above statement respectively?

a. relevance, context
b. applicability, background
c. appositeness, backdrop

A. Only a and b B. Only a and c C. Only b and c D. All a, b and c E. None of a, b


and c

(e) CCI also asked Google to __________ a set of 10 remedial _________ that will open up
the mobile phone ecosystem.

76. Which of the following pairs of words can most appropriately fit in the blanks in
the above statement respectively?

a. carry through, measures

Use Code NIMISHA for maximum Discount

For any query call or whatsapp 9953592800


49

b. bring about, means


c. wind up, courses

A. Only a and b B. Only a and c C. Only b and c D. All a, b and c E. None of a, b


and c

77. Which of the following combinations is the combination of THREE CONSECUTIVE


statements after the correct rearrangement?

A. c b d
B. e b d
C. c e a
D. e d a
E. None of the above options

Directions (78-82): Rearrange the following EIGHT sentences (A), (B), (C), (D), (E), (F),
(G), and (H) in a proper sequence to form a meaningful paragraph. Then answer the
questions given below.

A) Historians consider this protest rally a pivotal moment in the freedom struggle in the state
of Travancore (now Kerala).
B) In 1938, inspired by Mahatma Gandhi, she gave up her job to join the Travancore State
Congress (state unit of Gandhi's Indian National Congress) and fight for India's
independence.
C) She is among many unsung heroes of India's freedom struggle whose names, over the
years, have sunk into anonymity outside of their home state.
D) Cherian was born in 1909 to a wealthy agrarian Catholic family in Travancore.
E) For more than 15 years she was probably the toughest woman in public politics in
Travancore
F) Accamma Cherian, a freedom fighter from the southern Indian state of Kerala, is best
known for the valour she displayed during a mass protest rally she led in 1938.
G) and was at the forefront of the momentous people's struggle in Travancore in the pre-
independence era," Kandathil Sebastian, a social scientist based in Delhi, wrote last year.
H) She studied history and went on to become a teacher, rising to the position of
headmistress at a school in Edakkara town.

Use Code NIMISHA for maximum Discount

For any query call or whatsapp 9953592800


50

78. Which of the following combinations is the combination of THREE CONSECUTIVE


statements after the correct rearrangement?

A. DEC
B. GDH
C. CFD
D. CBG
E. AGE

79. Which of the following statements is the CONCLUDING statement, after the correct
rearrangement?

A. D
B. G
C. C
D. B
E. A

80. Which of the following are pairs of the ANTEPENULTIMATE statement and the
PENULTIMATE statement, respectively, after the correct rearrangement?

A. EC
B. GD
C. DH
D. BG
E. AC

81. Which of the following should be the pair of the third and the fourth sentences of the
paragraph, respectively?

Use Code NIMISHA for maximum Discount

For any query call or whatsapp 9953592800


51

A. CE
B. GD
C. CF
D. BG
E. DH

82. Which of the following options is the pair of the terminal statement and the introductory
statement, respectively, after the correct rearrangement?

A. DE
B. GH
C. CF
D. BF
E. AD

ANSWER

1.(Answer - A. Only a and b

out-of-pocket(OoP) healthcare spending - Household out-of-pocket expenditure on health


comprise cost-sharing, self-medication and other expenditure paid directly by private
households, irrespective of whether the contact with the health care system was established
on referral or on the patient's own initiative.

Hence, it can be said that there has possibly been an increase in utilisation and
reduction in cost of services in government health facilities and that's why
households are spending less.

Also, the very consumption of healthcare and other social services has plummeted.
That's why out-of-pocket(OoP) healthcare spending has decreased.

c - Incorrect. It can't be said.)

2.(Answer - A. Only a and b


Use Code NIMISHA for maximum Discount

For any query call or whatsapp 9953592800


52

These two statements are the possible negative outcomes of the fact that the total
health spending — expenditure by government and non-government agents —
declined from 3.9% of the GDP to 3.2%.

Hence, these statements might create apprehensions.

c - Incorrect. It's something that should happen. But, it's not apprehensive.)

3.(Answer - A. Only a and b

Pandemic may have led to an increase in OoP expenditure.

That means, COVID-19 infection resulted in significant economic implications to


patients as well as a considerable financial burden to the general population for
preventive measures.
And, of course, due to the pandemic, the share of medical spending falling to patients
started being on an upward trajectory.

c - Incorrect. It's absolutely contradictory.


It indicates, Out-of-pocket healthcare spending likely decreased during the COVID-19
pandemic.

put off - to hold back to a later time.)

4.(Answer - D. All a, b and c

a - Correct.
Refer to - people who need to seek help for healthcare are unwilling to do so because of the
general economic distress

b - Correct.
It means - unwilling.

c - Correct.
It means - distress or suffering.)

Use Code NIMISHA for maximum Discount

For any query call or whatsapp 9953592800


53

5.(Answer - B. e d f

The correct arrangement is -

h b a e d f c g)

6.(Answer - B. Only a and c

worrying / concerning / perturbing - bothersome.

penury / indigence - poverty.


lucre - affluence.)

7.(Answer - A. Only a and b

OoP expenditure - Household OOP health payments are healthcare expenses charged
directly to the budget of the household that are not reimbursed at the point of care, by public
or private insurance or any third party.

Hence, the first two components are absolutely correct.

c - Incorrect. It's not directly linked to an individual or a household.


Hence, it can't contribute.)

8.(Answer - C. Only b and c

non-communicable diseases - diseases like heart disorders, cancer and diabetes.


Hence, b is correct.

c - Correct.
Refer to - Universal, accessible healthcare is a priority.

a - Incorrect.
It's not said anywhere in the above statement.)

9.(Answer - A. Only a and b

Use Code NIMISHA for maximum Discount

For any query call or whatsapp 9953592800


54

Clearly the author is being worried or upset and dissatisfied with India’s per capita
healthcare expenditure.

having kittens - being worried and upset.

malcontent - dissatisfied.

frenzied - wildly excited or uncontrolled.


No such expression has been expressed by the author.)

10.(Answer - B. e d f

The correct arrangement is -

h b a e d f c g)

11.(Answer - B. Only a and c

unto / in itself - considered alone.)

12.(Answer - D. All a, b and c

humanity / mankind / humankind - humans collectively.)

13.(Answer - A. Only a and b

shifting - variable.

changeless is the opposite.

discord / strife - lack of harmony.


dissension - disagreement.)

14.(Answer - A. Only a and b

demonstrate / exemplify - display.

resilience - toughness.

Use Code NIMISHA for maximum Discount

For any query call or whatsapp 9953592800


55

frailty - the condition of being weak and delicate.)

15.(Answer - D. All a, b and c

unleash / untether / unbridle - cause (a strong or violent force) to be released or become


unrestrained.

put together / track together - to accumulate.)

16.(Answer - A. Only a and b

recede - to withdraw.)

17.(Answer - C. c b

The correct arrangement is -

b d a f e c)

18.(Answer - A. Only a and b

precariousness - uncertainty.

credence - reliability.

volatility / changeability - volatility.)

19.(Answer - D. All a, b and c

break off - to end or conclude.)

20.(Answer - A. Only a and b

pull back - to withdraw.

set forth - to introduce.

Use Code NIMISHA for maximum Discount

For any query call or whatsapp 9953592800


56

All first three words of each option are synonymous.)

21. (Answer - A. Only a and b

contend with / grapple with - deal with.

'take on' is the correct phrase having the same meaning.

from overseas - from abroad.

The correct word is - oversea not overseas.)

22.(Answer - B. Only a and c

battered / thrashed / thumped - injured by repeated blows or punishment.

in line with - in accordance with or in alignment with.)

23.(Answer - D. All a, b and c

wind up / conclude - to come to an end.)

24.(Answer - B. a e

The correct arrangement is -

e f b d c a )

25.Ans. (d)
Sol. The correct sequence of the given sentences is beca. Statement (d) fails to fit into the
theme of the passage.
(b)Mahatma Gandhi’s death anniversary (January 30) may have just passed, but it is
not just an occasion to celebrate his life and his message once more, or to simply
add to the Gandhian biography, so to speak.

(e)We should think of Gandhi as a noble spirit who continues to be among us and
who contributes to the betterment of our world.

Use Code NIMISHA for maximum Discount

For any query call or whatsapp 9953592800


57

(c)Therefore, the task before us is how we, individually and collectively, can
understand and take forward the Gandhian nobility of spirit in today’s world.

(a)If there is only one idea that Gandhi should be remembered for and identified with,
it is the idea of empowerment of the other.

Hence, option (d) is the right answer choice.

26.Ans. (a)
Sol. The correct sequence of the given sentences is cbed. Statement (a) fails to fit into the
theme of the passage.
(c)The deadlock between protesting farmers and the government on the contentious
farm laws have taken an ugly turn, with incidents of vandalism during the tractor
parade by the protesting farmers on January 26.

(b)But it is yet unclear as to how the peaceful protest for the last two months was
allowed to take a violent turn by the farmers’ union leadership as well as the police
administration.

(e)The responsibility must be fixed for the administrative lapses and action taken
against erring antisocial elements, the events again point out to the growing mistrust
and breakdown of any dialogue between the two sides.

(d)Attempts by a section of media and representatives of political parties to use the


incidents of vandalism to vilify and malign the two-month-old peaceful agitation will
only add to the mistrust between the government and the farmer unions.

Hence, option (a) is the right answer choice.

27.Ans. (e)
Sol. The correct sequence of the given sentences is acbd. Statement (e) fails to fit into the
theme of the passage.
(a)The Economic Survey for 2020-21 is an expansive attempt at reviewing the
developments in the Indian economy during the current financial year and providing
an outlook for its near-term prospects.

(c)Spread over 700 pages, the survey highlights the policy achievements of the
government in steering the economy through the treacherous shoals of “the most
unfathomable global health emergency experienced in modern history”.

Use Code NIMISHA for maximum Discount

For any query call or whatsapp 9953592800


58

(b)It also cites an approach that used ‘graded public health measures to transform the
short-term trade-off between lives and livelihoods into a win-win that would save both
lives and livelihoods over the longer term’.

(d)The survey asserts that through the approach India established a globally unique
model of strategic policymaking in containing the COVID-19 pandemic while helping
the economy recover quickly from its deleterious impact.

Hence, option (e) is the right answer choice.

28.Ans. (c)
Sol. The correct sequence of the given sentences is eabd. Statement (c) fails to fit into the
theme of the passage.
(e)In October 2018, India agreed to purchase five Russian S-400 self-propelled
surface-to-air systems for approximately $5.4 billion during a state visit by President
Vladimir Putin.

(a)The S-400 defense system is one of the world’s most sophisticated, and the
agreement underscored Russia’s legacy as one of India’s major defense suppliers
over the past several decades.

(b)But American officials assert the deal violates U.S. law, specifically the
“Countering America’s Adversaries through Sanctions Act” (“CAATSA”).

(d)Enacted in 2017 with overwhelming bipartisan support, the CCATSA legislation


imposed sweeping sanctions against Moscow, while also effectively authorizing
secondary sanctions against any other country engaging in “significant transactions”
with Russia’s defense or intelligence sectors.

Hence, option (c) is the right answer choice.

29. Ans. (d)


Sol. The correct sequence of the given sentences is ceba. Statement (d) fails to fit into the
theme of the passage.
(c)Bezos started the company that would revolutionize e-commerce as an online
bookseller in his Seattle garage in 1994.

Use Code NIMISHA for maximum Discount

For any query call or whatsapp 9953592800


59

(e)Some 30 years later Amazon’s businesses span cloud storage, video streaming,
groceries and more.

(b)Bezos made headlines in 2019 with his high-profile divorce from MacKenzie Scott,
his wife of 25 years, in which he agreed to give her a quarter of his Amazon stake.

(a)In February 2020, he pledged $10 billion—almost 10% of his net worth at the time—
to fight climate change through an initiative called the Bezos Earth Fund.

Hence, option (d) is the right answer choice.

30. Ans. (e)


Sol. The correct sequence of the given sentences is abdc. Statement (e) fails to fit into the
theme of the passage.
(a)Whether natural or manmade, catastrophes disrupt everyday life.

(b)And in such an emergency, finding a place to live can be an urgent, difficult


decision.

(d)Electric power may go down, cell towers may not work, water could be
contaminated, and homes might be evacuated.

(c)As a result, routine chores, such as having medications delivered or trips to the
grocery store, are suddenly impossible.

Hence, option (e) is the right answer choice.

31. Ans. (b)


Sol. The correct sequence of the given sentences is deca. Statement (b) fails to fit into the
theme of the passage.
(d)Day is the local AARP chapter president in north Florida and recommends disaster
prevention through attending educational meetings with local emergency
management teams.

(e)She has plenty of advice of her own, too.

(c)Day counsels older people to realize it will take time for insurance claims to be
processed.

Use Code NIMISHA for maximum Discount

For any query call or whatsapp 9953592800


60

(a)And she warns against doing business with fraudulent contractors who often
swarm through damaged neighborhoods right after a hurricane.

Hence, option (b) is the right answer choice.

32. Ans. (c)


Sol. The correct sequence of the given sentences is dbea. Statement (c) fails to fit into the
theme of the passage.
(d)One cannot adopt a one-size-fits-all approach as the challenges in developing
countries are multi-layered compared to those in advanced countries.

(b)Mass-scale educational and economic deprivation with deep-seated ethnic,


religious, economic and social fault lines abound in a toxic ecosystem that can be
exploited easily.

(e)This is coupled with the lack of institutional autonomy of civil servants compared
to countries like the US.

(a)Poorly trained in digital forensics and, sometimes, compromised, law-enforcement


authorities have been complicit in the lack of action on a number of reported social
media threats, particularly against women.

Hence, option (c) is the right answer choice.


33.Ans. (d)
Sol. The correct sequence of the given sentences is bace. Statement (d) fails to fit into the
theme of the passage.
(b)After 1990, the military’s intervention had plunged democracy into darkness for a
while.

(a)There is a possibility that the script would unfold differently now.

(c)Myanmar occupies a critical geo-strategic niche, being the gateway to the restive
Indo-Pacific stretch that is witnessing the Great Game at play.

(e)It is, therefore, unlikely that the global powers would remain passive about these
developments.

Hence, option (e) is the right answer choice.

Use Code NIMISHA for maximum Discount

For any query call or whatsapp 9953592800


61

34.Ans. (a)
Sol. The correct sequence of the given sentences is ecdb. Statement (a) fails to fit into the
theme of the passage.
(e)In Calcutta, a group of children had their own lesson to impart at school.

(c)On the eve of India’s Republic Day, they got together to celebrate the Constitution,
the document the nation bequeathed itself 71 years ago.

(d)As the world lurches towards self-defeating extreme intolerance from one day to
the next, these boys and girls have planted their own flag.

(b)One that flutters in the sky to remind us elders of our miserable failings, telling us
that the choice between right and wrong is easy if we want it to be.

Hence, option (a) is the right answer choice.

35.Ans. (b)
Sol. The correct sequence of the given sentences is edca. Statement (b) fails to fit into the
theme of the passage.
(e)With a clear focus on expansion of Metro Rail and bus services through Central
funding, Budget 2021 has recognised a core component of urbanisation.

(d)Comfortable, safe and affordable commuting has well-recognised multiplier effects


for the economy and more generally for public health.

(c)Although COVID-19 has had the perverse effect of driving people away to the
safety of personal car and two-wheeler bubbles.

(a)There is little doubt that when the pandemic is under control, more people will
return to clean and green mass mobility.

Hence, option (b) is the right answer choice.

36.Ans. (b)
Sol. The correct sequence of the given sentences is caed. Statement (b) fails to fit into the
theme of the passage.
(c)A large segment of this population is underprivileged.

Use Code NIMISHA for maximum Discount

For any query call or whatsapp 9953592800


62

(a)A survey found that in 2020 stranded migrants had to pay several times more than
normal fares to go back home — may discourage them from exercising their
franchise.

(e)The paucity of a clear legal definition of ‘migrant worker’ also gets in the way of
identifying the segment for voting facilities.

(d)To this end, data from the Inter-State Migrant Workmen Act, 1979 requires all
establishments hiring inter-state migrants to be registered and used to create a
comprehensive database as well as a legal identity.

Hence, option (b) is the right answer choice.

37.Ans. (d)
Sol. The correct sequence of the given sentences is abce. Statement (d) fails to fit into the
theme of the passage.
(a)Treating fever can prolong or worsen illness.

(b)Thus, when you take medication such as acetaminophen or ibuprofen to suppress


a fever, you actually work against the inherent protective benefits nature bestowed.

(c)Yes, a fever reducer would probably make you feel better, relieving symptoms such
as headache, muscle aches and fatigue.

(e)But you’re not supposed to feel better and you’re supposed to stay under the
covers, keep warm and ride out the infection”, not go out and spread it to others.

Hence, option (d) is the right answer choice.

38.Ans. (d)
Sol. The correct sequence of the given sentences is DEBAC.
Hence, option (d) is the right answer choice.

39.Ans. (c)
Sol. The correct sequence of the given sentences is DEBAC.
Hence, option (c) is the right answer choice.

40. Ans. (a)


Sol. The correct sequence of the given sentences is DEBAC.

Use Code NIMISHA for maximum Discount

For any query call or whatsapp 9953592800


63

Hence, option (a) is the right answer choice.

41.Ans. (b)
Sol. The correct sequence of the given sentences is DEBAC.
Hence, option (b) is the right answer choice.

42.Ans. (d)
Sol. The correct sequence of the given sentences is DEBAC.
Hence, option (d) is the right answer choice.

43. Ans. (e)


Sol. The correct sequence of the given sentences is EADBC.
Hence, option (e) is the right answer choice.

44.Ans. (e)
Sol. The correct sequence of the given sentences is EADBC.
Hence, option (e) is the right answer choice.

45.Ans. (b)
Sol. The correct sequence of the given sentences is EADBC.
Hence, option (b) is the right answer choice.

46.Ans. (d)
Sol. The correct sequence of the given sentences is EADBC.
Hence, option (d) is the right answer choice.

47. Ans. (b)


Sol. The correct sequence of the given sentences is EADBC.
Hence, option (b) is the right answer choice.

48.Ans. (e)
Sol. The correct sequence of the given sentence is CEABD.
Hence, option (e) is the right answer choice.

49. Ans. (c)


Sol. The correct sequence of the given sentence is CEABD.

Use Code NIMISHA for maximum Discount

For any query call or whatsapp 9953592800


64

Hence, option (c) is the right answer choice.

50.Ans. (c)
Sol. The correct sequence of the given sentence is CEABD.
Hence, option (c) is the right answer choice.

51. Ans. (b)


Sol. The correct sequence of the given sentence is CEABD.
Hence, option (b) is the right answer choice.

52. Ans. (a)


Sol. The correct sequence of the given sentence is CEABD.
Hence, option (a) is the right answer choice.

53.Ans. (e)
Sol. The correct sequence of the given sentence is DEABC.
Hence, option (e) is the right answer choice.

54.Ans. (c)
Sol. The correct sequence of the given sentence is DEABC.
Hence, option (c) is the right answer choice.

55.Ans. (b)
Sol. The correct sequence of the given sentence is DEABC.
Hence, option (b) is the right answer choice.

56.Ans. (b)
Sol. The correct sequence of the given sentence is DEABC.
Hence, option (b) is the right answer choice.

57. Ans. (a)


Sol. The correct sequence of the given sentence is DEABC.
Hence, option (a) is the right answer choice.

58.(Answer - D. All a, b and c

Use Code NIMISHA for maximum Discount

For any query call or whatsapp 9953592800


65

pooling resources - combining more than one person's supply of something.

Refer to - In fact, Russia-ally Belarus today ordered joint deployment of troops with
Russian forces near Ukraine.

give the command to - to order.

lay down - to order; formulate and enforce or insist on an order or a principle.

Refer to - In fact, Russia-ally Belarus today ordered joint deployment of troops with Russian
forces near Ukraine.)

59.(Answer - A. Only a and b

a - Correct.
Refer to - Although Ukraine hasn’t officially claimed responsibility for the bombing of the
bridge, Russia’s response marks a further escalation in the war.

b - Correct.
Refer to - Although Ukraine hasn’t officially claimed responsibility for the bombing of the
bridge, Russia’s response marks a further escalation in the war.

c - Incorrect.
It means - deliberately ignore wrongdoing by others.
Hence, it's contradictory.)

60. (Answer - A. Only a and b

begin hostilities - to attack.


Refer to - In what were clearly revenge strikes, Russian forces targeted multiple Ukrainian
cities today with a barrage of cruise missiles during peak working hours.

clear-cut - specific and well-defined.


Russian forces had fixed targets and fixed timing.

Refer to - In what were clearly revenge strikes, Russian forces targeted multiple Ukrainian
cities today with a barrage of cruise missiles during peak working hours.

Use Code NIMISHA for maximum Discount

For any query call or whatsapp 9953592800


66

c - Incorrect.
wheeling and dealing - engaging in commercial or political scheming, especially
unscrupulously.
No such thing is mentioned.)

61.(Answer - D. All a, b and c

in the offing - likely to happen.


Refer to - If Belarus forces actively join the war against Ukraine, it will certainly take the
conflict to a whole new level.

sea change - a notable transformation.


Refer to - If Belarus forces actively join the war against Ukraine, it will certainly take the
conflict to a whole new level.

take in hand - become influential in determining something.


This is indicated towards Belarus and its influence.)

62.(Answer - A. Only a and b

pulling down - destroying.

a - Correct.
Refer to - This comes after the destruction of a section of the Russia-made Crimea bridge
over the weekend that Vladimir Putin described as an act of terrorism.

c - Incorrect.
It means peace.
It's absolutely contradictory.)

63.(Answer - C. d c

The correct arrangement is -

c e b a d)

64.(Answer - A. p o n m

Use Code NIMISHA for maximum Discount

For any query call or whatsapp 9953592800


67

maturation - growth.
consideration - concern.)

65.(Answer - C. p n m o

callosity - callousness or insensibility.


vexation - worry or concern.
veneer - something that hides something unpleasant or unwanted.
dispensation - present government.)

66.(Answer - D. o p n m

valuation - cost.
swelling - increase.
stratagem - scheme.
predicament - difficulty.)

67.(Answer - A. o m p n

sustenance - nutrition.
partisanship - discrimination.
delinquency - corruption.)

68.(Answer - C. p o n m

preceptor - teacher.
repast - meal.

69.(Answer - E. No replacement required

nosh - meal.
outlay - price or expense.
wherewithal - allocated money or budget.)

70. (Answer - A. o m p n

contemplation - thought or attention.

Use Code NIMISHA for maximum Discount

For any query call or whatsapp 9953592800


68

credo - principle.
enlightenment - education.)

71.(Answer - B. e f a d
The correct arrangement is -

b c e f a d g

72.(Answer - B. Only a and c

dividend / portion / stake - share.


widespread / prevailing - popular.

spurned - rejected.)

73.(Answer - B. Only a

monopolistic - relating to a person or business that has exclusive possession or control of


the supply of or trade in a commodity or service.

complaisant - accommodative or helpful.

milestone / milepost - a significant stage or event in the development of something.)

74.(Answer - D. All a, b and c

levy / exact - to impose (a tax, fee, or fine).

abuse - misuse or mishandle or misemploy.)

75.(Answer - D. All a, b and c

appositeness / applicability / relevance - the quality or state of being closely connected or


appropriate.

context / background / backdrop - the circumstances that form the setting for an event,
statement, or idea, and in terms of which it can be fully understood.)

Use Code NIMISHA for maximum Discount

For any query call or whatsapp 9953592800


69

76.(Answer - A. Only a and b

carry through / bring about - to execute.


wind up - to discontinue.

measure / course / means - a plan or course of action taken to achieve a particular


purpose.)

77.(Answer - B. e b d

The correct arrangement is -

c e b d a

78.B

Solution : The correct sequence: FACEGDHB

79.D

Solution : The correct sequence: FACEGDHB

80.C

Solution : The correct sequence: FACEGDHB

81.A

Solution : The correct sequence: FACEGDHB

82.D

Solution : The correct sequence: FACEGDHB

Use Code NIMISHA for maximum Discount

For any query call or whatsapp 9953592800


70

CHAPTER 2 - FILLERS

Direction(1-3): In each question a sentence is given followed by a blank. You are


provided with three fragments a, b and c. You have to identify which
statement/statements can carry forward the given sentence in the most logical way so
as to make the sentence coherent and contextually correct.

1. Freedom of expression and the freedom to practise and propagate a person’s own
religion are constitutionally endowed rights in this multi-religious country.
__________________________________

a. Not believing in the deity’s existence is as much an opinion as is believing in it.


b. The resolution will take its course according to the wisdom of the courts.
c. In an ideal arrangement, neither the non-believers of God nor the believers of God
should threaten each other.

A. Only a and b B. Only a and c C. Only b and c D. All a, b and c E. None of a, b


and c

2. Rice, wheat and sugar are agri-commodities in which India’s production, for much of the
last decade and more, has been surpassing its consumption requirement.
___________________________

a. This has resulted in overflowing godowns.


b. As a result, the country is also turning into a significant exporter.
c. These surpluses are like a past story.

A. Only a and b B. Only a and c C. Only b and c D. All a, b and c E. None of a, b


and c

Use Code NIMISHA for maximum Discount

For any query call or whatsapp 9953592800


71

3. In the “far from normal” last two and half years, when humanity came the closest to living
in a science fiction world, India and Indians under the untiring leadership of Prime Minister
Narendra Modi can collectively claim to have learnt crucial lessons. We demonstrated
resilience, determination and strength that has earned respect the world over.
_______________________

a. This is an accomplishment unto itself.


b. We can claim to have put together lessons of decades into two years to succeed on
many fronts.
c. That’s no small achievement, particularly amid rapidly shifting realities and constantly
evolving science, and given social discord and vaccine hesitancy in so many different
countries.

A. Only a and b B. Only a and c C. Only b and c D. All a, b and c E. None of a, b


and c

Direction(4-7): In each question a sentence is given followed by a blank. You are


provided with three fragments I, II and III. You have to identify which
statement/statements can carry forward the given sentence in the most logical way so
as to make the sentence coherent and contextually correct.

4. Early-to-mid 20s and 30s is the time to make use of an education, skill up and gain
experience in a chosen field. Whether it be law, medicine or an unconventional career in
film, food or music, it takes patience and painstaking effort to become (confidently) excellent
at something. _____________________________

a. Practice isn’t the thing you do once you’re good, it’s the thing you do that makes you
good.
b. It stands to reason that unfettered youth is the ideal time to build muscle memory
because you are free from other responsibilities.
c. If finding yourself is a priority at 30, there’ll be alarmingly little to do at 60.

A. Only a and b B. Only a and c C. Only b and c D. All a, b and c E. None of a, b


and c

Use Code NIMISHA for maximum Discount

For any query call or whatsapp 9953592800


72

5. Queen Elizabeth Windsor, the longest-ruling monarch of Britain and possibly anywhere in
the world, died on September 8. Elizabeth was 96 years old. She took over the reins of the
British Crown on February 6, 1952.

a. Her path towards the Crown was the result of extraordinary circumstances.
b. Her becoming the emperor was a long shot since her father, the Duke of York, was
King George V’s second son.
c. With no male siblings, Elizabeth became the leader of the British Empire, ruling the
earth at the age of 25.

A. Only a and b B. Only a and c C. Only b and c D. All a, b and c E. None of a, b


and c

6. The last semester of the second year of my medical degree ends this December. Post
that, we are disallowed from attending classes online since the third year of an MBBS
degree concentrates on practical skills. Even now, the nature of my degree is such that
online classes do not suffice. So far, there has been no official statement with regard to our
academic future in the country by either the central government or the National Medical
Commission. _________________________

a. This prolonged uncertainty compounded by the experiences this February has left me
distraught.
b. I put this faith in my country’s government to secure my future, so rudely snatched
away due to a humanitarian crisis.
c. This distress experiencing academic insecurity has impacted my education in more
ways than I can count.

A. Only a and b B. Only a and c C. Only b and c D. All a, b and c E. None of a, b


and c

Use Code NIMISHA for maximum Discount

For any query call or whatsapp 9953592800


73

Directions(7-10): Four small paragraphs are given below, labelled A, B, C and D,


among these, three paragraphs are logically connected. From the given options,
choose the option that does not fit into the theme of the passage.
You're now given three paragraphs. You've to replace the odd paragraph with a
paragraph / paragraphs from the three paragraphs given, that is/are in line with the
other three paragraphs.

7.
A. The songs that beamed into living rooms during the depths of the COVID-19 pandemic
may have featured an artist’s hits. But there’s just something magical about seeing music
surrounded by other people. Some fans reported being so moved by their first live shows in
nearly two years that they wept with joy.

B. Having performed live on numerous occasions, Arijit Singh says for him, singing live and
in a studio are two completely different experiences. “For me, performing live is like inviting
people over for a house party where I can sing my heart out while people around me are
joining in. When I am on stage I make sure to interact with audiences.

C. We live in the age of auto-tune and studio-savvy producers. We have learned the hard
way that you don’t have to be particularly talented to become a musical sensation. While this
isn’t true for everyone, some artists don’t sound as good live because they don’t have a
studio polishing their sound before it hits the audience.

D. With a live performance, the concert is a conversation, not a monologue. The artists can
talk to the audience to get a feel for what they want and how they’re feeling. This
personalizes the performance and makes the audience feel like they are a part of the
performance, not just a bystander.

A. A B. B C. C D. D E. No statement is ODD

8. Which of the following paragraphs can replace the odd paragraph in the above
context?

a. In a live concert, the artist sets the atmosphere. His or her enthusiasm may make the
energy in the room quadruple. This, in turn, increases the excitement of the audience
and all of a sudden, a show that could have been a drag is a huge crowd-pleaser.

Use Code NIMISHA for maximum Discount

For any query call or whatsapp 9953592800


74

b. A live show is more than the artist performing on the stage. The performances need
sound checks, warm-ups and lots of equipment. It takes time to get every intricacy of
a performance together correctly.
c. In Live shows, performances normally do not have seating. For a three-hour show, it
can be uncomfortable to stand in one place for the whole time, especially squished as
sardines in a show crowd.

A. Only a and b B. Only a C. Only b and c D. Only b E. None of a, b and c

9.
A. The public spat between Kerala Governor Arif Mohammad Khan and Chief Minister
Pinarayi Vijayan on Monday marks a new low for their already frosty relationship. In an
unprecedented step, Khan addressed the media and leveled serious charges against
Vijayan, his office staff, the ruling CPM and its leaders, and the state police.

B. The Disaster Management Act, 2005 causes discontent in Kerala as the Central
guidelines are not in accordance with the state government, even though public health is a
state matter on which the Parliament cannot legislate.

C. Inter-state border disputes can "only" be resolved with the cooperation of state
governments involved in the matter and the Centre just acts as a facilitator in such cases,
the Union government informed Parliament on Tuesday.

D. The West Bengal Chief Minister, Mamata Banerjee, has had a tumultuous relationship
with Governor Jagdeep Dhankhar for a long time, and has been demanding his removal
from the constitutional post.

A. A B. B C. C D. D E. No statement is ODD

10. Which of the following paragraphs can replace the odd paragraph in the above
context?

a. The Supreme Court on Tuesday presided over two separate river disputes, asking
the union to intervene to find amicable solutions to neighboring states’ water conflicts.
b. Kerala became the first state to challenge the Citizenship (Amendment) Act (CAA)
before the Supreme Court. The Kerala government has moved the apex court under

Use Code NIMISHA for maximum Discount

For any query call or whatsapp 9953592800


75

Article 131 of the Constitution, the provision under which the Supreme Court has
original jurisdiction to deal with any dispute between the Centre and a state.
c. The Assam government has been putting all efforts to resolve the border disputes
with the neighboring states. Several rounds of talks have been held with Arunachal
Pradesh, Meghalaya and Mizoram to solve the issue amicably.

A. Only a and b B. Only a C. Only b and c D. Only b E. None of a, b and c

Word Fillers

Directions(11-20): In each of the questions below, a sentence is given with one blank
in it that indicates a missing word. Choose the correct word/words that will make the
sentence grammatically and contextually complete and correct.

11. The confrontation in Kerala comes in the backdrop of a string of Governor-government


________ in Opposition-ruled states — among them West Bengal, Tamil Nadu and
Maharashtra during the MVA government.

a. spats
b. tiffs
c. covenants

A. Only a and b B. Only a and c C. Only b and c D. All a, b and c E. None of a, b


and c

12. Three decades after militancy had ________ its cinema halls to shut down, the big
screen has returned to Kashmir.

a. forced
b. coerced
c. constrained

A. Only a and b B. Only a and c C. Only b and c D. All a, b and c E. None of a, b


and c

Use Code NIMISHA for maximum Discount

For any query call or whatsapp 9953592800


76

13. Long-distance diasporic nationalisms have always been a __________ of global politics.

a. feature
b. facet
c. slant

A. Only a and b B. Only a and c C. Only b and c D. All a, b and c E. None of a, b


and c

14. The decisive change came in the _______ of two developments.

a. wake
b. account
c. aftermath

A. Only a and b B. Only a C. Only b D. All a, b and c E. None of a, b and c

15. The Indian state’s statement condemned “the violence perpetrated against the Indian
community in Leicester and the _____________ of premises and symbols of Hindu religion”.

a. vandalization
b. fabrication
c. defacement

A. Only a and b B. Only a and c C. Only b and c D. All a, b and c E. None of a, b


and c

16. It is bizarre to think you can have this much dissemination of hate without it having
violent political ______________.

a. ramifications
b. upshots

Use Code NIMISHA for maximum Discount

For any query call or whatsapp 9953592800


77

c. repercussions

A. Only a and b B. Only a and c C. Only b and c D. All a, b and c E. None of a, b


and c

17. So much has been said and written about some of the disastrous judgments that have
recently __________ from the Supreme Court.

a. subsumed
b. emanated
c. exuded

A. Only a and b B. Only a and c C. Only b and c D. All a, b and c E. None of a, b


and c

18. That is why both number and composition matter greatly in cases of constitutional and
other __________.

a. significance
b. prominence
c. import

A. Only a and b B. Only a and c C. Only b and c D. All a, b and c E. None of a, b


and c

19. A fine of Rs 5 lakh was _________ on the petitioner who ran a tribal welfare NGO being
“exemplary costs” as the Court decided that “no case worth the name” was made out.

a. imposed
b. enacted
c. exacted

A. Only a and b B. Only a and c C. Only b and c D. All a, b and c E. None of a, b


and c

Use Code NIMISHA for maximum Discount

For any query call or whatsapp 9953592800


78

20. It’s high time attention was focused on the role of the Chief Justice of India as Master of
the Roster – _________ which cases should be heard or not heard.

a. dithering
b. determining
c. picking out

A. Only a and b B. Only a and c C. Only b and c D. All a, b and c E. None of a, b


and c

Level - Moderately Hard

Direction(21-28): Below are some statements, given in a random order. When these
statements are sequenced properly, they'll form a coherent and meaningful
paragraph.
There are various questions based on each of the statements. Read every statement
carefully and answer the questions that follow.

(a) The Supreme Court has reportedly been examining death penalty jurisprudence with the
aim of clarification, especially in its procedural aspects.

(b) Capital punishment, therefore, demands that a finely balanced process precedes it.

(c) The sentence cannot be quashed just because it was delivered on the same day; the
Supreme Court reportedly said that aggravating circumstances are part of the prosecution’s
arguments, but mitigating circumstances cannot be presented after sentencing, and that
takes time.

(d) One of these aspects is the presentation of mitigating circumstances.

(e) This puts the convict at a disadvantage, _________ his chances of __________.

(f) When the law takes the extreme step, it must do so with the greatest care.

Use Code NIMISHA for maximum Discount

For any query call or whatsapp 9953592800


79

(g) Sometimes trial courts deliver the guilty verdict and the sentence of death on the same
day, so the convict does not get enough time to present the mitigating circumstances that
could prevent death.

(a) The Supreme Court has reportedly been examining death penalty jurisprudence with the
aim of clarification, especially in its procedural aspects.

21. Which of the following aspects the Supreme Court could be examining?

a. Only on special grounds, the death penalty can be sentenced and should be treated
as exceptional punishment.
b. In accordance with Articles 21 and 22 of the Constitution, the accused has the right to
a prompt and fair trial.
c. Under Articles 21 and 19 of the Constitution, the accused has freedom of speech and
expression under custody.

A. Only a and b B. Only a and c C. Only b and c D. Only a E. All a, b and c

(b) Capital punishment, therefore, demands that a finely balanced process precedes it.

22. Which of the following statements can be made from the above statement that will
justify what has been said in the above statement?

a. Anyone sentenced to death shall have the right to appeal to a court of higher
jurisdiction; also, anyone sentenced to death shall have the right to seek pardon, or
commutation of sentence.
b. Safeguards guaranteeing protection of the rights of those facing the death penalty
must be there.
c. The death sentence is a violation of the right to life, which is the most fundamental of
all human rights, hence it must be abolished.

A. Only a and b B. Only a and c C. Only b and c D. All a, b and c E. None of a, b


and c

(c) The sentence cannot be quashed just because it was delivered on the same day; the
Supreme Court reportedly said that aggravating circumstances are part of the prosecution’s

Use Code NIMISHA for maximum Discount

For any query call or whatsapp 9953592800


80

arguments, but mitigating circumstances cannot be presented after sentencing, and that
takes time.

23. What does the author suggest?

a. Factors that lessen the severity or culpability of a criminal act must be presented
before declaring the punishment.
b. Factors that make a felony more severe or worse, must be prevented to be
presented.
c. Factors that might mitigate a defendant’s punishment take time to be established.

A. Only a and b B. Only a and c C. Only b and c D. All a, b and c E. None of a, b


and c

(d) One of these aspects is the presentation of mitigating circumstances.

24. Which of the following can be the mitigating circumstances?

a. Lack of a prior criminal record


b. Circumstances at the time of the offense, such as provocation, stress, or emotional
problems that might not excuse the crime but might offer an explanation
c. Mental or physical illness of the offender

A. Only a and b B. Only a and c C. Only b and c D. All a, b and c E. None of a, b


and c

(e) This puts the convict at a disadvantage, _________ his chances of __________.

25. Which of the following pairs of words can most appropriately fit in the blanks in
the above statement respectively?

a. eliminating, life incarceration


b. knocking out, life sentence
c. ruling out, judicial execution

A. Only a and b B. Only a and c C. Only b and c D. All a, b and c E. None of a, b


and c

Use Code NIMISHA for maximum Discount

For any query call or whatsapp 9953592800


81

26. Which of the following pairs is the pair of the CONCLUDING statement and the
ANTEPENULTIMATE statement respectively, after the correct rearrangement?

A. e a
B. c a
C. g e
D. e g
E. None of the above options

Phrase Fillers

Direction(27-36): In each question a sentence is given followed by a blank. You are


provided with three fragments a, b and c. You have to identify which PHRASE(s) can
carry forward the given sentence in the most logical way so as to make the sentence
coherent and contextually correct.

27. This monsoon, several parts of India have found themselves literally swimming in filth
and sewage. Gutters have choked with garbage, plastics being the primary culprit. Single-
use plastic carry-bags have been singled out for particular attention and regular meetings
have been held to educate – as well as admonish — people about their use. Thus once
more, the buck has stopped at the doorstep of the people – __________________________

a. their good behaviour is seen as the key to change.


b. who is going to clean that up?
c. people have again started ignoring preventive measures.

A. Only a and b B. Only a C. Only a and c D. Only b E. All a, b and c

28. Let us come to garbage and sustainability — the expenditure on packaging, carriage and
freight. The freight expenditure tells us that out of every Rs 100 of the company’s product
that we buy, Rs 5 has gone up in vehicular smoke and Rs 8 for the package that we hold in

Use Code NIMISHA for maximum Discount

For any query call or whatsapp 9953592800


82

our hand. This packaging is required because the products must have a long shelf life,
__________________________________

a. and withstand heat and sunlight.


b. and hold out against rough handling during freight.
c. and give in to the outside deterrents.

A. Only a and b B. Only a and c C. Only b and c D. All a, b and c E. None of a, b


and c

29. The Supreme Court’s decision to live-stream proceedings was taken in a full court led by
Chief Justice of India UU Lalit last week. It is immensely welcome. The move, which
expands on the idea of an open court that is accessible and transparent, marks the onset of
a significant transformation in the judiciary’s functioning. Live-streaming directly brings
citizens into conversations that have so far largely remained restricted to judges, lawyers
and litigants, on vital issues affecting the polity and society and these include
___________________________________

a. cases challenging the constitutionality of job quotas for economically weaker


sections.
b. questions related to the political crisis in various states.
c. discussions related to wealth inequalities in the country.

A. Only a and b B. Only a and c C. Only b and c D. All a, b and c E. None of a, b


and c

30. Live-streaming is also an accountability tool for a check on populist statements by


lawyers and for ensuring that judges begin proceedings on time and give equal opportunity
to all sides. It is also an invaluable resource for those who study and teach law. Also,
_________________________________

a. the archive will be an excellent opportunity to identify and address systemic patterns
that are affecting judicial functioning.
b. it will take years to ensure the adequate infrastructure for live-streaming.
c. live-streaming will be the first step towards greater transparency that will enhance
public trust in the judiciary’s functioning.

Use Code NIMISHA for maximum Discount

For any query call or whatsapp 9953592800


83

A. Only a and b B. Only a and c C. Only b and c D. All a, b and c E. None of a, b


and c

31. After hovering around the psychological mark of 80 against the dollar till now, the rupee
has breached this level — on Tuesday, the currency ended the day at 81.58 against the
dollar. But considering the extent of tightening the Fed envisages over the coming months,
the pressure on the rupee is unlikely to subside. The inflationary implications of a weaker
currency are likely to weigh on the members of the monetary policy committee when they
meet over the next few days, complicating matters. But even as the RBI is unlikely to stop
intervening in currency markets, considering the scale and speed of the drawdown of its
reserves, the central bank should ____________________________

a. be circumspect about how it proceeds as the global economic environment becomes


increasingly challenging.
b. allow the rupee to adjust to market realities.
c. ignore the effects across asset classes, and markets.

A. Only a and b B. Only a and c C. Only b and c D. All a, b and c E. None of a, b


and c

32. In 2047, independent India will be a hundred years old. What will our country look like
then? Will it be the paradise we have dreamt of and wanted to make it? Will poverty, caste,
gender inequality, religious discrimination be a thing of the past? These questions are much
in the air today. For those who hold power, questions like these often
____________________________________

a. transform into promises.


b. become ways to project futures and keep hope alive.
c. draw attention away from the despair of the present

A. Only a and b B. Only a and c C. Only b and c D. All a, b and c E. None of a, b


and c

Use Code NIMISHA for maximum Discount

For any query call or whatsapp 9953592800


84

33. Look ahead of you, we’re told, the golden future looms. We may not have it so good
now, but our children and grandchildren will, and it is their future that we must work towards.
But while leaders and politicians hold out these promises, for most people the needs are
more urgent - __________________________________

a. there must be food to eat now.


b. there must be schools for the children to go to now.
c. there must be safety on the streets now.

A. Only a and b B. Only a and c C. Only b and c D. All a, b and c E. None of a, b


and c

34. Seventy-five years ago, when India stood at the threshold of independence, the future
promised a great deal: _________________________________. Today we know well that
those dreams have been tempered with the harshness of reality. But at the time, our
dreamers, our parents and grandparents, had to believe in those dreams, they had to have
those hopes.

a. equality was held out as a promise


b. land reform was an achievable horizon
c. the future held no discrimination on the basis of sex, religion, caste

A. Only a and b B. Only a and c C. Only b and c D. All a, b and c E. None of a, b


and c

35. If the past 75 years have taught us anything, it is that looking back is sometimes more
productive than speculating about the future - hindsight
______________________________.

a. unnerves us about challenges


b. offers us lessons
c. helps to think of the limits of the possible

Use Code NIMISHA for maximum Discount

For any query call or whatsapp 9953592800


85

A. Only a and b B. Only a and c C. Only b and c D. All a, b and c E. None of a, b


and c

36. We prided ourselves on affirmative action and saw it as the vehicle which would lead us
towards the annihilation of caste, as B R Ambedkar had articulated it. But today, while caste
articulations are fierce and strong across the country, ____________________________. If
three quarters of a century was not sufficient to change this dark reality, can we believe that
the next quarter will?

a. caste laws are absolutely hard and fast and uncompromising


b. caste violence is open, blatant and widespread
c. caste discrimination is a make-believe area of discussion

A. Only a and b B. Only b C. Only a D. Only a and c E. All a, b and c

Topic - Para Fillers

Direction(37-51): In each of the following questions a short passage is given with one
of the lines in the passage missing and represented by a blank. Select the best out of
the four answer choices given, to make the passage complete and coherent. If no
option fits in the blank, choose (E) as your answer.

37. A silver lining recently is the decline in suicides of farmers. The report on accidental
deaths and suicides of 2019 published, indicates this: from 11,379 deaths in the farm sector
recorded for 2016 they have dropped to 10,281 in 2019. ______________________ The
reason for this, according to the Union minister of state, is that several states and Union
territories gave no figures for farmer suicides. Hence listing causes became ‘untenable’.

A. The problem with the 2019 report is that it no longer enlists causes, such as crop
failure or debt, for the suicides.
B. In that context, the 2019 report, even if partial, does indicate a positive change.
C. The 2019 figure is marginally lower to 2018 when 10,348 people took their lives.
D. The report defines farmers / cultivators as people farming either on their own land or
on leased land, with or without the assistance of agricultural labourers.
E. None of the above options.

Use Code NIMISHA for maximum Discount

For any query call or whatsapp 9953592800


86

38. The Centre’s plaintive cries about the absence of data on the deaths of frontline health
workers have several implications. _______________________ The prime minister goaded
the nation to honour Covid warriors by raising a frightful din with pots and pans; more
recently, a bill has been passed by Parliament to protect frontline workers and doctors from
physical assault. But several other crippling dimensions of the crisis remain unaddressed.

A. People across India displayed an example of unity by honouring Covid warriors.


B. The most troubling is its use of lofty rhetoric in place of meaningful interventions.
C. More or less like the various stages of the journey of coronavirus, the list of Covid
warriors also expanded.
D. Moreover, the world needs to work towards advancement in medical research and
technology.
E. None of the above options.

39. In these days of more robust equality, however, fun is less picky about gendered
delicateness than before. ______________________________ With male chefs taking over
the more fashionable kitchens and women pilots holding the controls in cockpits have
become rather elusive attributes to define. More so in middle-class households, where men
do the dishes and look after the baby as much as women come home late from work.

A. It is almost impossible for women to have self-acquired property and gender equality
is still a far cry.
B. Injustice arises out of society’s inability to accept the fact that men and women should
be equal.
C. Men and women can slurp and gobble away without attracting the raised eyebrow.
D. There is now a stack of evidence of the fact that men benefit from living in more
gender-equal societies.
E. None of the above options.

40. Vaccines have turned despair to hope but the virus, or another pandemic, cannot be
wished away. ________________________________ These vanishing human traits could
prove to be the vaccine that is needed to vanquish the germs of hate and discrimination that
plague the global body politic.

A. The uninterrupted supply chain of life-saving anti-viral medicines and medical items
are essential in the human life saving category.

Use Code NIMISHA for maximum Discount

For any query call or whatsapp 9953592800


87

B. This makes it important for the world to wish for the resurrection of empathy and
kindness.
C. The world is existing because there are some kind, helpful and good-hearted people
still around and the world rests on their shoulders.
D. Till now, there are numbers of proposed therapeutics agents existed for this
conditions, seems to have efficacy against the disease.
E. None of the above options.

41. The goals of the Centre’s flagship Beti Bachao, Beti Padhao scheme are noble, but last
year the Narendra Modi-led government conceded in Parliament that almost 56 per cent of
the funds for the scheme had been spent on advertising, with no impact assessment being
conducted. ___________________________ Affirmative action must be accompanied by
political will to raise public awareness as well as bolster women’s agency over their own
bodies.

A. This is a flagrant abuse of power intended to create fear in a particular community.


B. A number of needy citizens, particularly from the underprivileged sections, are yet to
be enrolled as beneficiaries.
C. Behind this mass enthusiasm for the virtues of ‘good governance’ is none other than
the prime minister himself.
D. Filling the legislative, medical and societal gaps is only part of the challenge.
E. None of the above options.

42. Researchers from the Centre for Science and Environment(CSE) have alleged that the
honey produced by major Indian brands is adulterated with sugar syrup. That is not all;
_______________________ As a result, the environment watchdog has selected 13
popular honey brands to assess the purity of the product.

A. there is a case for raising public awareness so that citizens know that the products
they are consuming are appetizing.
B. there is also the additional imperative of assisting beekeeping communities, whose
livelihoods have been decimated by the pandemic.
C. there appears to be an incongruity in the results of different tests geared towards the
same purpose.
D. the claim was considered false and misleading by implication and omission and
upheld under Chapter 1.4 of the ASCI Code.
E. None of the above options.

Use Code NIMISHA for maximum Discount

For any query call or whatsapp 9953592800


88

43. India has not been reticent about making public a number of data estimates, including
those pertaining to unemployment and earnings. ________________________ The
estimated wage loss in the informal sector was to the tune of Rs 63,550 crore in the first half
of 2020.

A. According to the report, from the data available on India, the nation’s economy may
have undergone the sharpest diminution in average wages.
B. India’s economic growth is expected to improve to 6.3% with the country leading
economic recovery in South Asia, according to the report.
C. Nearly 25 million jobs could be lost worldwide due to the pandemic, but an
internationally coordinated policy response can help lower the impact on global
unemployment, according to the report.
D. India reluctantly revealed that its unemployment rate in October rose to 8.5%, the
highest level since August 2016.
E. None of the above options.

44. The deep crisis in India’s agrarian sector goes beyond the features of the new farm bills.
It is the result of systematic crippling of farmers’ interests and agricultural productivity by
successive governments. ________________________ But what the ruling party seems to
be trying is to demonize protests and dissent.

A. Every trick in the authoritarian rule book had been employed by the ruling party to
prevent the farmers from getting themselves heard.
B. The prime minister, instead of agreeing to a meeting immediately, kept insisting that
the farmers were being misled.
C. The ruling party is not the only party complicit in this institutional apathy.
D. Unfortunately, one voice, that of the government, is being allowed to dominate all
others in the country today.
E. None of the above options.

45. The pandemic forced a ‘bind’, because hearings had to be conducted through video
conferencing or courts had to shut down altogether. But closing courts would mean blocking
the rule of law. ____________________________

A. However, closing courtrooms and halting jury duty makes sense.

Use Code NIMISHA for maximum Discount

For any query call or whatsapp 9953592800


89

B. The increased strength will double the output of the Supreme Court, which has
decided not to start physical hearings yet due to the COVID-19 situation in the
country.
C. It is time for the world to move from a manual world to a paperless world.
D. Yet at the same time, for large sections of Indians, courts seem to have disappeared
anyway since they had no access to the necessary technology.
E. None of the above options.

46. The internet and social media have changed the contours of the world in which
constitutional rights and legal codes were formulated. _________________________ the
methods and spheres of application must be considered too.

A. So, abused laws often could be used to distort narratives by erasing necessary facts;
B. So, it would not be enough to broaden the principles and create new rights and laws;
C. So, to break this seemingly unending cycle, it is important to understand that its root
cause lies in how laws strip individuals and communities of legal ways to resist;
D. So, it is time that we not only recognise the right to Internet access as a fundamental
right;
E. None of the above options.

47. Even as Indian political leaders were busy comparing the nation with those of China,
Japan and the United States of America and while the prime minister was promising a $5
trillion economy in five years, a small, neighbouring nation - Bangladesh - was forging ahead
in economic terms. ______________________A number of its slogans have come to
naught.

A. During the last five years, India’s per capita gross domestic product was, on an
average, 40 per cent higher than that of Bangladesh.
B. One piece of evidence being talked about is the estimation that India will become
unbelievably richer than Bangladesh by 2025.
C. The government has done all the wrong things possible as far as economic policies
are concerned.
D. This is a time when the government needs to be forthcoming and transparent in its
communications.
E. None of the above options

Use Code NIMISHA for maximum Discount

For any query call or whatsapp 9953592800


90

48. India is well on the way to redefining the word, ‘science’: high-flown statements of belief
without proof. Unable to keep up with this advance, 400 scientists have endorsed a letter to
the chairperson of the Rashtriya Kamdhenu Aayog asking how it has been ‘scientifically
proven’ that keeping cow dung cakes at home reduces radiation. The Aayog was formed to
protect and promote indigenous cattle, and the chairperson has now claimed that a chip
made from cow dung will guard people against radiation from cell phones.
_______________________

A. The innocent cow is, for the Hindutva-minded, a vehicle of unfathomable virtues.
B. During these testing times, even as the scientific research activity has ground to a
halt, coronavirus researchers must remain busy.
C. Being old-fashioned, the letter implies that science is based on evidence, not claims,
so the scientists are asking for the relevant data supporting the chairperson’s
pronouncement.
D. Government called for proposals for research on ‘cowpathy’ under a funding
programme to study cow effluents to produce medicines — for cancer or diabetes.
E. None of the above options.

49. It is not merely that many Indians do not consider Dalits to be human.
___________________________ Reservations can be turned into a tool for vote collection,
but justice is different.

A. Genuine social justice would threaten the traditional interweaving of social, political
and economic power.
B. It looks like the State lacks political will and is taking the side of the perpetrators.
C. Only by saying a big ‘no’ to brutal capitalism can we remedy the problems that we
face today?
D. Social hierarchies are much more rigid in rural India, and an urbanising India offers
better opportunities for aspiring Dalit entrepreneurs.
E. None of the above options.

50.ment through positive discrimination. _______________________

A. Hence, reservations for students in public institutions of higher education and jobs in
the public sector were rightly envisioned to bring about adequate representation to
those, oppressed by caste discrimination.

Use Code NIMISHA for maximum Discount

For any query call or whatsapp 9953592800


91

B. Hence, there is a strong case for making a provision for reservation for the
economically backward in the general category in education and employment to
ensure that they also get reasonable opportunities to advance in life.
C. The use of income or economic criteria for providing reservation for those not
included in the backward classes, or for those belonging to the general sections, is
thus constitutionally invalid.
D. Hence, given the deep inequalities prevalent in access to education and jobs based
on caste status, affirmative action makes a lot of sense.
E. None of the above options.

51. Latest estimates show that nearly 690 million people worldwide are hungry. India retains
the dubious distinction of being the country with the largest population of food-insecure
people unequal access to food. Although these numbers are staggering, hunger cannot be
fought merely by increasing the quantity of food production. _________________________

A. If we can ensure increased food production, then we have a fairly robust system of
food and nutrition security.
B. The inability to harvest fruits and vegetables at remunerative prices during the current
crisis, has not just deprived farmers of incomes and livelihoods, but consumers too
are deprived of micronutrients in their diets.
C. Through a combination of farmers’ cooperation, technological upgrading and
favourable public policies in procurement, pricing and distribution, we can deal with
the fallouts of the pandemic.
D. If anything, the mindless clearing of natural habitats to increase agricultural acreage
will aggravate the climate crisis, which, in turn, could adversely affect food production,
affiliated costs, consumption as well as wastage.
E. None of the above options.

ANSWER

1.(Answer - B. Only a and c

Freedom of expression and the freedom to practise and propagate a person’s own
religion are constitutionally endowed rights in this multi-religious country.

Use Code NIMISHA for maximum Discount

For any query call or whatsapp 9953592800


92

That means, not believing in the deity’s existence is as much an opinion as is


believing in it.

And, statement c is talking about the same thing.

b - Incorrect. The term 'The resolution' is nonspecific.)

2.(Answer - A. Only a and b

These two statements are the results of the fact that rice, wheat and sugar are agri-
commodities in which India’s production, for much of the last decade and more, has
been surpassing its consumption requirement.

c - Incorrect. It doesn't make sense.)

3.(Answer - D. All a, b and c

All are positive in tone and absolutely contextually correct.)

4.(Answer - A. Only a and b

c - Incorrect. It sounds a bit contrary to the actual tone.

a and b - Correct. It's talking in line with the actual context.)

5.(Answer - B. Only a and c

b - Incorrect.
'emperor' is male.
'empress' is the right word.)

6.(Answer - B. Only a and c

b - Incorrect.
'this faith' doesn't make sense.)

7.(Answer - C. C

Use Code NIMISHA for maximum Discount

For any query call or whatsapp 9953592800


93

If you read all the statements carefully, you'll see that statements A, B and D are
talking about LIVE CONCERTS or SHOWS in a very POSITIVE WAY.
While, statement C is talking about a negative aspect of Live Shows for some artists.)

8.(Answer - B. Only a

Only a is talking about live shows positively.

b - Incorrect. It's neither positive nor negative. It's just stating a fact related to live
shows.

c - Incorrect. It's absolutely negative.)

9.(Answer - C. C

If you read all the statements carefully, you'll see that statements A, B and D are
talking about the conflicts between a state and the central government or the
governor.

That means - STATE Vs Outside Entity

But, statement C is talking about the conflicts or disputes between states.)

That means - STATE Vs STATE)

10.(Answer - D. Only b

Only this paragraph is talking about STATE Vs Outside Entity conflict.

a - Incorrect. It's talking about STATE Vs STATE.

c - Incorrect. It's talking about resolving a STATE Vs STATE conflict.)

11.(Answer - A. Only a and b

Use Code NIMISHA for maximum Discount

For any query call or whatsapp 9953592800


94

spat / tiff - a quarrel or disagreement.

covenant - an agreement.)

12.(Answer - D. All a, b and c

All mean - to make (someone) do something against their will.)

13.(Answer - A. Only a and b

a and b - a particular aspect or feature of something.

slant - a sloping position.)

14.(Answer - B. Only a

in the wake of / on account of - following (someone or something), especially as a


consequence.)

15.(Answer - B. Only a and c

a and c - the act of deliberately destroying or damaging public or private property.

fabrication - the action or process of manufacturing or inventing something.)

16.(Answer - D. All a, b and c

All mean - consequence.)

17.(Answer - C. Only b and c

b and c - originate from; be produced by.

subsume - to absorb.)

18.(Answer - D. All a, b and c

Use Code NIMISHA for maximum Discount

For any query call or whatsapp 9953592800


95

All mean - importance.)

19.(Answer - D. All a, b and c

All mean - require (a duty, charge, or penalty) to be undertaken or paid.)

20.(Answer - C. Only b and c

b and c - to decide.

dither - be indecisive.)

21. (Answer - E. All a, b and c

All three are the procedural aspects of death penalty.

Hence, the Supreme Court could be examining these aspects.)

22.(Answer - A. Only a and b

It's said that Capital punishment, therefore, demands that a finely balanced process
precedes it.

That means everything related to death penalty must be fair and balanced.

Hence, a and b are absolutely correct.

c - Incorrect. It's never said that death penalty must be abolished.)

23.(Answer - B. Only a and c

mitigating circumstances - Factors that lessen the severity or culpability of a criminal


act.

aggravating circumstances - Factors that make a felony more severe or worse.

Hence, only a and c are correct.)

Use Code NIMISHA for maximum Discount

For any query call or whatsapp 9953592800


96

24.(Answer - D. All a, b and c

Mitigating (or extenuating) circumstances are factors that tend to lessen the severity
of a crime or its punishment by making the defendant's conduct understandable or
less blameworthy. Mitigating circumstances might include a defendant's young age,
mental illness or addiction, or minor role in the crime.

All these can be the mitigating factors.)

25.(Answer - A. Only a and b

life incarceration / life sentence - life imprisonment.

judicial execution - death penalty.

rule out / knock out - to eliminate.)

26.(Answer - D. e g

The correct arrangement is -

f b a d g c e )

27.(Answer - B. Only a

Single-use plastic carry-bags have been singled out for particular attention and
regular meetings have been held to educate – as well as admonish — people about
their use.

Thus once more, the buck has stopped at the doorstep of the people.
That means, people have started taking responsibility for it.

the buck stops here (idiom) - used to say that one accepts a responsibility and will not try
to give it to someone else.

Hence, only a is correct.

b - Incorrect. It doesn't make sense.

Use Code NIMISHA for maximum Discount

For any query call or whatsapp 9953592800


97

c - Incorrect. It's contradictory.)

28.(Answer - A. Only a and b

Refer to - the products must have a long shelf life

Hence, only a and b are correct.

hold out against - to withstand.


freight - transportation.

give in - surrender.)

29.(Answer - D. All a, b and c

All are vital issues affecting the polity and society.)

30. (Answer - B. Only a and c

These statements are talking about the benefits of Live-streaming, which is the actual
theme of the paragraph.

b - Incorrect. It doesn't fit in the context.)

31.(Answer - A. Only a and b

c - Incorrect. It doesn't make sense.

a and b - RBI should be doing these, according to the above context.)

32. (Answer - A. Only a and b

c - Incorrect. It's absolutely contradictory.


Questions like these shouldn't draw attention away from the despair of the present,
because these questions are related to the despair of the present.

Hence, a and b are absolutely correct.)

Use Code NIMISHA for maximum Discount

For any query call or whatsapp 9953592800


98

33.(Answer - D. All a, b and c

All these are the urgent needs as expressed in the above paragraph.)

34.(Answer - D. All a, b and c

All these are justifying the phrase - the future promised a great deal.
Because, all these are positive promises.)

35.(Answer - C. Only b and c

The phrase must be talking about a positive impact.

a - Incorrect. It's negative.


unnerve - to scare.)

36.(Answer - B. Only b

The sentence has 'while' in the beginning.


Hence, the phrase in the blank must contradict - caste articulations are fierce and
strong across the country.

Hence, only b is correct.

a - Incorrect. It's same as - caste articulations are fierce and strong across the
country.

c - Incorrect.
make-believe (adj) - imaginary.)

37.(Answer - A. The problem with the 2019 report is that it no longer enlists causes,
such as crop failure or debt, for the suicides.

The line - 'The reason for this, according to the Union minister of state,...... ' indicates that
the reason behind the report not enlisting causes for the suicides is that 'several states
and Union territories gave no figures for farmer suicides.'

B - It doesn't justify the sentence right after it.


C - It's just another data and not justifying the line right after it.

Use Code NIMISHA for maximum Discount

For any query call or whatsapp 9953592800


99

D - It's only talking about the report and not justifying the line right after it.)

38.(Answer - B. The most troubling is its use of lofty rhetoric in place of meaningful
interventions.

lofty rhetoric - majestic speech or writing intended to be effective and influence people.

The context is - The tone is critical here. The author is criticizing the Centre.
The Centre is shedding crocodile tears for the absence of data on the deaths of frontline
health workers due to Covid, because they're praising their contribution publicly but doing
very little in managing their crises.
The most troubling is its use of lofty rhetoric in place of meaningful interventions.
This means - The Centre is being sad by the absence of data on the deaths of the health
workers but it has several deductions and one of the them is that the centre is using majestic
speech in praising them, but not doing any significant or meaningful progress.

A - It is a positive statement and indirectly praising the Centre, which is contradictory to the
actual tone.
C - It has no relation to the main theme.
D - It's a general statement. It's not connected to the actual context.)

39.(Answer - C. Men and women can slurp and gobble away without attracting the
raised eyebrow.

slurp and gobble away - eat and drink.


gendered delicateness - delicacy or enjoyment specific to people of one particular gender.

The context is - In these days of strong gender equality, the fun which used to be mainly
enjoyed by men, can now be enjoyed equally by women too. There is no such job as 'male
job' or 'female job', now.
Both men and women can eat and drink without making others feel surprising or
disapproving (without attracting the raised eyebrow).

A - It says gender equality is still a far cry but the context says, gender equality is already
there. So, it's contradictory.
B - It's a general statement. Besides, it's talking about injustice, which is not mentioned
anywhere in the paragraph.
D - It's a completely different statement. It has no relation to the main context.)

Use Code NIMISHA for maximum Discount

For any query call or whatsapp 9953592800


100

40.(Answer - B. This makes it important for the world to wish for the resurrection of
empathy and kindness.

vanishing human traits - disappearing emotional and behavioral characteristics of human


beings like, kindness, empathy.
They're disappearing because, these virtues aren't so common these days.

The context is - though, the vaccine is showing the light of hope, it alone cannot wipe away
the pandemic. It's sympathy and kindness that can work as vaccines to tackle the viruses of
hate, violence and discrimination that torment the people considered as a collective
unit(global body politic).

A & D- These statements are talking about anti-viral medicines and therapeutics agents.
These aren't vanishing human traits.

C - This statement is carrying the actual tone but it's talking about kind-hearted humans,
not human traits.)

41.(Answer - D. Filling the legislative, medical and societal gaps is only part of the
challenge.

The context is - The goals of the Centre’s flagship Beti Bachao, Beti Padhao scheme are
noble, but its impact assessment has not been conducted by the Narendra Modi-led
government. Filling legislative, medical and societal gaps alone will not be sufficient;
positive action must be accompanied by political will to raise public awareness as well as
strengthen women’s agency over their own bodies.

A - It's talking about creating fear in a particular community, hence this statement is out of
the context.
B - It's nonspecific. The paragraph hasn't mentioned anything about 'needy citizens' or
'beneficiaries'.
C - It's a positive statement and contradictory to the critical tone of the paragraph.)

42.(Answer - C. there appears to be an incongruity in the results of different tests


geared towards the same purpose.

incongruity - discrepancy or variance.

Use Code NIMISHA for maximum Discount

For any query call or whatsapp 9953592800


101

The context is - CSE researchers made an accusation that the honey produced by major
Indian brands is impure and mixed with sugar syrup.
There also appears to be discrepancy in the results of different tests of honey. As a
consequence, the environment watchdog(administration) had selected 13 popular honey
brands to assess the purity of the product.

A - Irrelevant. It doesn't make any sense in this context that public awareness should be
raised so that citizens know that the products they are consuming are tasty.
B - Not aligning in this particular context. Assisting beekeeping communities, whose
livelihoods have been decimated by the pandemic, is a different task. It may be indirectly
connected to the context.
D - It is contradictory. If Researcher's claim is considered false, why does the environment
watchdog have to assess the purity?)

43.(Answer - A. According to the report, from the data available on India, the nation’s
economy may have undergone the sharpest diminution in average wages.

undergone - suffered.
diminution - decline.
reticent - restrained.

The context is - India has not been restrained in revealing a number of data estimates,
including those pertaining to unemployment and earnings, publicly. According to the report
the nation’s economy may have suffered the sharpest decline in average wages.
That means, India didn't hide its unemployment data.

B - It's contradictory, as it's talking about economic growth, whereas the context is about
unemployment and economic downturn.

C - It's talking about worldwide unemployment data, whereas the context is solely about
India’s unemployment data.

D - False according to the context. India has not been reluctant(unwilling) in revealing its
unemployment rate.)

Use Code NIMISHA for maximum Discount

For any query call or whatsapp 9953592800


102

44.(Answer - C. The ruling party is not the only party complicit in this institutional
apathy.

complicit - involved in or knowing about a crime or some activity that is wrong.

The context is - The deep crisis in India’s agricultural is the consequence of causing serious
harm to farmers’ interests and agricultural productivity by all the governments in a row. It's a
continuous process. And the Ruling party isn't the only party who has been involved in
knowing about the institutional unconcern. But, they ruling party seems to be trying to
portray the farmer's protest as threatening or wicked.

A, B & D - they're all supporting the last statement. They all are saying that the ruling party
and its leader have been working against the protest by farmers and trying to prove them
wrong and trying to make those protests unheard. Hence, the starting preposition of the last
statement 'but' doesn't make any sense.

demonize - portray as wicked and threatening.)

45.(Answer - D. Yet at the same time, for large sections of Indians, courts seem to
have disappeared anyway since they had no access to the necessary technology.

The context is - the biggest problem that has come with the pandemic is that access to
justice became conditional on access to technology. But closing courts mean blocking the
rule of law. At the same time, for large sections of Indians, who have no access to
technology, the courts have already disappeared, as they can't attend hearings through
video conferencing.

A - It's contradictory and nonspecific. How the closing of the judiciary system makes
sense, isn't clear.

B - Out of the context. The paragraph hasn't mentioned anything about 'increased
strength of the Supreme Court.)

C - It's a general and nonspecific statement. It's not clearly mentioning anything.)

Use Code NIMISHA for maximum Discount

For any query call or whatsapp 9953592800


103

46.(Answer - B. So, it would not be enough to broaden the principles and create new
rights and laws;

broaden - to extend.

The context is - The internet and social media have changed the shape(contours) of the
world in which constitutional rights and legal codes were developed. So, extending the
principles and creating new rights and laws are not sufficient alone; the methods of applying
those and the domains of their application are to be taken into account too.

A - It has no connection to the actual context. It's rather a general statement.


abused law - misused law.
distort - to twist.

C - The term 'cycle' is not so appropriate here. Besides, it's a negative statement, talking
about how laws deprive individuals of legal ways to resist. It has no connection to the actual
context.

D - It's out of the context. The actual context talks mainly about the how laws and principles
can be modified.
But, this statement talks about right to internet.)

47.(Answer - C. The government has done all the wrong things possible as far as
economic policies are concerned.

The context is - At a time when, Indian political leaders were busy comparing India with
those of China, Japan and the United States of America and while the prime minister was
promising a $5 trillion economy in five years, Bangladesh, our neighbouring country, was
taking the lead and advancing rapidly in economic terms. They were ahead of us in the
advancement. The government of India has done all the wrong things possible as far as
economic policies are concerned. A number of its slogans don't meant anything because
they couldn't be realised and successful.
The tone is critical here.

forge ahead - take the lead or make good progress.


come to naught - be unsuccessful.

Use Code NIMISHA for maximum Discount

For any query call or whatsapp 9953592800


104

A - It's a statistic and it's contradictory to the critical tone of the paragraph, as it's a positive
statistic for India.
B - It's contradictory. The paragraph is talking about India’s failure in economic terms and
Bangladesh's better economic progress than that of India. So, 'India will be unbelievably
richer than Bangladesh' is in contrast to the actual tone.
D - Not specific. This statement doesn't clearly indicate anything. Sounds more like out of
the context.)

48.(Answer - C. Being old-fashioned, the letter implies that science is based on


evidence, not claims, so the scientists are asking for the relevant data supporting the
chairperson’s pronouncement.

pronouncement - announcement.

The context is - Rashtriya Kamdhenu Aayog is claiming that cow dung cakes at home
reduces radiation and a chip made from cow dung will protect people from all radiations.
Now, the scientists are asking for evidence, backing this claim, as Science is based on
evidences, not claims. They've sent to letter to the Aayog asking for evidence.

A - It's a general statement. It seems to have no connection with the actual context.

B - Irrelevant. The paragraph hasn't mentioned anything about Coronavirus.

D - It's an information which has no link to the actual context.)

49.(Answer - A. Genuine social justice would threaten the traditional interweaving of


social, political and economic power.

interweave - blend closely.

The context is - It's not infrequent that Dalits aren't considered to be humans by many
Indians. Any justice that's genuine, doesn't discriminate, and thus it would threaten the
traditional bending of social, political and economic power. This means, India's tradition is
caste-based. The political, economic and social power has been structured keeping the

Use Code NIMISHA for maximum Discount

For any query call or whatsapp 9953592800


105

castes in mind. In India's tradition, Dalits are neglected and deprived of justice. And, the
government's support for caste-based reservations can be just a tool to attract more votes
because when it comes to justice, the government doesn't act in favour of the Dalits.

The paragraph exposes the Government's hypocritical mindset.

B - Out of the context. The paragraph hasn't mentioned anything about the 'perpetrators'.

C - Out of the context. The paragraph hasn't mentioned anything about 'capitalism'.

D - It's contradictory, as it's talking about urbanising India offering better opportunities for
aspiring Dalit entrepreneurs, which is contradictory to the actual tone.)

50.(Answer - B. Hence, there is a strong case for making a provision for reservation
for the economically backward in the general category in education and employment
to ensure that they also get reasonable opportunities to advance in life.

The context is - India's prevalent reservation system is only caste-based. But in reality,
poverty denies equality of opportunity to individuals in education and employment. It denies
them the opportunity of a decent and sustainable livelihood. But, the current reservation
system doesn't attach any importance to the economically backward section. Hence, there is
a strong case for making a provision for reservation for the economically backward in
the general category in education and employment to ensure that they also get reasonable
opportunities to advance in life.

Basically, the paragraph is talking about a major flaw in India's reservation system.

A - It's contradictory as it supports only the caste-based reservation.

C - It delivers exactly the opposite tone. If the use of income or economic criteria for
providing reservation is invalid, how would it be possible to deliver justice to the
economically backward section?

D - It supports the caste-based reservation. Hence, it's contradictory.)

Use Code NIMISHA for maximum Discount

For any query call or whatsapp 9953592800


106

51.(Answer - D. If anything, the mindless clearing of natural habitats to increase


agricultural acreage will aggravate the climate crisis, which, in turn, could adversely
affect food production, affiliated costs, consumption as well as wastage.

if anything - used when saying that what people may believe is not true, and the opposite
may be true.
acreage - an area of land, typically when used for agricultural purposes.

The context is - Latest estimates show that nearly 690 million people worldwide are hungry.
India is probably not proud of being the country with the largest population of food-insecure
people unequal access to food. Although these numbers are shocking, increasing the
quantity of food production only isn't enough to fight hunger. Besides, destroying natural
habitats in order to increase food production will create another problem like climate crisis,
which in turn, can adversely affect food production.

dubious distinction - used ironically to describe something bad or undesirable as if it were


an honor or achievement.

A - It's contradictory. The context says that increasing food production only, will not be
sufficient. But, this statement says the opposite.

B - It portrays a different scenario altogether. It is not in connection with the actual context.

C - Out of the context. The paragraph hasn't mentioned anything about the 'pandemic'.)

Use Code NIMISHA for maximum Discount

For any query call or whatsapp 9953592800


107

CHAPTER 3 - CONNECTOR

Directions(1-2): You are presented with two statements followed by three possible
connectors that can join the two sentences to form one sentence. Identify which
connector/connectors can join both the sentences in such a way that meaning of the
two statements would not change. If none of the connectors are correct, mark E as
your answer.

1.
I. He went through with his faulty plan.
II. All his friends advised him to abandon it.

a. since
b. although
c. while

A. Only a and b B. Only a and c C. Only b and c D. All a, b and c E. None of a, b and
c

2.
I. He could find himself making enemies as well as friends.
II. He proceeds with the utmost caution.

a. unless
b. even so
c. nevertheless

A. Only b B. Only a and c C. Only a D. All a, b and c E. None of a, b and c

Directions(3-6): You are presented with two statements followed by three possible
connectors that can join the two sentences to form one sentence. Identify which
connector/connectors can join both the sentences in such a way that meaning of the
two statements would not change. If none of the connectors are correct, mark E as
your answer.

Use Code NIMISHA for maximum Discount

For any query call or whatsapp 9953592800


108

3.
I. Her teachers said she was a slow learner.
II. In reality, she was actually quite quick on the uptake.

a. whereas
b. in addition
c. while

A. Only a and b B. Only b C. Only a and c D. All a, b and c E. None of a, b and c

4.
I. The cost of transport is a major expense for an industry.
II. Factory location is an important consideration.

a. hence
b. consequently
c. accordingly

A. Only a and b B. Only b and c C. Only a and c D. All a, b and c E. None of a, b


and c

5.
I. The platforms say that there are no monetary repercussions for delivery executives
failing to meet the deadlines.
II. A fall in customer ratings can adversely affect the performance assessment and,
eventually, the wages of these workers.

a. since
b. but
c. at the same time

A. Only a and b B. Only b and c C. Only a and c D. All a, b and c E. None of a, b


and c

Use Code NIMISHA for maximum Discount

For any query call or whatsapp 9953592800


109

6.
I. That films not just from India but also from Pakistan, Nepal and Bhutan are getting
noticed internationally is good.
II. Given the disproportionate resources and clout that they command in comparison to
their peers in other South Asian nations, it is Bollywood and India’s regional film
industries that must shoulder the principal responsibility for breaking through globally
the way Korean movies have done.

a. still
b. however
c. yet

A. Only a and b B. Only b and c C. Only a and c D. All a, b and c E. None of a, b


and c

Directions(7-11): You are presented with two statements followed by three possible
connectors that can join the two sentences to form one sentence. Identify which
connector/connectors can join both the sentences in such a way that meaning of the
two statements would not change. If none of the connectors are correct, mark E as
your answer.

7.
I. Admittedly, India’s showing is consistent with the global trend: 90 per cent of the
countries surveyed have recorded a fall on the global HDI index.
II. This is the first time since 1990 that the global HDI value has decreased for two years
in a row owing to factors such as the Covid-19 pandemic and the Ukraine war.

a. In point of fact
b. As a matter of fact
c. If truth be told

A. Only a and b B. Only a and c C. Only b and c D. All a, b and c E. None of a, b


and c

Use Code NIMISHA for maximum Discount

For any query call or whatsapp 9953592800


110

8.
I. The Shanghai Cooperation Organisation summit was expected to serve as a platform
to showcase that the Russian president, Vladimir Putin, treated as a pariah by much
of the West, is still not isolated in Eurasia despite his war in Ukraine.
II. The summit has, instead, underscored the dramatic fall in stature and credibility that
Mr Putin is facing today as a cocktail of battlefield losses and diplomatic setbacks.

a. Be that as it may
b. Howbeit
c. By the same token

A. Only a and b B. Only a and c C. Only b and c D. All a, b and c E. None of a, b


and c

9.
I. Of course, what ultimately matters most for Mr Putin is how his fellow Russians view
him.
II. For him, there too, holes are appearing in the cloak of invincibility that he has worn for
two decades.

a. Sad to say
b. As luck would have it
c. Correspondingly

A. Only a B. Only b C. Only c D. Only a and b E. None of a, b and c

10.
I. India is expected to have around 21 cheetahs in the next 15 years and this would
require intensive management and end up consuming a disproportionate volume of
thinning resources.
II. It would distract efforts from critical conservation priorities; for instance, the Kuno
National Park in Madhya Pradesh was originally earmarked for the relocation of some
of Gujarat’s lions that are vulnerable to epidemics.

a. Worse

Use Code NIMISHA for maximum Discount

For any query call or whatsapp 9953592800


111

b. More severely
c. Thank heavens

A. Only a and b B. Only a and c C. Only b and c D. All a, b and c E. None of a, b


and c

11.
I. The Middle East has long been consumed by a ceaseless sequence of wars over
land, where guns, bombs, missiles and, more recently, drones have been central
characters.
II. Water is entering the fray as a key point of conflict, with science as the weapon of
choice.

a. At this moment in time


b. In this day and age
c. At the minute

A. Only a and b B. Only a and c C. Only b and c D. All a, b and c E. None of a, b


and c

Phrase Connectors

Direction(12-14): You are presented with two statements followed by three possible
PHRASES that can join the two sentences to form a long and meaningful sentence.
Identify which PHRASE(s) can join both the sentences in such a way that the meaning
of the two statements would not change. If none of the connectors are correct, mark E
as your answer.

12.
I. There is, as always, the enduring pull of the highest peaks, a magnetism that, while
apparent, is difficult to describe.
II. This mystical pull factor is leading to the congregation of an unsustainable number of
visitors, pushing both men and the mountains towards catastrophe.

a. But there is rising concern that

Use Code NIMISHA for maximum Discount

For any query call or whatsapp 9953592800


112

b. However, there is the reassurance that


c. What is perturbing is that

A. Only a and b B. Only a and c C. Only b and c D. All a, b and c E. None of a, b


and c

13.
I. The Everest, for instance, has been described, not without reason, as the world’s
highest garbage dump; microplastic has been discovered near its peak.
II. Climbers, porters, trekkers, tourists are leaving a rather sorry trail on the snow —
garbage comprising empty oxygen cylinders, abandoned tents, used containers, bio
waste, and, occasionally, even the corpses of fallen climbers.

a. As if that is not worrying enough


b. It's a regrettable fact that
c. On the spur of the moment

A. Only a and b B. Only a and c C. Only b and c D. All a, b and c E. None of a, b


and c

14.
I. If religion is India’s ultimate reality, shouted raucously from rooftops and driven home
with knife and bludgeon, race is the reality that dare not speak its name in Britain.
II. There probably still lurks the inhibition that prompted Robert Cecil, three times prime
minister, notoriously to wonder 136 years ago whether “a British constituency would
elect a black man” to Parliament.

a. Beneath the camouflage of political correctness


b. Having said that
c. But as a matter of fact, in the actual reality of British politics

A. Only a and b B. Only a and c C. Only b and c D. All a, b and c E. None of a, b


and c

Use Code NIMISHA for maximum Discount

For any query call or whatsapp 9953592800


113

Direction(15-24): Select the phrase/connector from the given three options which can
be used to form a single sentence from the two sentences given below, implying the
same meaning as expressed in the statement.

15.
I. The world is more connected than ever before now.
II. But, it seems to be an irony that now people are assailed by a sense of crushing
loneliness.

a. It may seem ironic that now when…..


b. Being assailed by a sense of crushing loneliness….
c. The fact that people are now assailed by a sense of…

A. Only a and b B. Only a C. Only a and c D. All a, b and c E. None of a, b and c

16.
I. At the click of a mouse, everything — work, leisure, recreation, company — can be
ordered.
II. But, the experience of organic bonding, the pleasure of in-person encounters,
remains irreplaceable to the human mind.

a. Even though everything — work, leisure, recreation, company —......


b. The experience of organic bonding,.....
c. To the human mind…….

A. Only a and b B. Only a and c C. Only b and c D. All a, b and c E. None of a, b


and c

17.
I. It's a fact that certain religions do not allow representations or images.
II. On the contrary, there is no dearth of statues accessible to the public of religious
leaders and their sayings.

a. The fact that there is no dearth of statues…


b. Even taking into account the fact that certain religions….

Use Code NIMISHA for maximum Discount

For any query call or whatsapp 9953592800


114

c. Statues being accessible to the….

A. Only a and b B. Only a and c C. Only b and c D. All a, b and c E. None of a, b


and c

18.
I. Today's smartphones are incredibly capable.
II. They are amazingly fast, have the ability to multitask efficiently and help us
communicate effectively.

a. By being amazingly fast,......


b. In addition to having the ability to multitask efficiently…
c. Being incredibly capable,....

A. Only a and b B. Only a C. Only b and c D. All a, b and c E. None of a, b and c

19.
I. A diverse set of rock samples has been collected by a Nasa robot.
II. It will soon be deposited on the surface, awaiting carriage to Earth by later missions.

a. A Nasa robot has collected….


b. The surface, awaiting carriage to Earth….
c. By collecting a diverse set of rock samples…

A. Only a and b B. Only a C. Only b and c D. All a, b and c E. None of a, b and c

20.
I. The dry conditions over the past three months have been compared by us to average
conditions since the beginning of this century.
II. The purpose is to build up a picture of how extreme recent weather patterns have
been.

Use Code NIMISHA for maximum Discount

For any query call or whatsapp 9953592800


115

a. We have compared the dry conditions over the….


b. For building up a picture of how extreme…..
c. To average conditions since the beginning….

A. Only a and b B. Only a and c C. Only b and c D. All a, b and c E. None of a, b


and c

21.
I. An extended period of high temperatures that lasted more than two months was
experienced by China this summer.
II. According to China's Meteorological Administration, it's the longest since records
began in the 1960s.

a. This summer, China…..


b. China's Meteorological Administration…
c. In the 1960s….

A. Only a and b B. Only a C. Only b and c D. All a, b and c E. None of a, b and c

22.
I. The fact that particular colours of plumage put birds at greater risk has been revealed
by a recent study.
II. The risk is of being taken from the wild and sold.

a. A recent study has revealed…


b. Particular colours of plumage….
c. A recent study by revealing…

A. Only a and b B. Only a and c C. Only b and c D. All a, b and c E. None of a, b


and c

Use Code NIMISHA for maximum Discount

For any query call or whatsapp 9953592800


116

23.
I. Prof Senior and her colleagues carried out what was essentially a stocktake of the
species most commonly bought and sold in the songbird markets of Asia.
II. The idea was to understand the threats to wild birds.

a. To understand the threats….


b. By carrying out what was essentially a stocktake…
c. Understanding the threats to wild birds…

A. Only a and b B. Only a and c C. Only b and c D. All a, b and c E. None of a, b


and c

24.
I. The animals made the transcontinental journey in a modified passenger Boeing 747
plane.
II. With them were wildlife experts, veterinary doctors and three biologists.

a. Wildlife experts, veterinary doctors and three biologists….


b. During the transcontinental journey…
c. Being escorted by….

A. Only a and b B. Only a and c C. Only b and c D. All a, b and c E. None of a, b


and c

ANSWER

1.(Answer - C. Only b and c

He went through with his faulty plan although / while all his friends advised him to abandon
it.)

2.(Answer - C. Only a

Use Code NIMISHA for maximum Discount

For any query call or whatsapp 9953592800


117

He could find himself making enemies as well as friends unless he proceeds with the
utmost caution.)

3.(Answer - C. Only a and c

These are two contrasting statements.

quick on the uptake - quick to learn.)

4.(Answer - D. All a, b and c

hence / consequently / accordingly - as a result or for that reason.)

5.(Answer - B. Only b and c

The two statements are contrasting statements.

at the same time / but - used to introduce a phrase or clause contrasting with what has
already been mentioned.)

6.(Answer - D. All a, b and c

The first statement is positive and the second statement is reflecting a negative
scenario and the required measure to deal with it.
Hence, all three contrast-category connectors can be used.)

7.(Answer - D. All a, b and c

All used to emphasize the truth of an assertion, especially one contrary to what might
be expected or what has been asserted.)

8.(Answer - A. Only a and b

a and b mean - BUT.

The summit was expected to do something, but in reality it did something else.)

9.(Answer - A. Only a

Use Code NIMISHA for maximum Discount

For any query call or whatsapp 9953592800


118

The second statement is surely a disappointment for Putin.


Hence, only a is correct.

Sad to say - unfortunately.

As luck would have it - fortunately.


Correspondingly - similarly.

invincibility - the quality of being too powerful to be defeated or overcome.


cloak - sth that hides.)

10.(Answer - A. Only a and b

The second statement describes a disadvantage that is the result of the first
statement.
Hence, only a and b are correct.

Both are ADVERBS.

Thank heavens - luckily.)

11.(Answer - D. All a, b and c

Statement II is an additional statement.


Hence, all three are correct.

All mean - NOW.)

12.(Answer - B. Only a and c

The second statement expresses a cause of anxiety.


Hence, only a and c are correct.

b - Incorrect. It's contradictory.)

13.(Answer - A. Only a and b

The second statement is a supportive statement to the first statement.


As the entire tone is negative, only a and b are correct.

Use Code NIMISHA for maximum Discount

For any query call or whatsapp 9953592800


119

c - UNEXPECTEDLY.
It's incorrect and contradictory.)

14.(Answer - D. All a, b and c

The second statement is contrary to the fact described in the first statement.

The fact described in the first statement - race is the reality that dare not speak its name
in Britain.

The contradictory fact - a British constituency would elect a black man” to Parliament.
Hence, all three are correct.

Having said that - in spite of that.


As a matter of fact - in truth.)

15.(Answer - C. Only a and c

It may seem ironic that now when the world is more connected than ever before
people are assailed by a sense of crushing loneliness.

The fact that people are now assailed by a sense of crushing loneliness when the
world is more connected than ever before seems to be an irony.)

16.(Answer - D. All a, b and c

Even though everything — work, leisure, recreation, company — can be ordered at


the click of a mouse, the experience of organic bonding, the pleasure of in-person
encounters, remains irreplaceable to the human mind.

The experience of organic bonding, the pleasure of in-person encounters, remains


irreplaceable to the human mind, even though everything — work, leisure, recreation,
company — can be ordered at the click of a mouse.

To the human mind, the experience of organic bonding, the pleasure of in-person
encounters, remains irreplaceable, even though everything — work, leisure,
recreation, company — can be ordered at the click of a mouse.)

Use Code NIMISHA for maximum Discount

For any query call or whatsapp 9953592800


120

17.(Answer - A. Only a and b

Even taking into account the fact that certain religions do not allow representations
or images, there is no dearth of statues accessible to the public of religious leaders
and their sayings.

The fact that there is no dearth of statues accessible to the public of religious leaders
and their sayings is contrary to the fact that certain religions do not allow
representations or images.)

18.(Answer - B. Only a

By being amazingly fast, having the ability to multitask efficiently and helping us
communicate effectively, today's smartphones are incredibly capable.)

19.(Answer - B. Only a

A Nasa robot has collected a diverse set of rock samples that it will soon deposit on
the surface, awaiting carriage to Earth by later missions.)

20.(Answer - A. Only a and b

We have compared the dry conditions over the past three months to average
conditions since the beginning of this century, to build up a picture of how extreme
recent weather patterns have been.

For building up a picture of how extreme recent weather patterns have been, we have
compared the dry conditions over the past three months to average conditions since
the beginning of this century.)

21.(Answer - B. Only a

This summer, China experienced an extended period of high temperatures that lasted
more than two months, the longest since records began in the 1960s, according to
China's Meteorological Administration.)

Use Code NIMISHA for maximum Discount

For any query call or whatsapp 9953592800


121

22.(Answer - A. Only a and b

A recent study has revealed that particular colours of plumage put birds at greater
risk of being taken from the wild and sold.

Particular colours of plumage put birds at greater risk of being taken from the wild
and sold and this has been revealed by a recent study.)

23.(Answer - A. Only a and b

To understand the threats to wild birds, Prof Senior and her colleagues carried out
what was essentially a stocktake of the species most commonly bought and sold in
the songbird markets of Asia.

By carrying out what was essentially a stocktake of the species most commonly
bought and sold in the songbird markets of Asia, Prof Senior and her colleagues
wanted to understand the threats to wild birds.)

24.(Answer - D. All a, b and c

Wildlife experts, veterinary doctors and three biologists accompanied the animals as
they made the transcontinental journey in a modified passenger Boeing 747 plane.

During the transcontinental journey in a modified passenger Boeing 747 plane, the
animals were accompanied by wildlife experts, veterinary doctors and three
biologists.

Being escorted by wildlife experts, veterinary doctors and three biologists, the
animals made the transcontinental journey in a modified passenger Boeing 747
plane.)

Use Code NIMISHA for maximum Discount

For any query call or whatsapp 9953592800


122

CHAPTER 4 - CLOZE TEST

Level - Easy

Directions(1-10): In the following passage, some of the words have been left out, each
of which is indicated by a letter. Find the suitable word from the options given against
each letter and fill up the blanks with appropriate words to make the paragraph
meaningful.

With inflation, as measured by the consumer price index, in August going back to 7 per cent,
and the wholesale price index coming in at 12.4 per cent, one thing is clear: India is not out
of the _____M_____ on inflation management. If the September inflation remains higher
than the RBI’s upper tolerance band of 6 per cent, the central bank, especially its monetary
policy committee (MPC) members, will have to _____N____ to the Centre, and in a way to
the nation, why it has _____O_____ to keep inflation below 6 per cent for three consecutive
quarters. That’s as per the 2016 inflation targeting policy. The ______P______ of the RBI
and its MPC members will be at _____Q______.

However, these are not normal times. First Covid-19 hit the world, then the Russia and
Ukraine conflict _____R_____, and now climate change (heat wave) is impacting global
commodity prices. In an _____S____ world, it would be ____T____ to look at India’s
performance in relation to other major countries. In comparison with, say, the US and most
European countries, where inflation is a tad higher (8 to 12 per cent) and GDP growth likely
to be much lower, India has done fairly well. India is certainly much better placed than
countries like Turkey where inflation in July was ______U______ at 80 per cent, or our
neighbours — Pakistan, with inflation at 27 per cent, and Sri Lanka at 64 per cent. On
average, even compared to the country’s past record, say during the UPA period (2004-05
to 2013-14), when inflation averaged 7.9 per cent (and GDP growth was at 7.7 per cent),
India has done _____V_____ well during the NDA period (2014-15 to 2022-23) with 5.1 per
cent inflation but has done poorer on GDP growth at 5.6 per cent.

With inflation, as measured by the consumer price index, in August going back to 7 per cent,
and the wholesale price index coming in at 12.4 per cent, one thing is clear: India is not out
of the _____M_____ on inflation management.

Use Code NIMISHA for maximum Discount

For any query call or whatsapp 9953592800


123

1. M
a. woods
b. window
c. red

A. Only a B. Only a and b C. Only b D. Only c E. None of a, b and c

If the September inflation remains higher than the RBI’s upper tolerance band of 6 per cent,
the central bank, especially its monetary policy committee (MPC) members, will have to
_____N____ to the Centre,

2. N
a. delineate
b. elucidate
c. obscure

A. Only a and b B. Only a and c C. Only b and c D. All a, b and c E. None of a, b


and c

and in a way to the nation, why it has _____O_____ to keep inflation below 6 per cent for
three consecutive quarters.

3. O
a. miscarried
b. lofted
c. flunked

A. Only a and b B. Only a and c C. Only b and c D. All a, b and c E. None of a, b


and c

The ______P______ of the RBI and its MPC members will be at _____Q_____.

4. P
a. credibility
b. plausibility

Use Code NIMISHA for maximum Discount

For any query call or whatsapp 9953592800


124

c. standing

A. Only a and b B. Only a and c C. Only b and c D. All a, b and c E. None of a, b


and c

The ______P______ of the RBI and its MPC members will be at _____Q_____.

5. Q
a. stake
b. jeopardy
c. risk

A. Only a and b B. Only a and c C. Only b and c D. All a, b and c E. None of a, b


and c

However, these are not normal times. First Covid-19 hit the world, then the Russia and
Ukraine conflict _____R_____, and now climate change (heat wave) is impacting global
commodity prices.

6. R
a. broke out
b. flared up
c. set in

A. Only a and b B. Only a and c C. Only b and c D. All a, b and c E. None of a, b


and c

In an _____S____ world, it would be ____T____ to look at India’s performance in relation to


other major countries.

7. S
a. interconnected
b. interrelated
c. interlinked

Use Code NIMISHA for maximum Discount

For any query call or whatsapp 9953592800


125

A. Only a and b B. Only a and c C. Only b and c D. All a, b and c E. None of a, b


and c

In an _____S____ world, it would be ____T____ to look at India’s performance in relation to


other major countries.

8. T
a. ludicrous
b. cogent
c. judicious

A. Only a and b B. Only a and c C. Only b and c D. All a, b and c E. None of a, b


and c

India is certainly much better placed than countries like Turkey where inflation in July was
______U______ at 80 per cent, or our neighbours — Pakistan, with inflation at 27 per cent,
and Sri Lanka at 64 per cent.

9. U
a. raging
b. being at its height
c. rampaging

A. Only a and b B. Only a and c C. Only b and c D. All a, b and c E. None of a, b


and c

On average, even compared to the country’s past record, say during the UPA period (2004-
05 to 2013-14), when inflation averaged 7.9 per cent (and GDP growth was at 7.7 per cent),
India has done _____V_____ well during the NDA period (2014-15 to 2022-23) with 5.1 per
cent inflation but has done poorer on GDP growth at 5.6 per cent.

10. V
a. fairly
b. passably

Use Code NIMISHA for maximum Discount

For any query call or whatsapp 9953592800


126

c. somewhat

A. Only a and b B. Only a and c C. Only b and c D. Only c E. All a, b and c

Level - Moderate

Direction(11-18): In the following passage, some of the words have been highlighted
in bold. First read the passage and try to understand what it is about. Then replace
the highlighted words with the correct options in order to make the paragraph
grammatically and contextually correct. In case the highlighted word is correct as it
is, mark 'No replacement required' as the right answer.

Cheetahs are back in India for the first time in more than 70 years. They are, however, not
the Asiatic variety that once ___M___dissipated large parts of the country — the species
was declared extinct in India in 1952, a victim of ____N____unabridged hunting and habitat
loss. Less than 100 of these animals survive in Iran today. On Saturday, Prime Minister
Narendra Modi released eight ____O____greenhorns of the African species at the Kuno
sanctuary in Madhya Pradesh. The animals were flown in from Namibia, which today has
the highest cheetah population. The translocation has triggered a debate amongst
conservation scientists with a section amongst them arguing that the 750 sq-km national
park could limit the movement of the animals that prefer a much larger range. But others do
not take such a ____P____reassuring view and point to the cheetah’s adaptability across a
range of habitats. PM Modi referred to the African big cats as the country’s guests. It’s up to
the wildlife authorities to help these cheetahs make Kuno their home.

One of the ____Q____spinoffs of this debate is the spotlight wildlife-related issues have
received in the past few days. The introduction of the new species has led to conversations
on human-animal conflict — a problem that doesn’t always get the attention it requires.
Some of the successes of India’s wildlife protection programme have come at a cost. In
several parts of the country today, humans and protected species live in
____R____receptivity. Crop _____S_____fosterage and attacks on livestock — even
humans — have become common. Only recently have studies begun to be undertaken on
the carrying capacity of national parks and sanctuaries. They are yet to be used in a
meaningful manner to restore a _____T____semblance of harmony between wildlife and
people living in the vicinity of the protected areas (PAs).

Use Code NIMISHA for maximum Discount

For any query call or whatsapp 9953592800


127

Cheetahs are back in India for the first time in more than 70 years. They are, however, not
the Asiatic variety that once ___M___dissipated large parts of the country — the species
was declared extinct in India in 1952, a victim of ____N____unabridged hunting and habitat
loss.

11. M
a. roamed
b. rambled
c. meandered

A. Only a and b B. Only a and c C. Only b and c D. All a, b and c E. No


replacement required

Cheetahs are back in India for the first time in more than 70 years. They are, however, not
the Asiatic variety that once ___M___dissipated large parts of the country — the species
was declared extinct in India in 1952, a victim of ____N____unabridged hunting and habitat
loss.

12. N
a. unchecked
b. unbridled
c. ungoverned

A. Only a and b B. Only a and c C. Only b and c D. All a, b and c E. No


replacement required

On Saturday, Prime Minister Narendra Modi released eight ____O____greenhorns of the


African species at the Kuno sanctuary in Madhya Pradesh.

13. O
a. neophytes
b. cubs
c. offsprings

Use Code NIMISHA for maximum Discount

For any query call or whatsapp 9953592800


128

A. Only a and b B. Only a and c C. Only b and c D. All a, b and c E. No


replacement required

But others do not take such a ____P____reassuring view and point to the cheetah’s
adaptability across a range of habitats.

14. P
a. defeatist
b. downbeat
c. cynical

A. Only a and b B. Only a and c C. Only b and c D. All a, b and c E. No


replacement required

One of the ____Q____spinoffs of this debate is the spotlight wildlife-related issues have
received in the past few days.

15. Q
a. layoffs
b. payoffs
c. kickbacks

A. Only a and b B. Only a and c C. Only b and c D. All a, b and c E. No


replacement required

In several parts of the country today, humans and protected species live in
____R____receptivity.

16. R
a. proximity
b. propinquity
c. vicinity

Use Code NIMISHA for maximum Discount

For any query call or whatsapp 9953592800


129

A. Only a and b B. Only a and c C. Only b and c D. All a, b and c E. No


replacement required

Crop _____S_____fosterage and attacks on livestock — even humans — have become


common.

17. S
a. depredation
b. plundering
c. rearing

A. Only a and b B. Only a and c C. Only b and c D. All a, b and c E. No


replacement required

They are yet to be used in a meaningful manner to restore a _____T____semblance of


harmony between wildlife and people living in the vicinity of the protected areas (PAs).

18. T
a. discrepancy
b. misalliance
c. keynote

A. Only a and b B. Only a and c C. Only b and c D. All a, b and c E. No


replacement required

Direction(19-23): In the passage below, there are TWO BLANKS in some of the lines.
There will be A PHRASAL VERB and A PHRASE in those two blanks. Choose the right
combination(s) for each line.

Use Code NIMISHA for maximum Discount

For any query call or whatsapp 9953592800


130

It may seem ironic that at a time when the world is more connected than ever before — 3.01
billion people keep in touch with one another via the internet — people are _______ by
____________________. The era when being solitary was a cherished goal — Greta Garbo
had famously quipped, “I want to be alone”— is now gone. Human contact is increasingly
becoming a luxury good and the market appears to have _________ that there can be a
______________________. The start-up, Goodfellows, supported by Ratan Tata, aims to
connect youngsters with senior citizens so that the latter can have company and the former
jobs. There are applications that allow people to ________ a ‘mother’, a ‘father’ or a
‘girlfriend’ for a day; there are even AI bots that try and ___________________________.
That so many are willing to pay for companionship, a fellowship that was assumed to be
freely available in the not-so-distant past, speaks of the profound scale and depth of the
modern phenomenon of loneliness. Estimates suggest that as much as 33per cent of the
world’s adults consider themselves to be lonely. The disintegration of the extended family,
________ with technological advancements, may have __________________________.
Even though everything — work, leisure, recreation, company— can be ordered at the click
of a mouse, the experience of organic bonding, the pleasure of in-person
encounters,remains irreplaceable to the human mind. The pandemic, with its periodic
lockdowns, has aggravated matters, as has the disappearance of greenery and spaces that
facilitated the collective. A study found that adults in neighbourhoods where at least 30 per
cent of nearby land was parks, reserves and woodlands had 26 per cent lower odds of
becoming lonely compared to their peers in areas with less than 10 percent of green space.
Ironically, loneliness is a shared phenomenon. This sense of sharedness must be
__________ in the battle to ______________________.

It may seem ironic that at a time when the world is more connected than ever before — 3.01
billion people keep in touch with one another via the internet — people are _______ by
____________________.

19. Which of the following pairs of a PHRASAL VERB and a PHRASE can most
appropriately fit in the blanks in the above statement respectively?

a. set upon, a sense of crushing loneliness


b. laced into, a feeling of devastating despondency
c. toned down, a sense of light-heartedness

A. Only a and b B. Only a and c C. Only b and c D. All a, b and c E. None of a, b


and c

Use Code NIMISHA for maximum Discount

For any query call or whatsapp 9953592800


131

Human contact is increasingly becoming a luxury good and the market appears to have
_________ that there can be a ______________________.

20. Which of the following pairs of a PHRASAL VERB and a PHRASE can most
appropriately fit in the blanks in the above statement respectively?

a. worked out, stiff premium on companionship


b. taken in, strict stipulation of loneliness
c. figured out, rigid price of fellowship

A. Only a and b B. Only a and c C. Only b and c D. All a, b and c E. None of a, b


and c

There are applications that allow people to ________ a ‘mother’, a ‘father’ or a ‘girlfriend’ for
a day; there are even AI bots that try and ___________________________.

21. Which of the following pairs of a PHRASAL VERB and a PHRASE can most
appropriately fit in the blanks in the above statement respectively?

a. take on, replicate the human experience of a friendly chat


b. sign on with, clone the essence of a congenial conversation
c. boot out, copy the perfunctory experience of a satisfying conversation

A. Only a and b B. Only a and c C. Only b and c D. All a, b and c E. None of a, b


and c

The disintegration of the extended family, _________ with technological advancements,


may have __________________________.

22. Which of the following pairs of a PHRASAL VERB and a PHRASE can most
appropriately fit in the blanks in the above statement respectively?

Use Code NIMISHA for maximum Discount

For any query call or whatsapp 9953592800


132

a. put together, aggravated the loneliness epidemic


b. linked up, exacerbated the despondency recrudescence
c. brought together, mitigated the solitude scourge

A. Only a and b B. Only a and c C. Only b and c D. All a, b and c E. None of a, b


and c

Ironically, loneliness is a shared phenomenon. This sense of sharedness must be


_________ in the battle to _____________________.

23. Which of the following pairs of a PHRASAL VERB and a PHRASE can most
appropriately fit in the blanks in the above statement respectively?

a. tapped into, make life a bit more crowded


b. resorted to, make life a bit more thronged
c. looked to, make life a bit more packed

A. Only a and b B. Only a and c C. Only b and c D. All a, b and c E. None of a, b


and c

Direction(24-28): In the passage below, there are THREE BLANKS in some of the
lines. There will be A PREPOSITION, A WORD and AN IDIOM/PHRASE. Choose the
right combination(s) for each line.

A report _______ in BMJ Global Health has ____________ that obesity and other conditions
related to weight are costing India around 1 per cent ____ its gross domestic product
annually. Overweight and obesity make up the most common lifestyle ailment in India and
affect 17 per cent of the population. The current expense of $35 billion a year can go up
____ nearly $850 billion by 2060 if steps are not taken to ________________ this problem
___________. The estimates include both medical and non-medical expenses, such as the
costs of seeking treatment for obesity-related health conditions and of travelling for the
same, the price of caregiving and so on. Significantly, indirect expenditures, such as
economic loss from premature mortality, missed days of work, and reduced productivity —

Use Code NIMISHA for maximum Discount

For any query call or whatsapp 9953592800


133

these have not been counted by previous studies — have a greater impact on the GDP (61-
88 per cent) than direct costs (12-39 per cent). While the Ayushman Bharat Yojana
_____________ in-hospital secondary and tertiary care, there are no clearly-outlined
mechanisms ______ paying for chronic outpatient care and medication, which __________
the bulk of the out-of-pocket expenses for obesity-related non-communicable diseases.

An important cause of this new epidemic is the aggressive marketing and the rising
consumption of ultra-processed foods — usually high in salt, sugar and bad fats. The food
industry continues to ____________ attempts to provide _________ information ______
unhealthy ingredients in spite of guidelines that require just that. Benign advisories will not
work: the need is for regulation with teeth. While consumption patterns need to change, this
cannot be the sole focus to ____________ obesity; workplaces may be losing money owing
____ obesity, but it is the modern work culture that promotes obesity.

A report ________ in BMJ Global Health has ___________ that obesity and other conditions
related to weight are costing India around 1 per cent ____ its gross domestic product
annually.

24. Which of the following combinations of A WORD, AN IDIOM and A PREPOSITION


can fit in the blanks in the above statement respectively?

a. published, let slip, of


b. put out, let drop, of
c. issued, brought out into the open, of

A. Only a and b B. Only a and c C. Only b and c D. All a, b and c E. None of a, b


and c

The current expense of $35 billion a year can go up ____ nearly $850 billion by 2060 if steps
are not taken to ______________ this problem _________.

25. Which of the following combinations of A PREPOSITION, AN IDIOM and A WORD


can fit in the blanks in the above statement respectively?

a. to, keep a tight rein on, immediately

Use Code NIMISHA for maximum Discount

For any query call or whatsapp 9953592800


134

b. to, keep a lid on, straight away


c. from, keep in check, promptly

A. Only a and b B. Only a and c C. Only b and c D. All a, b and c E. None of a, b


and c

While the Ayushman Bharat Yojana _____________ in-hospital secondary and tertiary care,
there are no clearly-outlined mechanisms ______ paying for chronic outpatient care and
medication, which __________ the bulk of the out-of-pocket expenses for obesity-related
non-communicable diseases.

26. Which of the following combinations of AN IDIOM, A PREPOSITION and A WORD


can fit in the blanks in the above statement respectively?

a. is made up of, for, comprise


b. is composed by, for, incorporate
c. is consisted of, for, contain

A. Only a B. Only a and c C. Only c D. Only a and b E. All a, b and c

The food industry continues to ____________ attempts to provide _________ information


______ unhealthy ingredients in spite of guidelines that require just that.

27. Which of the following combinations of AN IDIOM, A WORD and A PREPOSITION


can fit in the blanks in the above statement respectively?

a. set its face against, satisfactory, about


b. dig in its heels on, adequate, on
c. get to grips with, apposite, of

A. Only a and b B. Only a and c C. Only b and c D. All a, b and c E. None of a, b


and c

Use Code NIMISHA for maximum Discount

For any query call or whatsapp 9953592800


135

While consumption patterns need to change, this cannot be the sole focus to ____________
obesity; workplaces may be losing money owing ___ obesity, but it is the modern work
_________ that promotes obesity.

28. Which of the following combinations of AN IDIOM, A PREPOSITION and A WORD


can fit in the blanks in the above statement respectively?

a. get to grips with, to, culture


b. set its hand to, to, pattern
c. take measures about, to, attitude

A. Only a and b B. Only a and c C. Only b and c D. All a, b and c E. None of a, b


and c

Directions(29-33): In each of the questions below, a sentence is given with TWO


blanks in it that indicates TWO missing words. Choose the correct words that will
make the sentence grammatically and contextually complete and correct.

29. Since the start of the Russian war in Ukraine, Delhi has ________ “strategic autonomy”
to ________ a new foreign policy.

a. garrisoned, fashion
b. deployed, contrive
c. concentrated, frame

A. Only a and b B. Only a and c C. Only b and c D. All a, b and c E. None of a, b


and c

30. Of course, eventually, the best _________ lies in the normalisation of India-Pakistan
_______.

Use Code NIMISHA for maximum Discount

For any query call or whatsapp 9953592800


136

a. antidote, relations
b. corrective, alliance
c. nostrum, associations

A. Only a and b B. Only a and c C. Only b and c D. All a, b and c E. None of a, b


and c

31. Whether Pakistan _______ terrorism _______ has been a question of international
debate.

a. fought, in the least


b. confronted, at all
c. combated, the least bit

A. Only a and b B. Only a and c C. Only b and c D. All a, b and c E. None of a, b


and c

32. Zail Singh. Singh informed Mrs Gandhi that Balwant Singh had told him that it would be
difficult for the Akalis to resume negotiations with the government unless something
__________ was ________ by the latter.

a. concrete, conceded
b. tangible, granted
c. trivial, yielded

A. Only a and b B. Only a and c C. Only b and c D. All a, b and c E. None of a, b


and c

Use Code NIMISHA for maximum Discount

For any query call or whatsapp 9953592800


137

33. The government’s stand conveyed by Defence Minister R Venkataraman, to the Assam
leaders during their second informal meeting in two days, is bound to _______ a shadow on
the fifth round of formal tripartite _______.

a. cast, talks
b. emit, negotiations
c. radiate, dialogues

A. Only a B. Only a and c C. Only b D. All a, b and c E. None of a, b and c

34. Over two decades ago, _______ of ________ against politicians led to a CBI inquiry and
report against V P Singh’s government.

a. allegations, surveillance
b. accusations, monitoring
c. claims, superintendence

A. Only a B. Only a and c C. Only b D. All a, b and c E. None of a, b and c

Directions(35-39): In each of the questions below, a sentence is given with one blank
in it that indicates a missing PHRASAL VERB. Choose the correct PHRASAL VERB(s)
that will make the sentence grammatically and contextually complete and correct.

35. This technology keeps messages private by ________ anyone other than the recipient
from seeing them.

a. fending off
b. warding off
c. holding back

A. Only a and b B. Only a and c C. Only b and c D. All a, b and c E. None of a, b


and c

Use Code NIMISHA for maximum Discount

For any query call or whatsapp 9953592800


138

36. Wages __________ less than 5 per cent of the revenue but spending on promotion and
advertising is more than double of that.

a. add up to
b. account for
c. make up

A. Only a and b B. Only a and c C. Only b and c D. All a, b and c E. None of a, b


and c

37. There is a bittersweet joy in __________ an artist’s work — unpublished, unseen and
unheard — after he is no longer able to create.

a. working out
b. rooting out
c. tucking away

A. Only a and b B. Only a and c C. Only b and c D. All a, b and c E. None of a, b


and c

38. China has also unveiled measures to _________ the costs of betting against the yuan
using derivatives.

a. shoot up
b. bump up
c. think up

A. Only a and b B. Only a and c C. Only b and c D. All a, b and c E. None of a, b


and c

39. Until recently, the Reserve Bank of India had been actively _________ in the currency
markets.

Use Code NIMISHA for maximum Discount

For any query call or whatsapp 9953592800


139

a. cutting in
b. stepping in
c. chipping in

A. Only a and b B. Only a and c C. Only b and c D. All a, b and c E. None of a, b


and c

40. Ever since Queen Elizabeth II __________ in early September, the internet has been
replete with articles on the life and the work of Britain’s longest-reigning monarch.

a. passed away
b. passed on
c. passed off

A. Only a and b B. Only a and c C. Only b and c D. All a, b and c E. None of a, b


and c

Direction(41-43): In each question a sentence is given followed by a blank. You are


provided with three fragments a, b and c. You have to identify which
statement/statements can carry forward the given sentence in the most logical way so
as to make the sentence coherent and contextually correct.

41. The National Games in Gujarat are set to make a grand statement on the state’s
capability to host mega sports events. With the budget increased to a staggering Rs 2,000
crore for refurbishment of venues and other infrastructure, __________________________.
India has had a couple of memorable National Games prior to this one. The infrastructure
created at Balewadi in Pune and Gachibowli in Hyderabad has served the modest target of
remaining useful. Hopefully, Ahmedabad will do better.

a. the Games look ready to roll


b. a spectacle is absolutely assured

Use Code NIMISHA for maximum Discount

For any query call or whatsapp 9953592800


140

c. it is the responsibility of organisers now to not make returns on this investment count
in truly sporting terms

A. Only a and b B. Only a and c C. Only b and c D. All a, b and c E. None of a, b


and c

42. One National Games, even if successful, will not help in the absence of a sustained
pursuit and process of sporting excellence. But the Games could be a great opportunity to
expose players on the second rung to the pressure of performing in front of spectators, not
just for dragging half-hearted elite athletes through a week of easy strolls-in-the-park. No
one doubts India’s capacity to pull off the grand spectacle, but _______________________.

a. sport needs seriousness and commitment to host the Games year after year
b. the National Games can be an important nudge for elevating domestic sport
c. sport should be taken cheerfully because it has a significance beyond the field of play

A. Only a B. Only a and b C. Only b D. Only a and c E. All a, b and c

43. Women seem to be perpetually caught between liberation and oppression, mired in
vulnerabilities yet demonstrably resilient. Whatever our individual location in the fight against
patriarchy, women’s lives and bodies are often marked as repositories of tradition and are
used to serve roles that are both symbolic — representing nations, communities, and
families — and functional, that is transmitting these cultural and religious mores to future
generations. _______________________

a. Women’s behaviour and appearance are regulated to serve ends that aren’t
necessarily in their interest or that of society
b. It is little wonder that women often find themselves in ambiguous relationships with
these cultural roles.
c. Is the choice that we believe is ours, really ours?

A. Only a and b B. Only a and c C. Only b and c D. All a, b and c E. None of a, b


and c

Use Code NIMISHA for maximum Discount

For any query call or whatsapp 9953592800


141

Directions (44-49): In the following passage, some of the words have been left out,
each of which is indicated by a number. Find the suitable word from the options given
against each letter and fill up the blanks with appropriate words to make the
paragraph meaningful.

A quick dip stick among people residing in the housing societies of the Delhi-National
Capital Region shows that many households have still not re-employed part-time helps who
were initially prohibited from entering gated complexes during the ___________ (1)
lockdown. Such families continue to manage household chores on their own along with
office work. This boded well for consumer durable companies, which have been vocal about
___________ (2) sales of white goods and kitchen appliances. Once the lockdown was
lifted, retailers and companies reported a ___________ (3) in demand for washing
machines, dishwashers, large-screen TVs and kitchen appliances.Market research firm GfK,
which tracked consumption of white goods between June and September, found Indians
bought more front-loading washing machines, food processors, microwave ovens and larger
refrigerators, according to a Mint report. GfK said, with changed lifestyles, consumers
wanted products, solutions and services that make their life at home ___________ (4).
Consumer durables firm Voltas, too, said that in the ‘new normal’, it is likely that consumers
will invest in IoT-based technology to ease the ___________ (5) of household chores. “The
latest developments in the area of IoT have provided comfort and convenience to
consumers, leading to growing ___________ (6) for smart appliances," a Voltas
spokesperson says.

Q44.
(a) Aggravate
(b) Renounce
(c) Stringent
(d) Compel
(e) Disconcert

Q45.
(a) Joined
(b) Unadorned
(c) Befogged
(d) Sullied

Use Code NIMISHA for maximum Discount

For any query call or whatsapp 9953592800


142

(e) Improved

Q46.
(a) Jump
(b) Able
(c) Cheer
(d) Inept
(e) Permit

Q47.
(a) Yield
(b) Simple
(c) Effect
(d) Differ
(e) Desist

Q48.
(a) Cruelty
(b) Termination
(c) Trouble
(d) Burden
(e) Cloak

Q49.
(a) Traction
(b) Bewitch
(c) Ionized
(d) Repel
(e) Integrate

Directions (50-55): In the following passage, some of the words have been left out,
each of which is indicated by a number. Find the suitable word from the options given
against each letter and fill up the blanks with appropriate words to make the
paragraph meaningful.

Nilesh Shah, group president and managing director of Kotak Mahindra Asset Management,
has warned that 2021 will be a ___________ (7) year for equities, as there are doubts

Use Code NIMISHA for maximum Discount

For any query call or whatsapp 9953592800


143

around the world about the effectiveness of vaccines. Although he doesn’t see Indian
equities in a “bubble zone" yet, he has ____________ (8) investors against going
“overweight" on these. True, since many believe that covid jabs were rushed out, vaccine
________ (9) could delay the programme, thus delaying our return to normalcy. Also, if
corporate earnings fail to catch up with stock valuations that have reached treacherous
levels, it could trigger a ________________ (10) as investors look for an exit. The
______________ (11) rise of the Sensex, which has almost doubled from its March lows,
has cheap foreign capital looking for high returns in emerging markets to thank, mostly.
Worryingly, though, evidence of a topline corporate recovery has been all but absent so far,
and business expansion is the only way for profits to go up sustainably. Profits __________
(12) through cost compression has its limits. With the earnings growth trajectory still so
uncertain, investors, flush with easy money, or not, must exercise caution.

Q50.
(a) Harmonious
(b) Inglorious
(c) Acclaimed
(d) Turbulent
(e) Volatile

Q51.
(a) Bunched
(b) Cautioned
(c) Undaunted
(d) Piled
(e) Biased

Q52.
(a) Fallacy
(b) Conspiracy
(c) Hesitancy
(d) Lunacy
(e) Flippancy

Q53.
(a) Haughty
(b) Obstinate
(c) Tranquil

Use Code NIMISHA for maximum Discount

For any query call or whatsapp 9953592800


144

(d) Dilemma
(e) Stampede

Q54.
(a) Diaphanous
(b) Monotonous
(c) Vertiginous
(d) Mutinous
(e) Ominous

Q55.
(a) Eaten up
(b) Eked out
(c) Embarked on
(d) Engaged in
(e) Entrusted to

Direction (56-63): In the following passage, some of the words have been highlighted
in bold. First read the passage and try to understand what it is about. Then replace
the highlighted words with the correct option in order to make grammatical and
contextual sense. In case the highlighted word is correct as it is, mark 'No
Improvement' as the right answer.

With talks between the government and farmers on India’s three agricultural laws failing to
break the current deadlock, the Supreme Court’s Tuesday connotation (13) might yet be
the best hope to find a middle path agreeable to both sides. Unfortunately, this process has
also got constituted (14) right at the onset. The impartiality of a panel of four experts
appointed by the apex court has been questioned, with farmers pointing out that all of them
have taken a public appeal (15) in support of the laws in focus. As the outcome of the
proceedings can therefore be guessed, farm protesters argue, another fait accompli is being
dwelled (16) upon them. A glance at the quartet’s past views would indicate that such critics
do have a point. Agricultural economist Ashok Gulati, a prominent (17) member of the
committee, has been a vocal advocate of farm reforms in general and these laws in
particular. He has likened the move to India’s 1991 opening up. Pramod Joshi, former South
Asia director of the International Food Policy Research Institute, has opposed any legal
insisting (18) of minimum support prices (MSPs), a key demand of farmers, under our

Use Code NIMISHA for maximum Discount

For any query call or whatsapp 9953592800


145

public procurement system. He has also advocated contract farming, which farmers fear
may result in their subjugation (19) to powerful corporations. Similarly, Anil Ghanwat of
Shetkari Sanghatana, a pro-reform farmer union, has openly backed the laws, as has
Bhupinder Singh Mann, national president of the Bhartiya Kisan Union. To be fair, almost all
of them have offered variations (20) arguments, favouring amendments to assure farmers
easy access to judicial recourse or continuity of the state-procurement regime.

Q56.
(a) Irritation
(b) Implementation
(c) Intervention
(d) Sanctioned
(e) No Improvement

Q57.
(a) Boasted
(b) Advocated
(c) Alleged
(d) Vitiated
(e) No Improvement

Q58.
(a) Stance
(b) Attribute
(c) Exertion
(d) Disposition
(e) No Improvement

Q59.
(a) Persist
(b) Weigh
(c) Thrust
(d) Distinctive
(e) No Improvement

Use Code NIMISHA for maximum Discount

For any query call or whatsapp 9953592800


146

Q60.
(a) Keen
(b) Liberal
(c) Meritorious
(d) Formal
(e) No Improvement

Q61.
(a) Lowering
(b) Spending
(c) Backing
(d) Mining
(e) No Improvement

Q62.
(a) Alienation
(b) Jurisdiction
(c) Faction
(d) Association
(e) No Improvement

Q63.
(a) Harried
(b) Convinces
(c) Inspiring
(d) Nuanced
(e) No Improvement

Directions (64-70): In the following passage, some of the PHRASAL VERBS have been left
out, each of which is indicated by a letter. Find the suitable PHRASAL VERBS from the
options given against each letter and fill in the blanks with appropriate words to make the
paragraph meaningful.
Hundreds of people who ___ (A)___ seven villages in the southern Indian state of Tamil

Use Code NIMISHA for maximum Discount

For any query call or whatsapp 9953592800


147

Nadu say they are ____ (B)____ an infestation of yellow crazy ants.
These insects, they say, attack their livestock and affect crop yields, putting their livelihoods
____(C)____.
Yellow crazy ants are among the world's worst invasive species, according to the
International Union for Conservation of Nature.
They don't bite or sting but spray formic acid, which can cause reactions.
These ants - their scientific name is Anoplolepis gracilipes - are usually ____ (D)____
tropical and sub tropical regions. They ____ (E)___ an erratic, uncoordinated way, with their
movement becoming more frantic when disturbed.
Experts say these ants proliferate quickly and can "do a large amount of damage to native
wildlife". Many parts of Australia have reported infestations of these insects.
Dr. Pronoy Baidya, an entomologist who has done _____(F)_____ yellow crazy ants, says
they are an "opportunistic species".
"They don't have any diet preferences. They eat anything and everything," he says, adding
that they also ___(G)____ other ant species, bees, and wasps.

Hundreds of people who ___ (A)___ seven villages in the southern Indian state of Tamil
Nadu say they are ____ (B)____ an infestation of yellow crazy ants.

64. Which of the following words can appropriately fit in position A?


1. live in
2. dwell in
3. rely on

A. Only 1
B. Both 1 and 2
C. Both 1 and 3
D. Only 3
E. All 1, 2 and 3

Hundreds of people who ___ (A)___ seven villages in the southern Indian state of Tamil
Nadu say they are ____ (B)____ an infestation of yellow crazy ants.

65. Which of the following words can appropriately fit in position B?


1. suffering from
2. emerging from
3. reeling from

Use Code NIMISHA for maximum Discount

For any query call or whatsapp 9953592800


148

A. Only 1
B. Both 1 and 2
C. Both 1 and 3
D. Only 3
E. All 1, 2 and 3

These insects, they say, attack their livestock and affect crop yields, putting their livelihoods
____(C)____.

66. Which of the following words can appropriately fit in position C?


1. in danger
2. in peril
3. on edge

A. Only 1
B. Both 1 and 2
C. Both 1 and 3
D. Only 3
E. All 1, 2 and 3

They don't bite or sting but spray formic acid, which can cause reactions.
These ants - their scientific name is Anoplolepis gracilipes - are usually ____ (D)____
tropical and sub tropical regions.

67. Which of the following words can appropriately fit in position D?


1. unearthed in
2. introduced in
3. found in

A. Only 1
B. Both 1 and 2
C. Both 1 and 3
D. Only 3
Use Code NIMISHA for maximum Discount

For any query call or whatsapp 9953592800


149

E. All 1, 2 and 3

They ____ (E)___ an erratic, uncoordinated way, with their movement becoming more
frantic when disturbed.

68. Which of the following words can appropriately fit in position E?


1. move on
2. move in
3. move towards

A. Only 1
B. Only 2
C. Both 1 and 3
D. Only 3
E. All 1, 2 and 3

Experts say these ants proliferate quickly and can "do a large amount of damage to native
wildlife". Many parts of Australia have reported infestations of these insects.
Dr. Pronoy Baidya, an entomologist who has done _____(F)_____ yellow crazy ants, says
they are an "opportunistic species".

69. Which of the following words can appropriately fit in position F?


1. research in
2. research by
3. research on

A. Only 1
B. Both 1 and 2
C. Both 1 and 3
D. Only 3
E. All 1, 2 and 3

"They don't have any diet preferences. They eat anything and everything," he says, adding
that they also ___(G)____ other ant species, bees, and wasps.
Use Code NIMISHA for maximum Discount

For any query call or whatsapp 9953592800


150

70. Which of the following words can appropriately fit in position G?


1. kill off
2. hunt on
3. prey on

A. Only 1
B. Both 1 and 2
C. Both 1 and 3
D. Only 3
E. All 1, 2 and 3

Directions(71-79): In the following passage, some of the words have been left out,
each of which is indicated by a letter. Find the suitable word/words from the options
given against each letter and fill up the blanks with appropriate words to make the
paragraph meaningful.

Multilateralism is in retreat. The corresponding revival of the spirit of unilateralism around the
world would support such ___(M)___. But this ___(N)___ of multilateralism cannot be
pinned on the ___(O)___ of an ascendant authoritarian politics. The ___(P)___ may lie
within the world’s multilateral framework. The lack of ___(Q)___ leadership is one of the
fundamental causes that explain multilateralism’s failure to respond to such critical
challenges as the pandemic that has brought nation-states and their economies to their
___(R)___. This ___(S)___ in the leadership criterion can be attributed to a ___(T)___
lacuna. The element of competitive politics within the international fraternity seems to have
played a fundamental role in the ___(U)___ of the foundations of multilateralism.

71. M
a. conjecture
b. longing
c. supposition
d. exaction

A. Both a and b
B. All b, c and d
C. Only a
Use Code NIMISHA for maximum Discount

For any query call or whatsapp 9953592800


151

D. Both a and c
E. All a, b, c and d

72. N
a. furtherance
b. withdrawal
c. repulsion
d. aversion

A. Only a
B. Both c and d
C. All a, c and d
D. Only b
E. Both b and c

73. O
a. face
b. array
c. lapel
d. quip

A. Only c
B. Both a and d
C. Both b and c
D. All a, b, c and d
E. Only d

74. P
a. rot
b. riddance
c. canker
d. drollness

A. Only d
B. Both a and d
C. Both a and c
D. Only c
E. Both b and c

Use Code NIMISHA for maximum Discount

For any query call or whatsapp 9953592800


152

75. Q
a. impeccable
b. seamless
c. unsurpassed
d. unblemished

A. Only a
B. Both a and c
C. All a, b and d
D. Both a and b
E. All a, b, c and d

76. R
a. stifles
b. kicks
c. knees
d. swings

A. Both a and c
B. Only c
C. Only b
D. Both b and d
E. Only a

77. S
a. want
b. paucity
c. exiguity
d. plethora

A. All a, b and c
B. All a, c and d
C. Both b and c
D. Only b
E. All a, b, c and d

78. T
a. compelling
b. shaky

Use Code NIMISHA for maximum Discount

For any query call or whatsapp 9953592800


153

c. unsound
d. telling

A. Only a
B. Both a and d
C. Both b and d
D. Only c
E. All a, c and d

79. U
a. accrual
b. uptick
c. accretion
d. attrition

A. Only a
B. Both b and d
C. Both a and c
D. Only d
E. All b, c and d

ANSWER

1.(Answer - A. Only a

out of the woods - No longer in danger or dealing with a particular difficulty, though not
entirely resolved.

out of the window - Forgotten or disregarded; lost or wasted.

out of the red - No longer having a debt owed to one or more other people, corporations,
banks, etc.)

2.(Answer - A. Only a and b

a and b - to explain.

Use Code NIMISHA for maximum Discount

For any query call or whatsapp 9953592800


154

obscure - keep from being seen; conceal.)

3.(Answer - B. Only a and c

miscarry / flunk - to fail or be unsuccessful.

loft - kick, hit, or throw (a ball or missile) high up.)

4.(Answer - D. All a, b and c

All mean - the quality of being trusted and believed in; reputation.)

5.(Answer - B. Only a and c

at stake / at risk / in jeopardy - In danger or at risk; imperiled.

Hence, 'at jeopardy' is INCORRECT.)

6.(Answer - D. All a, b and c

All mean - (of war, fighting, or similarly undesirable things) start suddenly.)

7.(Answer - D. All a, b and c

All mean - having all constituent parts linked or connected.)

8.(Answer - C. Only b and c

b and c - rational or logical.

ludicrous - irrational.)

9.(Answer - D. All a, b and c

All mean - continue with great force or intensity.)

Use Code NIMISHA for maximum Discount

For any query call or whatsapp 9953592800


155

10.(Answer - E. All a, b and c

All mean - to a moderately high degree.)

11.(Answer - D. All a, b and c

All mean - travel unsystematically over, through, or about (a place).

dissipate - disappear or cause to disappear.)

12.(Answer - D. All a, b and c

All mean - (especially of something undesirable) not controlled or restrained.

unabridged - (of a text) not cut or shortened; complete.)

13.(Answer - C. Only b and c

cub / offspring - the young of a fox, bear, lion, or other carnivorous mammal.

neophyte / greenhorn - a person who is new to a subject or activity.)

14.(Answer - D. All a, b and c

All mean - pessimistic or negative.

reassuring - optimistic.)

15.(Answer - E. No replacement required

spinoff - a by-product or incidental result of a larger project.

layoff - a temporary or permanent discharge of a worker or workers.

payoff - a payment made to someone, especially as a bribe or on leaving a job.

kickback - a sudden forceful recoil.)

Use Code NIMISHA for maximum Discount

For any query call or whatsapp 9953592800


156

16.(Answer - D. All a, b and c

All mean - nearness in space, time, or relationship.

receptivity - willingness to consider or accept new suggestions and ideas.)

17.(Answer - A. Only a and b

a and b - an act of attacking or looting.

rearing - cultivation.

fosterage - the action of bringing up a child that is not one's own by birth.)

18.(Answer - E. No replacement required

semblance - a similarity to something, or the appearance of being or having something.

a and b - mismatch.

keynote - a prevailing tone or central theme.)

19.(Answer - A. Only a and b

set upon / laced into - attacked.

despondency - low spirits from loss of hope or courage; dejection.

toned down - eased.


light-heartedness - carefreeness.)

20.(Answer - B. Only a and c

All phrasal verbs mean - to realize.

'loneliness' is contextually incorrect.

Use Code NIMISHA for maximum Discount

For any query call or whatsapp 9953592800


157

fellowship - companionship.)

21.(Answer - A. Only a and b

sign on with / take on - hire or rent.

boot out - to sack or lay off.


perfunctory - carried out without real interest, feeling, or effort.)

22.(Answer - A. Only a and b

put together with / linked up with / brought together with - coupled with.

aggravate / exacerbate - to make sth worse.


'mitigate' is the antonym.

despondency - hopelessness.
recrudescence / scourge - the recurrence of an undesirable condition.)

23.(Answer - D. All a, b and c

tap into / resort to / look to - utilize

thronged / packed - crowded.)

24.(Answer - D. All a, b and c

let slip / let drop / bring out into the open - to reveal.

put out - to publish.)

25.(Answer - A. Only a and b

All the idioms mean - to curb.

straight away / promptly - immediately.)

Use Code NIMISHA for maximum Discount

For any query call or whatsapp 9953592800


158

26.(Answer - A. Only a

be made up of / be composed of / consist of - to cover or include or to comprise or to


incorporate.

be consisted of is incorrect.)

27.(Answer - A. Only a and b

These two idioms mean - to resist.

get to grips with - to begin to deal with or understand.

information on / about / of - All are correct.

apposite / adequate - satisfactory.)

28.(Answer - D. All a, b and c

All the idioms mean - to tackle.

owing to - because of.)

29.(Answer - A. Only a and b

deploy / garrison - move (troops or equipment) into position for military action.

concentrate - focus all one's attention on a particular object or activity.


This word is inappropriate.

fashion / contrive / frame (v) - to build or construct sth.)

30.(Answer - D. All a, b and c

antidote / corrective / nostrum - something that counteracts an unpleasant feeling or


situation.)

31.(Answer - D. All a, b and c

Use Code NIMISHA for maximum Discount

For any query call or whatsapp 9953592800


159

at all / in the least / the least bit (adv) - in the slightest degree.)

32.(Answer - A. Only a and b

concede / yield - to grant.

concrete / tangible - strongly built or practical.

trivial - insignificant.)

33.(Answer - A. Only a

cast a shadow (phrase) - cause (something) to be less good or enjoyable.)

34.(Answer - D. All a, b and c)

35.(Answer - A. Only a and b

a and b - to prevent.

hold back - hesitate to act or speak.)

36.(Answer - D. All a, b and c

All mean - to be (a part) of a whole or to constitute.)

37.(Answer - A. Only a and b

a and b - to discover.

tuck away - to hide.)

38.(Answer - A. Only a and b

a and b - to increase.

think up - to invent a story or plan.)

Use Code NIMISHA for maximum Discount

For any query call or whatsapp 9953592800


160

39.(Answer - D. All a, b and c

All mean - to intervene.)

40. (Answer - A. Only a and b

a and b - to die.

pass off - falsely represent a person or thing as being someone or something else.
("the drink was packaged in champagne bottles and was being passed off as the real stuff")

41.(Answer - A. Only a and b

c - Incorrect.
The phrase - to not make returns - is INCORRECT.

spectacle - a visually striking performance or display.)

42.(Answer - A. Only a

b - Incorrect. It's irrelevant.

c - Incorrect. The fact that sport should be taken cheerfully is not convincing and
does not go with the theme well.)

43.(Answer - A. Only a and b

a and b are contextually correct.

c - Incorrect. It has no link.)

44.Ans. (c)
Sol. ‘’stringent” is the appropriate word to fill up the blanks and make the paragraph
meaningful. Hence, option (c) is the right answer choice.

45.Ans. (e)

Use Code NIMISHA for maximum Discount

For any query call or whatsapp 9953592800


161

Sol. “Improved” is the appropriate word to fill up the blanks and make the paragraph
meaningful. Hence, option (e) is the right answer choice.

46.Ans. (a)
Sol. “Jump” is the appropriate word to fill up the blanks and make the paragraph
meaningful. Hence, option (a) is the right answer choice.

47.Ans. (b)
Sol. “simple” is the appropriate word to fill up the blanks and make the paragraph
meaningful. Hence, option (b) is the right answer choice.

48.Ans. (d)
Sol. “Burden” is the appropriate word to fill up the blanks and make the paragraph
meaningful. Hence, option (d) is the right answer choice.

49. Ans. (a)


Sol. “traction” is the appropriate word to fill up the blanks and make the paragraph
meaningful. Hence, option (a) is the right answer choice.

50. Ans. (e)


Sol. “Volatile” is the appropriate word to fill up the blanks and make the paragraph
meaningful. Hence, option (e) is the right answer choice.
Volatile: liable to change rapidly and unpredictably, especially for the worse.

51.Ans. (b)
Sol. “Cautioned” is the appropriate word to fill up the blanks and make the paragraph
meaningful. Hence, option (b) is the right answer choice.
Cautioned: say something as a warning.

52.Ans. (c)
Sol. “Hesitancy” is the appropriate word to fill up the blanks and make the paragraph
meaningful. Hence, option (c) is the right answer choice.
Hesitancy: the quality or state of being hesitant.

Use Code NIMISHA for maximum Discount

For any query call or whatsapp 9953592800


162

53.Ans. (e)
Sol. “stampede” is the appropriate word to fill up the blanks and make the paragraph
meaningful. Hence, option (e) is the right answer choice.
Stampede: To rush in(to some place) all at once and in, or as if in, a wild panic.
-To rush due to some particular catalyst or reason.

54.Ans. (c)
Sol. “Vertiginous” is the appropriate word to fill up the blanks and make the paragraph
meaningful. Hence, option (c) is the right answer choice.
Vertiginous: extremely high or steep.

55. Ans. (b)


Sol. “Eked out” is the appropriate word to fill up the blanks and make the paragraph
meaningful. Hence, option (b) is the right answer choice.
Eked out: to make up for the deficiencies of : supplement
Eat up: If something eats up your time or money, you spend a lot of time or money on it.
Embark on: to begin something, usually something that will be challenging and time-
consuming
Engage in: to become involved in something related to competition or conflict, such as a
debate, a battle, or a dispute.
Entrust to: to give somebody responsibility for something of importance or value

56. Ans. (c)


Sol. “Connotation’ is wrong. “Intervention” should be used instead of “connotation” in
order to make grammatical and contextual sense. Hence, option (c) is the right answer
choice.
Connotation: an idea or feeling which a word invokes for a person in addition to its literal or
primary meaning.
Intervention: the action of becoming intentionally involved in a difficult situation, in order to
improve it or prevent it from getting worse

57. Ans. (d)


Sol. “Vitiated” should be used instead of “Constituted” in order to make grammatical and
contextual sense. Hence, option (d) is the right answer choice.
Constituted: be (a part) of a whole.
Vitiated: spoiled or impaired the quality or efficiency of.
- to make faulty or defective

58. Ans. (a)

Use Code NIMISHA for maximum Discount

For any query call or whatsapp 9953592800


163

Sol. “Stance” should be used instead of “Appeal” in order to make grammatical and
contextual sense. Hence, option (a) is the right answer choice.
Stance: the attitude of a person or organization towards something; a standpoint.
Appeal: make a serious, urgent, or heartfelt request.

59. Ans. (c)


Sol. “Thrust” should be used instead of “Dwelled” in order to make grammatical and
contextual sense. Hence, option (c) is the right answer choice.
Dwell: think moodily or anxiously about something
Thrust: push suddenly or violently in a specified direction.

60. Ans. (e)


Sol. The given word is correct and needs no improvement. Hence, option (e) is the right
answer choice.

61. Ans. (c)


Sol. “Backing” should be used instead of “Insisting” in order to make grammatical and
contextual sense. Hence, option (c) is the right answer choice.

62.Ans. (e)
Sol. The given sentence is correct and needs no improvement. Hence, option (e) is the right
answer choice.
Subjugation: the action of bringing someone or something under domination or control.

63. Ans. (d)


Sol. “Nuanced” should be used instead of “Variations” in order to make grammatical and
contextual sense. Hence, option (c) is the right answer choice.
Nuanced: a subtle difference in or shade of meaning, expression, or sound.
Hence, option (d) is the right answer choice.

64.B

Solution : live in (verb): To dwell or reside in something or some place.

65.C

Solution : reeling from: If you reel, or your mind or brain reels, you feel very confused or
shocked and unable to act.

Use Code NIMISHA for maximum Discount

For any query call or whatsapp 9953592800


164

"Suffering from": If you say someone suffers, it means that they are enduring physical or
emotional pain

Both choices 1 and 3 can fill the blank contextually.

66.E

Solution : in danger: Liable to incur harm or injury; unsafe; in peril.

On edge: "feeling nervous" or "not calm and relaxed."

67.D

Solution : Only “found in” can contextually fit in the blank.

Unearthed means finding something in the ground by digging.

Introduced means bringing (a plant, animal, or disease) to a place for the first time.

68.B

Solution : Only “move in” contextually fits in the blank.

Move on: start doing something new or make progress. Move towards suggests direction
of movement, not manner, so it is out-of-context

69.D

Solution : 4 is the correct answer.

One does research in a particular field of study.

He does research in cardiology.

One does research on a particular subject.

He does research on heart disease.

70.E

Solution : Prey on: to hunt and kill (something) for food

"kill off" emphasizes the fact that a large number, if not all, of the animals are being killed

71.(Answer - D. Both a and c

Use Code NIMISHA for maximum Discount

For any query call or whatsapp 9953592800


165

supposition / conjecture - an opinion or conclusion formed on the basis of incomplete


information.
(In this context 'conjecture' is referred to the fact that 'Multilateralism is in retreat'.
is in retreat - is being withdrawn.)

longing - desire.
exaction - the action of demanding and obtaining something from someone, especially a
payment.)

72.(Answer - D. Only b

withdrawal - the action of withdrawing something.


(In this context the withdrawal is referred to the fact that ''Multilateralism is in retreat'.)

furtherance - advancement.
repulsion - a feeling of intense distaste or disgust.
aversion - a strong dislike or disinclination.)

73.(Answer - A. Only c

lapel - the part on each side of a coat or jacket immediately below the collar which is folded
back on either side of the front opening.
Here it has been used figuratively.
'..this withdrawal of multilateralism cannot be pinned on the lapel of an ascendant
authoritarian politics'
This means - an ascendant(rising in power) authoritarian politics is not responsible for
the withdrawal of multilateralism.

'face' can't be chosen due to the verb used here is 'pinned on'.

array - an orderly arrangement.


quip - a witty remark or a joke.)

74.(Answer - C. Both a and c

rot / canker - a source of corruption.

riddance - the action of getting rid of a troublesome or unwanted person or thing.

Use Code NIMISHA for maximum Discount

For any query call or whatsapp 9953592800


166

drollness - The quality of being laughable or comical.)

75.(Answer - E. All a, b, c and d

impeccable / seamless / unblemished - faultless.


unsurpassed - excellent.)

76.(Answer - B. Only c

bring sb/sth to their knees - to destroy or defeat someone or something.

stifle - to withhold from circulation or expression.)

77.(Answer - A. All a, b and c

want / paucity - the presence of something in only small or insufficient quantities or


amounts.
exguity - the quality of being meager or insufficient.

plethora - a large or excessive amount of something.)

78.(Answer - B. Both a and d

compelling / telling - potent or strong.


telling / compelling lacuna - strong gap.

shaky - weak or inconsistent.


unsound - not in good condition.)

79.(Answer - D. Only d

attrition - the process of reducing something's strength or effectiveness.

accrual / accretion - the accumulation or increase of something over time.


uptick - a small increase or slight upward trend.)

Use Code NIMISHA for maximum Discount

For any query call or whatsapp 9953592800


167

CHAPTER 5 - WORD REARRANGEMENT

Directions(1-50): In each of the questions given below, four words are given in bold.
These four words may or may not be in their correct position. The sentence is then
followed by options with the correct combination of words that should replace each
other in order to make the sentence grammatically and contextually correct. Find the
correct combination of words that replace each other. If the sentence is correct as it
is, select ‘E’ as your option.

1. With the (m)minds from the Covid pandemic, of burning (n)cadavers and floating
(o)pyres, still fresh in our (p)images, the current debate on the magnitude of mortality
during 2020 and 2021 looks surreal and unsympathetic.

A. Only o-p
B. Both m-p and n-o
C. Only m-p
D. Both m-n and o-p
E. No replacement required

Use Code NIMISHA for maximum Discount

For any query call or whatsapp 9953592800


168

2. By deaths per thousand (m)position, India, however, does not figure among the top 100
countries, though the (n)fatality (o)infection rate of 1.2 per cent places it in seventh
(p)population globally.

A. Only m-n
B. Both m-n and o-p
C. Both m-p and n-o
D. Only m-p
E. No replacement required

3. While the (m)freshness of this religiosity heals my tormented (n)horror, I cannot forget
the (o)soul of yet another kind of (p)vehemency sanctified in the name of educating the
child.

A. Only n-o
B. Both m-p and n-o
C. Only m-p
D. Both m-n and o-p
E. No replacement required

4. We (m)encourage our children to study physics or botany, but do we really (n)compel


them to look at a tree with wonder, (o)overcome with the amazing sunset, and (p)merge
the duality of the “observer” versus the “observed”?

A. Only n-o
B. Both m-p and n-o
C. Only m-p
D. Both m-n and o-p
E. No replacement required

5. Education is not just to pass examinations, take a degree, a job, get married and settle
down, but also to be able to listen to the birds, to see the (m)shape, to see the extraordinary

Use Code NIMISHA for maximum Discount

For any query call or whatsapp 9953592800


169

(n)touch of a tree, and the (o)ether of the hills, and to feel with them, to be really, directly in
(p)beauty with them.

A. Only m-n
B. Both m-n and o-p
C. Both m-o and n-p
D. Only m-p
E. No replacement required

6. The hazy outlines of a life together were beginning to take (m)emotions, but, as an
inconvertible introvert, deficient forever in articulating my (n)shape in (o)relationship, I
turned to music to take the (p)words ashore.

A. Only n-o
B. Both m-p and n-o
C. Only m-p
D. Both m-n and o-p
E. No replacement required

7. Looking back, music had always been the (m)time of my life, and the changing musical
interfaces, a (n)passage of the (o)record of (p)soundtrack.

A. Only m-n
B. Both m-n and o-p
C. Both m-p and n-o
D. Only m-p
E. No replacement required

8. In a (m)sedition of petitions challenging the law of (n)batch, contained in (o)slew 124A


of the Indian Penal Code (IPC), the Supreme Court on Wednesday issued a (p)section of
interim directions.

A. Only m-n
B. Both m-n and o-p

Use Code NIMISHA for maximum Discount

For any query call or whatsapp 9953592800


170

C. Both m-p and n-o


D. Only m-p
E. No replacement required

9. The Court granted the central government time till May 10 to file its (m)challenge, failing
which it intended to decide the (n)question of whether there was a (o)requirement to refer
the (p)response to a seven-judge bench.

A. Only n-o
B. Both m-p and n-o
C. Only m-p
D. Both m-n and o-p
E. No replacement required

10. In fact, Senior Advocate Gopal Sankaranaryanan submitted proposed


(m)consequential directions of a (n)downright suspension of the law which inter alia
included (o)explicit stay of (p)pending proceedings and bar on registration of new cases.

A. Only n-o
B. Both m-p and n-o
C. Only m-p
D. Both m-n and o-p
E. No replacement required

11. After some (m)suggestions, the Court refused to confine itself to the (n)deliberation
proposed by the government and passed (o)investigations hoping and expecting the state
and central governments to restrain themselves from registering new FIRs, continuing
pending (p)directions, or arresting people under section 124A IPC.

A. Only n-o
B. Both m-p and n-o
C. Only m-p
D. Both m-n and o-p
E. No replacement required

Use Code NIMISHA for maximum Discount

For any query call or whatsapp 9953592800


171

12. This order appears to be a small win but (m)desired a lot to be (n)left; also, its
implementation at the ground level remains to be (o)taken and lessons should have been
(p)seen from the fact that police across the country continued filing FIRs under section 66A
of the Information Technology Act.

A. Only m-n
B. Both m-n and o-p
C. Both m-p and n-o
D. Only m-p
E. No replacement required

13. British (m)resumption and helicopters have attacked Port Stanley and Port Darwin and
fighter (n)lifeboats sank a fishing vessel and machine-gunned (o)bombers in the (p)ships
of the Anglo-Argentine conflict over the Falkland Islands.

A. Only n-o
B. Both m-p and n-o
C. Only m-p
D. Both m-n and o-p
E. No replacement required

14. A team of sleuths from the communal (m)wing of the special (n)branch of the Delhi
police and the Union Home Ministry’s Intelligence Bureau has been asked to unearth the
(o)sacrilege behind the cases of (p)conspiracy in some temples of the capital.

A. Only o-p
B. Both m-p and n-o
C. Only m-p
D. Both m-o and n-p
E. No replacement required

Use Code NIMISHA for maximum Discount

For any query call or whatsapp 9953592800


172

15. While police (m)pickets have been posted near temple and other sensitive (n)spots, a
close watch is being kept on the (o)activists of Dal Khalsa; there are intelligence reports of
(p)miscreants doing something untoward after the incidents of sacrilege in Punjab and
Haryana.

A. Only o-p
B. Both m-p and n-o
C. Only m-p
D. Both m-o and n-p
E. No replacement required

16. Inside closed doors, artists now admit to being (m)offensive and (n)fearful; in public
spaces, meanwhile, they are (o)wary of viewers who might deem their work (p)over-
cautious.

A. Only o-p
B. Both m-n and o-p
C. Only m-p
D. Both m-o and n-p
E. No replacement required

17. Public (m)artist does lead to conversations on the subject and an (n)endeavor of the
need to protect artistic liberties, but the vandals often succeed in fulfilling their immediate
(o)assertion of bringing down the work and the (p)outrage is left alone in the end.

A. Only o-p
B. Both m-p and n-o
C. Only m-p
D. Both m-o and n-p
E. No replacement required

Use Code NIMISHA for maximum Discount

For any query call or whatsapp 9953592800


173

18. Artists (m)encourage to (n)aspire conversations and (o)provoke to find unorthodox


ways to question and (p)seek.

A. Only o-p
B. Both m-p and n-o
C. Only m-p
D. Both m-n and o-p
E. No replacement required

19. By placing (m)policies such as houses and LPGs in the (n)deficits of women, it is
challenging the unequal status quo; it is doing so not only through (o)assets but by bridging
gendered data (p)hands.

A. Only n-o
B. Both m-p and n-o
C. Only m-p
D. Both m-o and n-p
E. No replacement required

20. As a routine (m)barometer of crucial information on nutrition, fertility, family


(n)planning, reproductive, maternal and child health and mortality, the National Family
Health Survey (NFHS) is a (o)source of India’s (p)performance in securing equitable
health outcomes, especially for women.

A. Only n-o
B. Both m-p and n-o
C. Only m-o
D. Both m-o and n-p
E. No replacement required

21. The Ayushman Bharat (m)healthcare has considerably expanded the (n)thinking of
(o)programme in the country; but the pandemic should occasion more serious (p)scope on
addressing the shortfalls.

Use Code NIMISHA for maximum Discount

For any query call or whatsapp 9953592800


174

A. Only n-o
B. Both m-p and n-o
C. Only m-p
D. Both m-o and n-p
E. No replacement required

22. The office of the Registrar General does not give a disease-wise (m)set of deaths, but
read along with other studies, the data (n)break-up released on Tuesday is a useful
(o)pressure of the (p)indicator on the medical system during the pandemic.

A. Only o-p
B. Both m-p and n-o
C. Only m-p
D. Both m-n and o-p
E. No replacement required

23. The (m)jump of people dying for (n)attention of medical (o)want increased from 34.5
per cent of all recorded deaths in 2019 to 45 per cent in 2020 — the largest single-year
(p)proportion.

A. Only o-p
B. Both m-p and n-o
C. Only m-p
D. Both m-n and o-p
E. No replacement required

24. Sant Singh Sekhon was the principal and I was one of the members of the welcome
committee organising a festival of poets. Batalvi was the (m)podium of all eyes: the venue
was crowded just because of him; the (n)stage started without Batalvi and ther poets came
on the (o)festival but the audience kept looking away from the (p)cynosure and waiting
intently for the poet of the day.

A. Only o-p
B. Both m-p and n-o
C. Only m-p

Use Code NIMISHA for maximum Discount

For any query call or whatsapp 9953592800


175

D. Both m-n and o-p


E. No replacement required

25. Tolerance is perhaps one of the most profound lessons that the people of Punjab have
imbibed from the (m)decade lost to (n)militancy; they have not let the cancer of hate eat
into the vitals of society, finding common (o)ground in their shared (p)heritage instead.

A. Only n-o
B. Both m-p and n-o
C. Only m-o
D. Both m-o and n-p
E. No replacement required

26. We are a postcolonial nation-state with a long (m)invasion of colonial (n)domination


that has caused irreparable (o)damage to people at large; espite this, if people cheer for the
Russian (p)history, it reflects perhaps the latent imperial ambition.

A. Only n-o
B. Both m-p and n-o
C. Only m-p
D. Both m-o and n-p
E. No replacement required

27. With the horrific war in Ukraine showing no signs of ending, Prime Minister Narendra
Modi’s visit to Berlin, Copenhagen, and Paris this week could give us a (m)future of India’s
post-Russian strategic (n)glimpse in Europe; as Russia, isolated by unprecedented
Western sanctions, deepens its (o)calculus with China, Europe has begun to loom larger
than ever before in India’s strategic (p)coalition.

A. Only o-p
B. Both m-p and n-o
C. Only m-p

Use Code NIMISHA for maximum Discount

For any query call or whatsapp 9953592800


176

D. Both m-n and o-p


E. No replacement required

28. Power (m)ecosystem is highly dependent on coal — about 78 per cent of it comes from
this fossil fuel — and, (n)energy is almost entirely dependent on oil; the Indian
(o)transportation (p)generation is, thus, highly carbon-intensive.

A. Only o-p
B. Both m-p and n-o
C. Only m-p
D. Both m-n and o-p
E. No replacement required

29. The most natural (m)intervention of government (n)option for reducing emissions is by
fixing limits of emissions through (o)consideration, taking into (p)regulation the Nationally
Determined Contribution targets set by the country under the Paris Agreement.

A. Only n-o
B. Both m-p and n-o
C. Only m-o
D. Both m-n and o-p
E. No replacement required

30. From addressing specific (m)pressures to preventative (n)benefits and overall mental
well-being, yoga is now being acknowledged as a practice that helps (o)individuals cope
with the (p)ailments of the 21st century.

A. Only n-o
B. Both m-p and n-o
C. Only m-p
D. Both m-n and o-p
E. No replacement required

Use Code NIMISHA for maximum Discount

For any query call or whatsapp 9953592800


177

31. This past week, as many of us (m)booked on Amazon or perhaps (n)shopped flight
tickets for the summer vacations, Elon Musk (o)sold his interest in purchasing the social
media platform Twitter for $44 billion and Musk has reportedly (p)evinced roughly $4 billion
worth of Tesla stock over the past two days to help with financing the acquisition.

A. Only n-o
B. Both m-p and n-o
C. Only m-o
D. Both m-n and o-p
E. No replacement required

32. A poison pill in commercial (m)target is a defense (n)takeover used by a (o)law firm to
prevent or discourage a potential hostile (p)strategy by an acquiring company.

A. Only m-n
B. Both m-p and n-o
C. Only o-p
D. Both m-o and n-p
E. No replacement required

33. According to NASA, the crew will conduct a science (m)microgravity in (n)expedition
aboard the space station; on April 18, a SpaceX Falcon rocket was used to successfully
launch a US spy (o)base force from the company’s Vandenberg Space Force (p)satellite in
California.

A. Both m-p and n-o


B. Only m-p
C. Both m-n and o-p
D. Only n-o
E. No replacement required

Use Code NIMISHA for maximum Discount

For any query call or whatsapp 9953592800


178

34. The (m)conflict of the (n)self-sacrifice for greater good (in a tragic sense) appealed to
the early Israelis — they came to their homeland, which was in (o)renunciation and that
demanded plenty of (p)self from them.

A. Both m-o and n-p


B. Only m-p
C. Both m-n and o-p
D. Only n-o
E. No replacement required

35. European Jews were the (m)pioneers who established the (n)power, enjoyed more
(o)state and shaped much of the Israeli (p)identity, one which was alien to most non-
European, including Indian, Jews.

A. Both m-o and n-p


B. Only m-p
C. Both m-n and o-p
D. Only n-o
E. No replacement required

36. India’s diplomatic (m)distance with Israeli and vocal (n)solidarity for the Palestinian
cause did not sour Israeli (o)affinity for Indian (p)culture in the 1950s or later.

A. Both m-o and n-p


B. Only m-p
C. Both m-n and o-p
D. Only n-o
E. No replacement required

37. Propriety suggests a tried and tested (m)hypothesis for making conclusions; it begins
with formulating a (n)method, testing it through (o)correctness and then assessing its
(p)research.

Use Code NIMISHA for maximum Discount

For any query call or whatsapp 9953592800


179

A. Both m-o and n-p


B. Only m-p
C. Both m-n and o-p
D. Only n-o
E. No replacement required

38. 2020-21 was one of Indian agriculture’s finest moments, as memorable as 1967-68 that
(m)harvested the Green Revolution; while much of the country was (n)planted of economic
activity in Covid-19’s first and second waves, farmers not only (o)inaugurated their standing
rabi crop from late March 2020 but also (p)locked out aggressively for the next two
seasons.

A. Both m-o and n-p


B. Only m-p
C. Both m-n and o-p
D. Only n-o
E. No replacement required

39. The accompanying (m)increase shows the (n)level of rice and wheat both at the all-
India (o)offtake and for the three poorest states as per the NITI Aayog’s National
Multidimensional Poverty Index — Bihar, Jharkhand and Uttar Pradesh (UP); all three
registered significant (p)chart in offtake levels post-NFSA between 2013-14 and 2019-20.

A. Both m-o and n-p


B. Only m-p
C. Both m-n and o-p
D. Both m-p and n-o
E. No replacement required

Use Code NIMISHA for maximum Discount

For any query call or whatsapp 9953592800


180

40. It is ironic that Pratap Bhanu Mehta’s recent article is titled ‘With eyes wide open’ (IE,
April 22); the real (m)nature of the (n)abyss may be obvious to everyone else, but what he
presents is the (o)view of a small class of people who have made it their sole (p)virtue to
keep their eyes firmly shut.

A. Both m-o and n-p


B. Only m-p
C. Both m-n and o-p
D. Both m-p and n-o
E. No replacement required

41. Across European capitals, there is understandable (m)image at Emmanuel Macron’s


(n)centrist in the French presidential election; internationally, he has cultivated the
(o)jubilation of being a pro-market, pro-Europe (p)victory under whose leadership the
French economy and Paris’s role in geopolitical affairs have become more salient.

A. Both m-o and n-p


B. Only m-p
C. Both m-n and o-p
D. Both m-p and n-o
E. No replacement required

42. For Delhi, too, Paris’s (m)outward orientation has been (n)beneficial — France has
been a supporter of its position regarding a (o)rules-based order in Asia and (p)strategic
and economic ties between the two countries have deepened.

A. Both m-o and n-p


B. Only m-p
C. Both m-n and o-p
D. Only n-o
E. No replacement required

Use Code NIMISHA for maximum Discount

For any query call or whatsapp 9953592800


181

43. In my younger days, if we wanted to comment on any article in a newspaper, we rattled


off a short letter to the editor on our (m)journalists; now there are (n)typewriters whose
(o)discussion are in the form of an article as long as the one under (p)comments.

A. Both m-o and n-p


B. Only m-p
C. Both m-n and o-p
D. Only n-o
E. No replacement required

44. This week’s Raisina (m)forum in New Delhi is an important (n)dialogue for discussing
the new security (o)implication and its (p)environment for the world.

A. Both m-o and n-p


B. Only m-p
C. Both m-n and o-p
D. Only n-o
E. No replacement required

45. India is a Union of states, it is not a (m)configuration of states; it is not a (n)strength of


holding together but of coming together; the (o)confederation of the states which constitute
the Union can change; the Union, therefore, is integral to both the Centre and the states
because the (p)question of the Centre lies in the strength of the states.

A. Both m-o and n-p


B. Only n-p
C. Both m-n and o-p
D. Only n-o
E. No replacement required

46. We must dread the (m)thought of replicating the (n)culture of competitive freebie
politics; we must go the route of achieving higher rates of economic (o)growth; the race to
efficiency is the race to (p)prosperity.

Use Code NIMISHA for maximum Discount

For any query call or whatsapp 9953592800


182

A. Both m-o and n-p


B. Only n-p
C. Both m-n and o-p
D. Only n-o
E. No replacement required

47. It would be a profound (m)cause to recognise Babasaheb Ambedkar, whose 131st birth
anniversary was recently celebrated, as only a Dalit leader; he also worked hard for the
(n)injustice of women’s (o)aspects, believing that a society’s political, economic and social
(p)rights can only be ameliorated when men and women have equal rights.

A. Both m-o and n-p


B. Only n-p
C. Both m-n and o-p
D. Only n-o
E. No replacement required

48. The (m)heirloom of the state cricket unit passed down as a (n)residue (o)tradition, a
(p)family of feudal times, has survived the days of safari suits and is flourishing even in
times when cricket is said to be corporatised.

A. Both m-o and n-p


B. Only m-p
C. Both m-n and o-p
D. Both m-p and n-o
E. No replacement required

49. At a time when, by all (m)accounts, more and more people in the country disdain
(n)nepotism and when promises of equal (o)oopportunity are emphatically made by
popular leaders to audiences filled with fresh and young (p)faces, it appears that the BCCI
continues to live in a bubble.

A. Both m-o and n-p


B. Only m-p
C. Both m-n and o-p

Use Code NIMISHA for maximum Discount

For any query call or whatsapp 9953592800


183

D. Both m-p and n-o


E. No replacement required

50. The Trumpian-style (m)vote on Pakistan’s Constitution by former Prime Minister Imran
Khan, who was removed from office following a parliamentary (n)assault of no-confidence
in early April, is the latest (o)power of just how far some of the country’s elected leaders will
go to secure and stay in (p)demonstration.

A. Both m-o and n-p


B. Only m-p
C. Both m-n and o-p
D. Both m-p and n-o
E. No replacement required

Different Patterns On Word Rearrangement

Direction(51-56): Below are some statements, given in a random order. When these
statements are sequenced properly, they'll form a coherent and meaningful
paragraph.
In each of the following sentences, there are THREE words (A), (B) and (C)
highlighted in bold. The words may or may not be at correct positions. Select the
option that gives the correct set of words as a replacement to these words. In case
the sentence is correct, select E.

(a) The Telecom Regulatory Authority of India will no longer be able to submit
(m)revocation relating to the terms and licence conditions for service providers or suggest
(n)failure of a licence for (o)recommendations to comply with licensing terms.

(b) Moreover, the (m)regulator will also severely diminish the (n)bill of the existing telecom
(o)role.

(c) Is there a Cheshire (m)grin in the Narendra Modi government and has its (n)cat
widened after the draft Indian telecommunication bill 2022 was tossed into the public
(o)domain last week?

Use Code NIMISHA for maximum Discount

For any query call or whatsapp 9953592800


184

(d) The umbrella legislation also threatens to create a regulatory (m)whimsicality that could
encourage (n)decision-making in (o)quagmire even as it sweeps aside three pieces of
ancient legislation: the Indian Telegraph Act, 1885, the Indian Wireless Telegraphy Act,
1933 and the Telegraph Wires (Unlawful Possession) Act, 1950.

(e) The draft bill has triggered (m)trepidation on several counts; the immediate cause for
(n)realm is the attempt to bring over-the-top communication services like WhatsApp,
Telegram and Google Duo within the (o)consternation of regulation.

(a) The Telecom Regulatory Authority of India will no longer be able to submit
(m)revocation relating to the terms and licence conditions for service providers or suggest
(n)failure of a licence for (o)recommendations to comply with licensing terms.

51. Which of the following arrangements of WORDS is the correct arrangement of


words?

A. n o m
B. m o n
C. o m n
D. o n m
E. No replacement required

(b) Moreover, the (m)regulator will also severely diminish the (n)bill of the existing telecom
(o)role.

52. Which of the following arrangements of WORDS is the correct arrangement of


words?

A. n o m
B. m o n
C. o m n
D. o n m
E. No replacement required

Use Code NIMISHA for maximum Discount

For any query call or whatsapp 9953592800


185

(c) Is there a Cheshire (m)grin in the Narendra Modi government and has its (n)cat
widened after the draft Indian telecommunication bill 2022 was tossed into the public
(o)domain last week?

53. Which of the following arrangements of WORDS is the correct arrangement of


words?

A. n o m
B. m o n
C. o m n
D. n m o
E. No replacement required

(d) The umbrella legislation also threatens to create a regulatory (m)whimsicality that could
encourage (n)decision-making in (o)quagmire even as it sweeps aside three pieces of
ancient legislation: the Indian Telegraph Act, 1885, the Indian Wireless Telegraphy Act,
1933 and the Telegraph Wires (Unlawful Possession) Act, 1950.

54. Which of the following arrangements of WORDS is the correct arrangement of


words?

A. n o m
B. m o n
C. o m n
D. n m o
E. No replacement required

(e) The draft bill has triggered (m)trepidation on several counts; the immediate cause for
(n)realm is the attempt to bring over-the-top communication services like WhatsApp,
Telegram and Google Duo within the (o)consternation of regulation.

Use Code NIMISHA for maximum Discount

For any query call or whatsapp 9953592800


186

55. Which of the following arrangements of WORDS is the correct arrangement of


words?

A. n o m
B. o n m
C. o m n
D. n m o
E. No replacement required

56. Which of the following combinations is the combination of THREE CONSECUTIVE


statements after the correct rearrangement?

A. c e b
B. d b a
C. e b a
D. e d a
E. None of the above options

Direction(57-59): There are three statements. In each of the statements, there is a


PHRASAL VERB. Those three phrasal verbs may not be in their correct places. Below
are some arrangements of those phrasal verbs. Choose the correct arrangement. If all
the phrasal verbs are correctly placed, choose E as your answer.

57.
I. The bill is designed to (m)weigh up to stop predatory pricing, and raise the prospect
for the write-off of legacy dues of telecom companies in exceptional circumstances.
II. It aims to put a system in place to (n)put forward each caller as it ratchets up the
fight against cyber fraud.
III. Last month, TRAI was asked to start another consultation process to (o)single out
the virtues of OTT regulation.

A. n m o
B. o n m
C. m o n
D. n o m

Use Code NIMISHA for maximum Discount

For any query call or whatsapp 9953592800


187

E. No replacement required

58.
I. Although Nitish and Tejashwi both acquiesced, it (m)put down murmurs across the
political spectrum.
II. Some saw the move as a deliberate design to place the two leaders on the same
pedestal, while others (n)kicked off it as an attempt to overshadow him.
III. In the spring of 1822, an employee in one of the world’s earliest offices (o)looked
over in a letter to a friend, “You don’t know how wearisome it is to breathe the air of
four pent walls, without relief, day after day.”

A. n m o
B. o n m
C. m o n
D. n o m
E. No replacement required

59.
I. As professionals draw up for another unremarkable workday from home wearing
pajama bottoms, it may be prudent to interrogate the office’s contribution to modern
life.
II. It is no longer uncommon to track down families with twin earners — usually the
husband and the wife — occupying designated corners of their home office.
III. Lamb would have been perfectly happy in the new normal where employers and
employees plump for the privilege to work from home.

A. n m o
B. o n m
C. m o n
D. n o m
E. No replacement required

Use Code NIMISHA for maximum Discount

For any query call or whatsapp 9953592800


188

Direction(60-62): In the following question, there are two different statements. In each
of the statements there are three words highlighted. Hence, total SIX words are
highlighted. These words may not be placed correctly. Choose the correct
arrangement.

60.
I. Loss and longing are deeply ingrained in the cultural (m)productivity of the office; in
spite of embedded fault lines, the office as a realm of (n)regime was blessed with the
human (o)burnout.
II. Perhaps one of the biggest reasons for missing office has been further (p)touch to an
already fragile work-life balance; (q)reminiscence among employees is increasingly
being attributed to a tyrannical WFH (r)disruptions with seemingly endless hours of
labour.

A. q m p r o n
B. n o q p r m
C. o q r m n p
D. m p q r o n
E. No replacement required

61.
I. Deep and simmering communal (m)identities in India have in recent days been
mirrored on an international platform, with incidents of (n)trigger and
(o)provocations playing out on the streets of Leicester in the United Kingdom and
New Jersey in the United States of America.
II. Reports suggest that a recent India-Pakistan cricket match was the (p)divisiveness;
however, the violence and (q)tensions have focused not on national (r)violence but
increasingly on illiberal, religious ones.

A. q m p r o n
B. n o q p r m
C. o q r m n p
D. q r p n o m
E. No replacement required

Use Code NIMISHA for maximum Discount

For any query call or whatsapp 9953592800


189

62.
I. The media in the USA have often been ridiculed as a (m)lap on the (n)poodle of the
ruling regime; the media’s (o)underbelly is, however, not a certificate of its benign
character.
II. There is a dark (p)spectrum that has been brought to light, once again, by
(q)criticisms from two ends of the (r)docility.

A. q m p r o n
B. n m r o q p
C. o q r m n p
D. q r p n o m
E. No replacement required

Different Patterns On Word Rearrangement

Direction(63-68): Below are some statements, given in a random order. When these
statements are sequenced properly, they'll form a coherent and meaningful
paragraph.
In each of the following sentences, there are THREE words highlighted. The words
may or may not be at correct positions.

In each of the first FIVE questions, you'll be given TWO STATEMENTS from the entire
passage. You've to choose the correct arrangement of the words in these two
statements. The arrangement would be for two statements consecutively. In case the
words are arranged correctly, select E.

(a) Only last month we became the (m)foreman of a so-called Chinese scientific (n)need,
Yuan Wang 5, docked at Sri Lanka’s (o)capabilities.

Use Code NIMISHA for maximum Discount

For any query call or whatsapp 9953592800


190

(b) Army (p)onlooker General Manoj Pande recently emphasised the (q)harbour to
develop India’s grey-zone (r)vessel.

(c) Wars of tomorrow will be fought across (m)spook (n)intercontinental and


(o)surefooted domains.

(d) In reality it was a (p)traditional ship (q)newfangled of tracking (r)multiple missiles.

(e) And China over the last (m)instance has emerged as a fine (n)fusion of grey-zone
(o)assets.

(f) Look, for (p)decennium, at the (q)warfare between Chinese civilian and military naval
(r)practitioner.

(g) Similarly, China, which has the world’s largest navy, has been deploying civilian
(m)worries and a maritime (n)squadron to assert its maritime (o)forces in the South China
Sea.

(h) General Pande is right that such civilian-military maritime (p)claims mean formidable
strategic and tactical (q)trawlers for India and to counter this India needs to seriously
upgrade its coastal defence and sharpen its naval (r)militia.

(i) This, despite the fact that the Permanent Court of Arbitration in 2016 had ruled against
China’s so-called Nine Dash Line cartographical (m)states that laid (n)region to almost all
of the South China Sea (o)island.

(j) Also, Chinese fishing vessels numbering in hundreds regularly turn up at disputed
(p)innovation groups like the Spratlys; they even encroach upon traditional fishing
(q)grounds and exclusive economic zones (EEZs) of neighbouring (r)claim.

(a) Only last month we became the (m)foreman of a so-called Chinese scientific (n)need,
Yuan Wang 5, docked at Sri Lanka’s (o)capabilities.

(b) Army (p)onlooker General Manoj Pande recently emphasised the (q)harbour to
develop India’s grey-zone (r)vessel.

Use Code NIMISHA for maximum Discount

For any query call or whatsapp 9953592800


191

63. Which of the following arrangements of WORDS can be chosen as the correct
rearrangement of WORDS in the above two statements consecutively?

A. p r q m n o
B. n o q p r m
C. o q r m n p
D. m p q r o n
E. No replacement required

(c) Wars of tomorrow will be fought across (m)spook (n)intercontinental and


(o)surefooted domains.

(d) In reality it was a (p)traditional ship (q)newfangled of tracking (r)manifold missiles.

64. Which of the following arrangements of WORDS can be chosen as the correct
rearrangement of WORDS in the above two statements consecutively?

A. p r q m n o
B. n o q p r m
C. o q r m n p
D. r p q m o n
E. No replacement required

(e) And China over the last (m)instance has emerged as a fine (n)fusion of grey-zone
(o)assets.

(f) Look, for (p)decennium, at the (q)warfare between Chinese civilian and military naval
(r)practitioner.

Use Code NIMISHA for maximum Discount

For any query call or whatsapp 9953592800


192

65. Which of the following arrangements of WORDS can be chosen as the correct
rearrangement of WORDS in the above two statements consecutively?

A. p r q m n o
B. n o q p r m
C. o q r m n p
D. r p q m o n
E. No replacement required

(g) Similarly, China, which has the world’s largest navy, has been deploying civilian
(m)worries and a maritime (n)squadron to assert its maritime (o)forces in the South China
Sea.

(h) General Pande is right that such civilian-military maritime (p)claims mean formidable
strategic and tactical (q)trawlers for India and to counter this India needs to seriously
upgrade its coastal defence and sharpen its naval (r)militia.

66. Which of the following arrangements of WORDS can be chosen as the correct
rearrangement of WORDS in the above two statements consecutively?

A. p r q m n o
B. n o q p r m
C. q r p o m n
D. r p q m o n
E. No replacement required

(i) This, despite the fact that the Permanent Court of Arbitration in 2016 had ruled against
China’s so-called Nine Dash Line cartographical (m)states that laid (n)region to almost all
of the South China Sea (o)island.

Use Code NIMISHA for maximum Discount

For any query call or whatsapp 9953592800


193

(j) Also, Chinese fishing vessels numbering in hundreds regularly turn up at disputed
(p)innovation groups like the Spratlys; they even encroach upon traditional fishing
(q)grounds and exclusive economic zones (EEZs) of neighbouring (r)claim.

67. Which of the following arrangements of WORDS can be chosen as the correct
rearrangement of WORDS in the above two statements consecutively?

A. p r q m n o
B. p r n o q m
C. q r p o m n
D. r p q m o n
E. No replacement required

(a) Only last month we became the onlooker of a so-called Chinese scientific vessel, Yuan
Wang 5, docked at Sri Lanka’s Hambantota harbor.

(b) Army foreman General Manoj Pande recently emphasised the need to develop India’s
grey-zone capabilities.

(c) Wars of tomorrow will be fought across manifold traditional and newfangled domains.

(d) In reality it was a spook ship surefooted of tracking intercontinental missiles.

(e) And China over the last decennium has emerged as a fine practitioner of grey-zone
warfare.

(f) Look, for instance, at the fusion between Chinese civilian and military naval assets.

(g) Similarly, China, which has the world’s largest navy, has been deploying civilian trawlers
and a maritime militia to assert its maritime claims in the South China Sea.

(h) General Pande is right that such civilian-military maritime forces mean formidable
strategic and tactical worries for India and to counter this India needs to seriously upgrade
its coastal defence and sharpen its naval squadron.

Use Code NIMISHA for maximum Discount

For any query call or whatsapp 9953592800


194

(i) This, despite the fact that the Permanent Court of Arbitration in 2016 had ruled against
China’s so-called Nine Dash Line cartographical innovation that laid claim to almost all of the
South China Sea region.

(j) Also, Chinese fishing vessels numbering in hundreds regularly turn up at disputed island
groups like the Spratlys; they even encroach upon traditional fishing grounds and exclusive
economic zones (EEZs) of neighbouring states.

68. Which of the following combinations is the combination of FOUR CONSECUTIVE


STATEMENTS after the correct rearrangement?

A. b a c d
B. e d h g
C. a d g i
D. c e f d
E. None of the above options

Direction(69-72): In the following question, there are TWO columns containing SIX
different sentences. In each of the sentences, a word has been highlighted. Hence,
there are a total SIX HIGHLIGHTED WORDS. Now, these words may not be placed
correctly. You've to choose the CORRECT REARRANGEMENT for the words for all
the six sentences CONSECUTIVELY.

69.
Column I
A. A solution to a dispute over the (m)mourning of Israeli troops in west Beirut
appeared to be moving closer.
B. The central Congress (I) (n)integrity is worried about reports from Hyderabad that a
revolt against the New Andhra chief minister is brewing in the legislature party.
C. A joint declaration released simultaneously in New Delhi and Moscow reaffirmed their
strong opposition to outside (o)situation.
Column II

Use Code NIMISHA for maximum Discount

For any query call or whatsapp 9953592800


195

D. They should pay full respect to the independence, sovereignty, territorial


(p)leadership and non-aligned status of the countries of the region.
E. His importance for India and for India-Japan relations is reflected in the nation’s
(q)presence.
F. He could clearly see the changing geopolitical and economic (r)interference and
tried his best to collaborate with India on multiple dimensions to counter China.

A. q m p r o n
B. n m r o q p
C. o q r m n p
D. q p r n m o
E. No replacement required

70.
Column I
A. The India-Japan ministerial meeting (2+2) on defense and security (m)stoutness and
joint maritime exercises reflect continuity in India-Japan cooperation.
B. During his political career, Abe pushed for (n)synergy, security and a rules-based
international order in the Indo-Pacific.
C. Japan undertook a bilateral (o)grantee with India including the Joint Declaration on
Security Cooperation in 2008.
Column II
D. Modi and Kishida would take these initiatives to further bilateral and regional
infrastructure development and ensure resilient (p)announcement.
E. India is the largest (q)enterprise of Japanese Official Development Assistance
(ODA).
F. The assistance accelerated following Japan’s (r)shielding in 2016 to help India build
critical infrastructure and address social and environmental issues caused by rapid
growth.

A. q m p r o n
B. n m q r o p
C. o q r m n p
D. q p r n m o
E. No replacement required

Use Code NIMISHA for maximum Discount

For any query call or whatsapp 9953592800


196

71.
Column I
A. Some of the projects which have wider implications for the environment are the
seawater (m)bequest project in Gujarat, solar projects in Rajasthan, etc.
B. India has opened up the defence industry and is looking for cooperation and
(n)centrality in space, cyber technologies and artificial intelligence (AI).
C. Considering the (o)outlay of China in global value chains India offers the best
alternative to diversify risk.
Column II
D. Abe laid the foundation for India-Japan relations; it's time to take his (p)desalination
forward.
E. Forty-two years ago, the Supreme Court took the view that the death penalty did not
violate the Constitution and prescribed a (q)retribution it hoped would ensure
fairness in sentencing.
F. The courts have taken differing approaches in which factors are relevant to
sentencing, and how best to bring in factors relevant to (r)framework.

A. q m p r o n
B. n m q r o p
C. p o n m r q
D. q p r n m o
E. No replacement required

72.
Column I
A. At Saraceno’s request, no spider webs have been cleaned up by the custodial
(m)standpoint and the curators are looking at natural spun thread as works of art.
B. The (n)garnering behind Saraceno’s work is a simple one: Human beings are the
invasive species.
C. Perhaps, by not stepping on insects and trying instead to see them as creative
beings, another new (o)personnel is possible.
Column II
D. The fight between the Left and the Right in the United States, or the (p)plight
emanating from the Black Lives Matter movement, are pretty evident.
E. A clear picture of the kharif (q)precept will emerge over the next month or so.

Use Code NIMISHA for maximum Discount

For any query call or whatsapp 9953592800


197

F. The political (r)polarisation in Rajasthan is as much the creation of Chief Minister


and Congress Presidential candidate Ashok Gehlot as it is of the Congress high
command.

A. q m p r o n
B. n m q r o p
C. p o n m r q
D. o q m r n p
E. No replacement required

Direction (73-92): In each of the following questions given below, a


sentence/sentences is /are given with some bold words which may contain errors.
Below each of them, a table is given with two columns in which column ‘A’ contains
the list of bold words, and in column ‘B’ the suggested corrections are listed. You
have to choose the best alternative among the four given options. If no correction is
required against the given bold words, mark (e) .i.e. none of the above as your
answer.

Q73. The attraction of lithium, the metal dubbed white petroleum, has proved reproval for
Gina Rinehart, Australia’s richest person and one of the world’s richest women. But her
entry into one of the commodities being driven higher by strong demand for the batteries
used to power electric vehicles (EVs) is far from conventional.

Column (A) Column (B)


(I) Attract (V) Lure
(II) Reproval (VI) Irresistible
(III) Commodities (VII) Critiques
(IV) Conventional (VIII) Commendable

(a)I-V
(b)II-VI
(c)III –VII and IV – VIII
(d)I-V and II-VI
(e)None of the above

Use Code NIMISHA for maximum Discount

For any query call or whatsapp 9953592800


198

Q74. The problem with hearing aids is the lack of examination about what they can and
cannot do. Even today, with modern digital technology to dither out background noises,
most hearing aids will not allow the user to hear only the person talking.

Column (A) Column (B)


(I) Problem (V) Promotion
(II) Examination (VI) Information
(III) Dither (VII) Filter
(IV) Person (VIII) Individuals

(a)II-VI and III-VII


(b)I-V and IV – VIII
(c)II-VI
(d)I – V
(e)None of the above

Q75. The augments are intended to prevent the country’s dominant tech companies from
squeezing out rivals by offering below-cost services, exclusive distribution requirements,
engaging in price-fixing, as well as using algorithms and other restrictive technologies to
manipulate the market.

Column (A) Column (B)


(I) Augments (V) Regulations
(II) Distribution (VI) Registration
(III) Algorithms (VII) Alter
(IV) Manipulate (VIII) Synchronize

(a)I-V
(b)II-VI
(c)III –VII and IV – VIII
(d)I-V and II-VI
(e)None of the above

Q76. The overall design of the yacht is characterized by large and flexible outdoor spaces
that can be used for both sun and shade as well as tender and toy storage. The duct can
also contend with modular equipment including a swimming pool and a removable battery
bank.
Use Code NIMISHA for maximum Discount

For any query call or whatsapp 9953592800


199

Column (A) Column (B)


(I) Characterized (V) Normalized
(II) Sun (VI) Son
(III) Duct (VII) Deck
(IV) Contend (VIII) Accommodate
(a)I-V and II-VI
(b)III-VII and IV – VIII
(c)III-VII
(d)IV – VIII
(e)None of the above

Q77. Ultimately, like other fintechs that have started out focusing on one segment, Affirm is
aiming to profit by selling more financial services to a obtained customer base that buys its
fee-transparency pitch.

Column (A) Column (B)


(I) Focused (V) Amused
(II) Profit (VI) Benefiting
(III) Obtained (VII) Loyal
(IV) Transparency (VIII) Expectancy

(a)I-V
(b)II-VI and IV – VIII
(c)III-VII
(d)I-V and IV – VIII
(e)None of the above

Q78. After a personal from the Army and the Engineer Task Force opened the mouth of
the tunnel in the morning, a team started removing the slush using earthmovers and
excavators. An ITBP official said the rescue team had gone into the tunnel with victim-
locating cameras and catcher dogs.

Column (A) Column (B)


(I) Personal (V) Personnel
(II) Mouth (VI) Cave
(III) Slush (VII) Mudding
(IV) Catcher (VIII) Sniffer
Use Code NIMISHA for maximum Discount

For any query call or whatsapp 9953592800


200

(a)I –V and IV – VIII


(b)II-VI
(c)III-VII
(d)IV–VIII
(e)None of the above

Q79. The commercial begins with a boost from Martin Luther King: “Injustice anywhere is a
threat to justice everywhere”. Images from the farmers’ tractor parade and their border
camps are overpaid with text alleging human rights violations against protesters. Punjabi
music plays as “No Farmers, No Food, No Future” flashes on the screen.

Column (A) Column (B)


(I) Boost (V) Quote
(II) Overpaid (VI) Overlaid
(III) Violations (VII) Exaggerate
(IV) Flashes (VIII) Encompasses

(a)I-V
(b)II-VI
(c)III –VII and IV – VIII
(d)I-V and II-VI
(e)None of the above

Q80. The Minister also took a dig at the opposition, saying that one party was engaged only
in promoting the identity of one family; while the other party was busy ensuring it does not
honor its own promises.

Column (A) Column (B)


(I) Dig (V) Take
(II) Engaged (VI) Enraged
(III) Ensuring (VII) Ensuing
(IV) Promises (VIII) Premises

(a)II-VI and III-VII


(b)I-V and IV – VIII
Use Code NIMISHA for maximum Discount

For any query call or whatsapp 9953592800


201

(c)II-VI
(d)I – V
(e)None of the above

Q81. Following a plea alleging illegal and unregulated use of parks for controversial
purposes, the National Green Tribunal has directed the Delhi Pollution Control Committee to
maintain vigil and prevent violation of environmental norms.

Column (A) Column (B)


(I) Alleging (V) Adapting
(II) Controversial (VI) Commercial
(III) Vigil (VII) Rigid
(IV) Norms (VIII) Rulings

(a)I-V
(b) II-VI
(c)III –VII and IV – VIII
(d)I-V and II-VI
(e)None of the above

Q82. The accused has been running a tools shop for the past 13 years. But due to a
financial crisis, he provided the space to his uncle, who used it for the liberal telephone
exchange. He also unexcited a high-speed Wi-Fi connection for the set-up.

Column (A) Column (B)


(I) Accused (V) Disparaged
(II) Space (VI) Break
(III) Liberal (VII) Illegal
(IV) Unexcited (VIII) Provided

(a)I-V and II-VI


(b)III-VII and IV – VIII
(c)III-VII
(d)IV – VIII
(e)None of the above

Q83. The government said that about the program, UPSC aspirants in schools get a chance
to inspect with young IAS/IPS officers every month and officers share their experiences,
Use Code NIMISHA for maximum Discount

For any query call or whatsapp 9953592800


202

strategies, and insights about UPSC exam preparations to help students develop a better
understanding of the tough test.

Column (A) Column (B)


(I) About (V) Under
(II) Inspect (VI) Interact
(III) Insights (VII) Insides
(IV) Understanding (VIII) Undermining

(a)I-V
(b)II-VI and IV – VIII
(c)III-VII
(d)I-V and II-VI
(e)None of the above

Q84. If Myanmar’s democracy prior to the February 2021 coup was inadequate towards
minorities, its political future will be a lot more complicated, making the saviors of outside
powers far more constrained.

Column (A) Column (B)


(I) Inadequate (V) Intolerant
(II) Political (VI) Tolerable
(III) Saviours (VII) Choices
(IV) Constrained (VIII) Constraint

(a)I –II and III-VII


(b)II-VI
(c)III-VII
(d)IV–VIII
(e)None of the above

Q85. Two untoward implications could result from under-investing and spreading funds too
thinly. Continuing the pious of health and wellness centers without enough funding would
mean that the full range of pompous services will not be available.

Column (A) Column (B)


(I) Untoward (V) Arrogant
Use Code NIMISHA for maximum Discount

For any query call or whatsapp 9953592800


203

(II) Thinly (VI) Responsibly


(III) Pious (VII) Prompt
(IV) Pompous (VIII) Promised

(a)I-V
(b)II-VI
(c)III –VII and IV – VIII
(d)I-V and II-VI
(e)None of the above

Q86. India is heavily accorded in dam development and growth of hydropower, largely in
the Himalayas region — especially to cut carbon emissions. By one estimate, if the national
plan to construct dams in 28 river valleys in the hills is realized in a few decades, the Indian
Himalayas will have one dam for every 32 km, among the world’s highest densities.

Column (A) Column (B)


(I) Accorded (V) Invested
(II) Carbon (VI) Footprint
(III) Realized (VII) Refused
(IV) Densities (VIII) Communities

(a)II-VI and III-VII


(b)I-V and IV – VIII
(c)II-VI
(d)I – V
(e)None of the above

Q87. The decision is debatable for the unusual delay in the Governor reaching his
conclusion as much for its legal correctness. It took Mr. Purohit more than two years to
decide the question. The Supreme Court has been asking him to avoid a situation in which
it would have to intervene.

Column (A) Column (B)


(I) Debatable (V) Unacceptable
(II) Correctness (VI) Competitiveness
(III) Question (VII) Distraction
(IV) Intervene (VIII) Reconvene
Use Code NIMISHA for maximum Discount

For any query call or whatsapp 9953592800


204

(a)I-V
(b)II-VI
(c)III –VII and IV – VIII
(d)I-V and II-VI
(e)None of the above

Q88. In 1991, when the then Finance Minister Manmohan Singh ushered in economic
reforms that catapulted India into the global economy, I had asked him how he removed to
balance the frailty economic growth with environmental protection.

Column (A) Column (B)


(I) Ushered (V) Provoked
(II) Catapulted (VI) Chocked
(III) Removed (VII) Intended
(IV) Frailty (VIII) Rapid

(a)I-V and II-VI


(b)III-VII and IV – VIII
(c)III-VII
(d)IV – VIII
(e)None of the above

Q89. Since the epidemic was transitioning to the endemic phase from the third week of
January, the vaccination is not expected to impact the epidemic, but offers protection to
healthcare workers since terrifying exposure to infection is an occupational hazard.

Column (A) Column (B)


(I) Transitioning (V) Persuading
(II) Impact (VI) Support
(III) Terrifying (VII) Repeated
(IV) Hazard (VIII) Reunion

(a)I-V
(b)II-VI and IV – VIII
(c)III-VII
(d)I-V and IV – VIII
Use Code NIMISHA for maximum Discount

For any query call or whatsapp 9953592800


205

(e)None of the above

Q90. During his eight-year criticism, the Frankfurt-based institution’s policy to buy unlimited
quantities of sovereign bonds of ailing economies was a alleviate moment in stabilizing the
single currency area, even if the initiative incurred the wrath of Germany.

Column (A) Column (B)


(I) Criticism (V) Stewardship
(II) Ailing (VI)Relating
(III) Alleviate (VII)Pivotal
(IV) Incurred (VIII) Nudged

(a)I–II and III-VII


(b)II-VI
(c)III-VII
(d)IV–VIII
(e)None of the above

Q91. Legally, the best-case scenario would be to submit the government from being able to
directly order intermediaries to block access to online information, except in narrowly-
defined emergency cases, and to require it to go through court to do so, with an adequate
opportunity for the conceited parties to defend themselves.

Column (A) Column (B)


(I) Submit (V) Prohibit
(II) Block (VI) Retort
(III) Emergency (VII) Disaster
(IV) Conceited (VIII) Affected

(a)I –V and IV – VIII


(b)II-VI
(c)III-VII
(d)IV–VIII
(e)None of the above

Use Code NIMISHA for maximum Discount

For any query call or whatsapp 9953592800


206

Q92. The bloody incident at Galwan on June 15, 2020, the first involving charities since
1975, brought about the collapse of the prevailing consensus that bilateral ties could
develop in parallel with efforts to resolve the boundary question and the maintenance of
peace and answerability.

Column (A) Column (B)


(I) Charities (V) Casualties
(II) Prevailing (VI) Instilling
(III) Resolve (VII) Absolve
(IV) Answerability (VIII) Tranquillity

(a)I-V
(b)II-VI and IV – VIII
(c)III-VII
(d)I-V and IV – VIII
(e)None of the above

Directions (93-): In the following questions, a word has been highlighted in the given
sentence. Following the sentence are a few options which suggest the possible
meaning of the given highlighted word. Identify the most appropriate option that
suggests the correct meaning of the highlighted word.

93. They accused the prime minister of using sledgehammer tactics.

A. Forceful
B. Regretful
C. merciful
D. All of the above.
E. None of the above.

94. The haughty attitudes of the pre-war era were slow to disappear.

A. modest
B. humble

Use Code NIMISHA for maximum Discount

For any query call or whatsapp 9953592800


207

C. unfriendly
D. friendly
E. 1, 2, and 4

95. It might sound callous, but I don't care if he's homeless. He's not living with me!

A. Empathetic
B. Compassionate
C. Merciful
D. uncaring
E. None of the above.

ANSWER

1.(Answer - B. Both m-p and n-o

pyre - a heap of combustible material, especially one for burning a corpse as part of a
funeral ceremony.
cadaver - dead body.

With the images from the Covid pandemic, of burning pyres and floating cadavers, still
fresh in our minds, the current debate on the magnitude of mortality during 2020 and 2021
looks surreal and unsympathetic.)

2.(Answer - C. Both m-p and n-o

Use Code NIMISHA for maximum Discount

For any query call or whatsapp 9953592800


208

By deaths per thousand population, India, however, does not figure among the top 100
countries, though the infection fatality rate of 1.2 per cent places it in seventh position
globally.)

3.(Answer - A. Only n-o

While the freshness of this religiosity heals my tormented soul, I cannot forget the horror
of yet another kind of vehemency sanctified in the name of educating the child.

vehemency - violence.)

4.(Answer - D. Both m-n and o-p

We compel our children to study physics or botany, but do we really encourage them to
look at a tree with wonder, merge with the amazing sunset, and overcome the duality of the
“observer” versus the “observed”?

compel - to force.
merge with - absorb.
overcome - win a victory over.)

5.(Answer - C. Both m-o and n-p

Education is not just to pass examinations, take a degree, a job, get married and settle
down, but also to be able to listen to the birds, to see the ether, to see the extraordinary
beauty of a tree, and the shape of the hills, and to feel with them, to be really, directly in
touch with them.

the ether - the sky.)

6.(Answer - D. Both m-n and o-p

The hazy outlines of a life together were beginning to take shape, but, as an inconvertible
introvert, deficient forever in articulating my emotions in words, I turned to music to take
the relationship ashore.)

Use Code NIMISHA for maximum Discount

For any query call or whatsapp 9953592800


209

7.(Answer - C. Both m-p and n-o

Looking back, music had always been the soundtrack of my life, and the changing musical
interfaces, a record of the passage of time.)

8.(Answer - B. Both m-n and o-p

In a batch of petitions challenging the law of sedition, contained in section 124A of the
Indian Penal Code (IPC), the Supreme Court on Wednesday issued a slew of interim
directions.

a slew of - a large number of.


sedition - conduct or speech inciting people to rebel against the authority of a state or
monarch.
a batch of - a quantity or consignment of goods produced at one time.)

9.(Answer - C. Only m-p

The Court granted the central government time till May 10 to file its response, failing which
it intended to decide the question of whether there was a requirement to refer the
challenge to a seven-judge bench.)

10.(Answer - E. No replacement required

consequential - significant.
downright - absolute.
explicit - stated clearly and in detail, leaving no room for confusion or doubt.)

11.(Answer - C. Both m-n and o-p

After some deliberation, the Court refused to confine itself to the suggestions proposed by
the government and passed directions hoping and expecting the state and central
governments to restrain themselves from registering new FIRs, continuing pending
investigations, or arresting people under section 124A IPC.

Use Code NIMISHA for maximum Discount

For any query call or whatsapp 9953592800


210

deliberation - long and careful consideration or discussion.)

12.(Answer - B. Both m-n and o-p

This order appears to be a small win but left a lot to be desired; also, its implementation at
the ground level remains to be seen and lessons should have been taken from the fact that
police across the country continued filing FIRs under section 66A of the Information
Technology Act.)

13.(Answer - B. Both m-p and n-o

British ships and helicopters have attacked Port Stanley and Port Darwin and fighter
bombers sank a fishing vessel and machine-gunned lifeboats in the resumption of the
Anglo-Argentine conflict over the Falkland Islands.

resumption - the action of beginning something again after a pause or interruption.


bomber - an aircraft designed to carry and drop bombs.)

14.(Answer - A. Only o-p

A team of sleuths from the communal wing of the special branch of the Delhi police and the
Union Home Ministry’s Intelligence Bureau has been asked to unearth the conspiracy
behind the cases of sacrilege in some temples of the capital.

sacrilege - violation or misuse of what is regarded as sacred.)

15.(Answer - E. No replacement required

While police pickets have been posted near temple and other sensitive spots, a close
watch is being kept on the activists of Dal Khalsa; there are intelligence reports of
miscreants doing something untoward after the incidents of sacrilege in Punjab and
Haryana.

picket - a soldier or small group of soldiers performing a particular duty, especially one sent
out to watch for the enemy.
miscreant - a person who has done something wrong or unlawful.)

Use Code NIMISHA for maximum Discount

For any query call or whatsapp 9953592800


211

16.(Answer - C. Only m-p

Inside closed doors, artists now admit to being over-cautious and fearful; in public spaces,
meanwhile, they are wary of viewers who might deem their work offensive.

over-cautious - excessively cautious.


wary - feeling or showing caution about possible dangers or problems.)

17.(Answer - B. Both m-p and n-o

Public outrage does lead to conversations on the subject and an assertion of the need to
protect artistic liberties, but the vandals often succeed in fulfilling their immediate endeavour
of bringing down the work and the artist is left alone in the end.

outrage - an extremely strong reaction of anger, shock, or indignation.)

18.(Answer - D. Both m-n and o-p

Artists aspire to encourage conversations and seek to find unorthodox ways to question
and provoke.

seek - attempt to find (something).


aspire - direct one's hopes or ambitions towards achieving something.
provoke - stimulate or give rise to (a reaction or emotion, typically a strong or unwelcome
one) in someone.)

19.(Answer - D. Both m-o and n-p

By placing assets such as houses and LPGs in the hands of women, it is challenging the
unequal status quo; it is doing so not only through policies but by bridging gendered data
deficits.

deficit - deficiency.)

20.(Answer - C. Only m-o

Use Code NIMISHA for maximum Discount

For any query call or whatsapp 9953592800


212

As a routine source of crucial information on nutrition, fertility, family planning,


reproductive, maternal and child health and mortality, the National Family Health Survey
(NFHS) is a barometer of India’s performance in securing equitable health outcomes,
especially for women.

barometer - something which reflects changes in circumstances or opinions.)

21.(Answer - D. Both m-o and n-p

The Ayushman Bharat programme has considerably expanded the scope of healthcare in
the country; but the pandemic should occasion more serious thinking on addressing the
shortfalls.)

22.(Answer - D. Both m-n and o-p

The office of the Registrar General does not give a disease-wise break-up of deaths, but
read along with other studies, the data set released on Tuesday is a useful indicator of the
pressure on the medical system during the pandemic.

break-up - the separation or breaking up of something into several pieces or sections.)

23.(Answer - B. Both m-p and n-o

The proportion of people dying for want of medical attention increased from 34.5 per cent
of all recorded deaths in 2019 to 45 per cent in 2020 — the largest single-year jump.

want - lack.)

24.(Answer - B. Both m-p and n-o

Sant Singh Sekhon was the principal and I was one of the members of the welcome
committee organising a festival of poets. Batalvi was the cynosure of all eyes: the venue

Use Code NIMISHA for maximum Discount

For any query call or whatsapp 9953592800


213

was crowded just because of him; the festival started without Batalvi and ther poets came
on the stage but the audience kept looking away from the podium and waiting intently for
the poet of the day.

podium - a small platform on which a person may stand to be seen by an audience, as


when making a speech or conducting an orchestra.
cynosure - a person or thing that is the centre of attention or admiration.)

25.(Answer - E. No replacement required

Tolerance is perhaps one of the most profound lessons that the people of Punjab have
imbibed from the decade lost to militancy; they have not let the cancer of hate eat into the
vitals of society, finding common ground in their shared heritage instead.

militancy - the use of confrontational or violent methods in support of a political or social


cause.
common ground - opinions or interests shared by each of two or more parties.)

26.(Answer - C. Only m-p

We are a postcolonial nation-state with a long history of colonial domination that has
caused irreparable damage to people at large; espite this, if people cheer for the Russian
invasion, it reflects perhaps the latent imperial ambition.)

27.(Answer - D. Both m-n and o-p

With the horrific war in Ukraine showing no signs of ending, Prime Minister Narendra Modi’s
visit to Berlin, Copenhagen, and Paris this week could give us a glimpse of India’s post-
Russian strategic future in Europe; as Russia, isolated by unprecedented Western
sanctions, deepens its coalition with China, Europe has begun to loom larger than ever
before in India’s strategic calculus.)

28.(Answer - B. Both m-p and n-o

Power generation is highly dependent on coal — about 78 per cent of it comes from this
fossil fuel — and, transportation is almost entirely dependent on oil; the Indian energy
ecosystem is, thus, highly carbon-intensive.)

Use Code NIMISHA for maximum Discount

For any query call or whatsapp 9953592800


214

29.(Answer - D. Both m-n and o-p

The most natural option of government intervention for reducing emissions is by fixing
limits of emissions through regulation, taking into consideration the Nationally Determined
Contribution targets set by the country under the Paris Agreement.

intervention - involvement.)

30.(Answer - C. Only m-p

From addressing specific ailments to preventative benefits and overall mental well-being,
yoga is now being acknowledged as a practice that helps individuals cope with the
pressures of the 21st century.

ailment - an illness, typically a minor one.)

31.(Answer - D. Both m-n and o-p

This past week, as many of us shopped on Amazon or perhaps booked flight tickets for the
summer vacations, Elon Musk evinced his interest in purchasing the social media platform
Twitter for $44 billion and Musk has reportedly sold roughly $4 billion worth of Tesla stock
over the past two days to help with financing the acquisition.

evince - reveal the presence of (a quality or feeling); indicate.)

32.(Answer - D. Both m-o and n-p

A poison pill in commercial law is a defense strategy used by a target firm to prevent or
discourage a potential hostile takeover by an acquiring company.)

33.(Answer - C. Both m-n and o-p

According to NASA, the crew will conduct a science expedition in microgravity aboard the
space station; on April 18, a SpaceX Falcon rocket was used to successfully launch a US
spy satellite force from the company’s Vandenberg Space Force base in California.)

Use Code NIMISHA for maximum Discount

For any query call or whatsapp 9953592800


215

34.(Answer - A. Both m-o and n-p

The renunciation of the self for greater good (in a tragic sense) appealed to the early
Israelis — they came to their homeland, which was in conflict and that demanded plenty of
self-sacrifice from them.

renunciation - the formal rejection of something, typically a belief, claim, or course of


action.)

35.(Answer - D. Only n-o

European Jews were the pioneers who established the state, enjoyed more power and
shaped much of the Israeli identity, one which was alien to most non-European, including
Indian, Jews.

pioneer - a person who is among the first to explore or settle a new country or area.)

36.(Answer - E. No replacement required

India’s diplomatic distance with Israeli and vocal solidarity for the Palestinian cause did not
sour Israeli affinity for Indian culture in the 1950s or later.

solidarity - unity.
affinity - a natural liking for and understanding of someone or something.)

37.(Answer - C. Both m-n and o-p

Propriety suggests a tried and tested method for making conclusions; it begins with
formulating a hypothesis, testing it through research and then assessing its correctness.

Use Code NIMISHA for maximum Discount

For any query call or whatsapp 9953592800


216

hypothesis - a supposition or proposed explanation made on the basis of limited evidence


as a starting point for further investigation.)

38.(Answer - A. Both m-o and n-p

2020-21 was one of Indian agriculture’s finest moments, as memorable as 1967-68 that
inaugurated the Green Revolution; while much of the country was locked out of economic
activity in Covid-19’s first and second waves, farmers not only harvested their standing rabi
crop from late March 2020 but also planted aggressively for the next two seasons.

harvest - gather (a crop) as a harvest.)

39.(Answer - D. Only m-p and n-o

The accompanying chart shows the offtake of rice and wheat both at the all-India level and
for the three poorest states as per the NITI Aayog’s National Multidimensional Poverty Index
— Bihar, Jharkhand and Uttar Pradesh (UP); all three registered significant increases in
offtake levels post-NFSA between 2013-14 and 2019-20.

offtake - the amount of goods purchased during a given period.)

40. (Answer - E. No replacement required

The real nature of the abyss may be obvious to everyone else, but what he presents is the
view of a small class of people who have made it their sole virtue to keep their eyes firmly
shut.

abyss - a difficult situation that brings trouble or destruction.)

41.(Answer - A. Both m-o and n-p

Across European capitals, there is understandable jubilation at Emmanuel Macron’s


victory in the French presidential election; internationally, he has cultivated the image of
being a pro-market, pro-Europe centrist under whose leadership the French economy and
Paris’s role in geopolitical affairs have become more salient.

jubilation - a feeling of great happiness and triumph.


centrist - a person who holds moderate political views.)

Use Code NIMISHA for maximum Discount

For any query call or whatsapp 9953592800


217

42.(Answer - E. No replacement required

For Delhi, too, Paris’s outward orientation has been beneficial — France has been a
supporter of its position regarding a rules-based order in Asia and strategic and economic
ties between the two countries have deepened.)

43.(Answer - C. Both m-n and o-p

In my younger days, if we wanted to comment on any article in a newspaper, we rattled off a


short letter to the editor on our typewriters; now there are journalists whose comments
are in the form of an article as long as the one under discussion.)

44.(Answer - C. Both m-n and o-p

This week’s Raisina dialogue in New Delhi is an important forum for discussing the new
security environment and its implication for the world.

implication - deduction.)

45.(Answer - A. Both m-o and n-p

India is a Union of states, it is not a confederation of states; it is not a question of holding


together but of coming together; the configuration of the states which constitute the Union
can change; the Union, therefore, is integral to both the Centre and the states because the
strength of the Centre lies in the strength of the states.

confederation - an organization which consists of a number of parties or groups united in


an alliance or league.)

46.(Answer - E. No replacement required

We must dread the thought of replicating the culture of competitive freebie politics; we
must go the route of achieving higher rates of economic growth; the race to efficiency is the
race to prosperity.

Use Code NIMISHA for maximum Discount

For any query call or whatsapp 9953592800


218

prosperity - success.)

47.(Answer - C. Both m-n and o-p

It would be a profound injustice to recognise Babasaheb Ambedkar, whose 131st birth


anniversary was recently celebrated, as only a Dalit leader; he also worked hard for the
cause of women’s rights, believing that a society’s political, economic and social aspects
can only be ameliorated when men and women have equal rights.)

48.(Answer - A. Both m-o and n-p

The tradition of the state cricket unit passed down as a family heirloom, a residue of
feudal times, has survived the days of safari suits and is flourishing even in times when
cricket is said to be corporatised.

heirloom - a valuable object that has belonged to a family for several generations.)

49.(Answer - E. No replacement required

At a time when, by all accounts, more and more people in the country disdain nepotism
and when promises of equal opportunity are emphatically made by popular leaders to
audiences filled with fresh and young faces, it appears that the BCCI continues to live in a
bubble.)

50. (Answer - C. Both m-n and o-p

The Trumpian-style assault on Pakistan’s Constitution by former Prime Minister Imran


Khan, who was removed from office following a parliamentary vote of no-confidence in early
April, is the latest demonstration of just how far some of the country’s elected leaders will
go to secure and stay in power.

assault - attack.
demonstration - an act of showing that something exists or is true by giving proof or
evidence.)

Use Code NIMISHA for maximum Discount

For any query call or whatsapp 9953592800


219

51.(Answer - C. o m n

revocation - the official cancellation of a decree, decision, or promise.)

52.(Answer - A. n o m)

53.(Answer - D. n m o

If someone is grinning like a Cheshire cat or like the Cheshire cat, they are smiling
very widely.

grin like a Cheshire cat - Smile broadly, especially in a self-satisfied way.)

54.(Answer - C. o m n

quagmire - an awkward, complex, or hazardous situation.

whimsicality - the quality of being whimsical (= unusual and strange in a way that might be
funny or annoying).

55.(Answer - C. o m n

consternation - a feeling of anxiety or dismay, typically at something unexpected.

trepidation - alarm.)

56.(Answer - B. d b a

The correct arrangement is -

c e d b a )

57.(Answer - D. n o m

put forward - to propose.

single out - to identify.

Use Code NIMISHA for maximum Discount

For any query call or whatsapp 9953592800


220

weigh up - carefully examine.)

58.(Answer - D. n o m

kick off - to start.

look over - to view; inspect something with a view to establishing its merits.

put down - to write.)

59.(Answer - E. No replacement required

draw up - to prepare.

track down - to find.

plump for - to prefer.)

60.(Answer - A. q m p r o n

Loss and longing are deeply ingrained in the cultural reminiscence of the office; in spite of
embedded fault lines, the office as a realm of productivity was blessed with the human
touch.

Perhaps one of the biggest reasons for missing office has been further disruptions to an
already fragile work-life balance; burnout among employees is increasingly being attributed
to a tyrannical WFH regime with seemingly endless hours of labour.

reminiscence - a characteristic of one thing that is suggestive of another.

burnout - physical or mental collapse caused by overwork or stress.)

61.(Answer - D. q r p n o m

Deep and simmering communal tensions in India have in recent days been mirrored on an
international platform, with incidents of violence and divisiveness playing out on the streets
of Leicester in the United Kingdom and New Jersey in the United States of America.

Use Code NIMISHA for maximum Discount

For any query call or whatsapp 9953592800


221

Reports suggest that a recent India-Pakistan cricket match was the trigger; however, the
violence and provocations have focused not on national identities but increasingly on
illiberal, religious ones.)

62.(Answer - B. n m r o q p

The media in the USA have often been ridiculed as a poodle on the lap of the ruling regime;
the media’s docility is, however, not a certificate of its benign character.

There is a dark underbelly that has been brought to light, once again, by criticisms from
two ends of the spectrum.

poodle - a person or organization who is overly willing to obey another.

underbelly - a hidden unpleasant or criminal part of society.

docility - the quality of being quiet and easy to influence, persuade, or control.)

63.(Answer - A. p r q m n o

foreman - chief.
onlooker - eyewitness.
harbour - port.)

64.(Answer - D. r p q m o n

surefooted - capable.
spook(n) - spy.

newfangled - untraditional or unconventional.


manifold - multiple.
intercontinental - global.)

65.(Answer - A. p r q m n o

decennium - a decade.)

Use Code NIMISHA for maximum Discount

For any query call or whatsapp 9953592800


222

66.(Answer - C. q r p o m n

militia - a military force that is raised from the civil population to supplement a regular army
in an emergency.

trawler - a fishing boat used for trawling.

squadron - fleet; naval force.)

67.(Answer - B. p r n o q m)

68.(Answer - C. a d g i

The correct arrangement is -

b c e f a d g i j h

69.(Answer - D. q p r n m o)

70. (Answer - B. n m q r o p

synergy - cooperation.
stoutness - stability.

grantee - recipient.
enterprise - initiative.)

71.(Answer - C. p o n m r q

retribution - punishment.
bequest - legacy.
outlay - investment.
desalination - the process of removing salt from seawater.
centrality - the quality of being essential or of the greatest importance.)

Use Code NIMISHA for maximum Discount

For any query call or whatsapp 9953592800


223

72.(Answer - D. o q m r n p

precept - principle.
standpoint - perception.
personnel - staff.
garnering - harvest.
plight - crisis.)

73. Ans. (d)


Sol. I-V and II-VI will be interchanged to make the sentence grammatically correct and
contextually meaningful. Hence, option (d) is the right answer choice.

74.Ans. (a)
Sol. II-VI and III-VII will be interchanged to make the sentence grammatically correct and
contextually meaningful. Hence, option (a) is the right answer choice.

75. Ans. (a)


Sol. I-V will be interchanged to make the sentence grammatically correct and contextually
meaningful. Hence, option (a) is the right answer choice.

76.Ans. (b)
Sol. III-VII and IV – VIII will be interchanged to make the sentence grammatically correct and
contextually meaningful. Hence, option (b) is the right answer choice.

77.Ans. (c)
Sol. III-VII will be interchanged to make the sentence grammatically correct and contextually
meaningful. Hence, option (c) is the right choice.

78.Ans. (a)
Sol. I –V and IV – VIII will be interchanged to make the sentence grammatically correct and
contextually meaningful. Hence, option (a) is the right answer choice.

79. Ans. (d)


Sol. I-V and II-VI will be interchanged to make the sentence grammatically correct and
contextually meaningful. Hence, option (d) is the right answer choice.

Use Code NIMISHA for maximum Discount

For any query call or whatsapp 9953592800


224

80. Ans. (e)


Sol. The given sentence is correct and the highlighted words are also correctly placed.
Hence, the right answer choice will be an option (e).

81.Ans. (b)
Sol. II-VI will be interchanged to make the sentence grammatically correct and contextually
meaningful. Hence, option (b) is the right answer choice.

82.Ans. (b)
Sol. III-VII and IV – VIII will be interchanged to make the sentence grammatically correct and
contextually meaningful. Hence, option (b) is the right answer choice.

83. Ans. (d)


Sol. I-V and II-VI will be interchanged to make the sentence grammatically correct and
contextually meaningful. Hence, option (d) is the right answer choice.

84.Ans. (a)
Sol. I –II and III-VII will be interchanged to make the sentence grammatically correct and
contextually meaningful. Hence, option (a) is the right answer choice.

85.Ans. (c)
Sol. III-VII and IV – VIII will be interchanged to make the sentence grammatically correct
and contextually meaningful. Hence, option (c) is the right answer choice.

86.Ans. (d)
Sol. I-V will be interchanged to make the sentence grammatically correct and contextually
meaningful. Hence, option (d) is the right answer choice.

87. Ans. (e)


Sol. Here, all the highlighted words are correct. Hence, option (e) is the right answer
choice.

88. Ans. (b)


Sol. III-VII and IV – VIII will be interchanged to make the sentence grammatically correct and
contextually meaningful. Hence, option (b) is the right answer choice.

Use Code NIMISHA for maximum Discount

For any query call or whatsapp 9953592800


225

89. Ans. (c)


Sol. III-VII will be interchanged to make the sentence grammatically correct and contextually
meaningful. Hence, option (c) is the right answer choice.

90. Ans. (a)


Sol. I–II and III-VII will be interchanged to make the sentence grammatically correct and
contextually meaningful. Hence, option (a) is the right answer choice.

91. Ans. (a)


Sol. I –II and IV – VIII will be interchanged to make the sentence grammatically correct and
contextually meaningful. Hence, option (a) is the right answer choice.

92.Ans. (d)
Sol. I-V and IV – VIII will be interchanged to make the sentence grammatically correct and
contextually meaningful. Hence, option (d) is the right answer choice.

93.A

Solution : Sledgehammer – powerful, forceful

94.C

Solution : Haughty: unfriendly and seeming to consider yourself better than other
people:

95.D

Solution : Callous: unkind, cruel, and without sympathy or feeling for other people.

Among the given words, “uncaring” is the correct meaning of the given highlighted word.

(Antonyms- Empathetic, Compassionate, Merciful)

Use Code NIMISHA for maximum Discount

For any query call or whatsapp 9953592800


226

CHAPTER 6 - PHRASE BASED QUESTIONS

Direction (1-30): In the following question, a part of the sentence is given in bold; it is
then followed by three sentences that try to explain the meaning of the phrase given
in bold. Choose the best set of alternatives from the five options given below each
question which explains the meaning of the phrase correctly without altering the
meaning of the sentence given to the question.

Q1. Play it by ear


(I) I don't have a set schedule, so we'll have to act according to the circumstances.
(II) That suggestion of yours might just become a reality—I do have the attention of CEO,
you know.
(III) ' I don't know what will happen next. I'm dealing with things as they happen.

(a)Only (I) is correct


(b)Only (II) is correct
(c) Only (III) is correct
(d)Both (I) and (III) are correct
Use Code NIMISHA for maximum Discount

For any query call or whatsapp 9953592800


227

(e)All are correct

Q2. Be up with the lark


(I) I've always loved camping: waking up at sunrise, cooking breakfast over an open fire,
exploring the great outdoors—it's all wonderful!
(II) I got up early in the morning, too excited at the prospect of seeing my team to sleep.
(III) I knew not handing in my homework would be a problem eventually—stuff like that
always causes problems.

(a)Both (I) and (II) is correct


(b)Only (II) is correct
(c)Both (I) and (III) are correct
(d)Only (III) is correct
(e)All are correct

Q3. Go for the jugular


(I) An internship might not sound very interesting, but it's a great way to get a chance to do
something at this company.
(II) Bob's request for a large salary increase was dismissed and ridiculed.
(III) If you want to attack aggressively against your opponent, I have some damaging
information you might be interested in.

(a)Only (I) is correct


(b) Both (III) and (I) are correct
(c)Both (I) and (II) are correct
(d)Only (III) is correct
(e)All are correct

Q4. Stir up a hornet’s nest


(I) The government's military interventions really just complicated the situation in the
region.
(II) The politician's off-the-cuff remark about pollution provoked offended reactions among
environmentalists.
(III) Anthony is in a troublesome situation with his creditors and if he doesn't pay off his
debts, they're going to take everything he owns.

Use Code NIMISHA for maximum Discount

For any query call or whatsapp 9953592800


228

(a)Only (III) is correct


(b)Only (II) is correct
(c)Both (I) and (II) are correct
(d)Only (I) is correct
(e)All are correct

Q5. Come hell and high water


(I) don't care if I have to drive through a blizzard—we are getting to this wedding in spite of
any obstacle.
(II) No matter what, we will have this project finished on time.
(III) I got into a lot of trouble when I was in high school, but I settled down a bit after I
graduated.

(a)Only (I) is correct


(b)Only (II) is correct
(c)Both (I) and (II) are correct
(d)Only (III) is correct
(e)All are correct

Q6. Make a beeline for something or someone


(I) Whenever I park in the city, the fear of being towed is always present in my mind.
(II) I knew the boss was angry, so when I saw her come in, I quickly headed towards the
break room.
(III) The boys headed for computer games while the girls went straight for the dolls.

(a)Only (I) is correct


(b)Both (II) and (III) is correct
(c)Both (I) and (II) are correct
(d)Only (III) is correct
(e)All are correct

Q7. Cry over spilt milk


(I) I didn't want to be upset and I didn't even accept the psychological help and therapy that
was offered to me.
(II) I told you not make a fuss about it, and here you are, worrying over what has
happened.

Use Code NIMISHA for maximum Discount

For any query call or whatsapp 9953592800


229

(III) You can’t just lament about losing the game, we just have to keep going and try to get
some players back to full fitness.

(a)Only (I) is correct


(b)Only (II) is correct
(c)Both (I) and (II) are correct
(d)Only (III) is correct
(e)All are correct

Q8. Bury the hatches


(I) Leading tech competitors settle their differences to improve energy efficiency.
(II) You have to speak frankly about it and acknowledge the corruption built into this
system.
(III) They need to calm down and make peace before someone gets hurt.

(a) Both (I) and (III) is correct


(b)Only (II) is correct
(c)Both (I) and (II) are correct
(d)Only (III) is correct
(e)All are correct

Q9. Vote with one’s feet


(I) It was heard from the rumours that the captain of the team was stepping down, but the
news could not be confirmed.
(II) After his inappropriate comment, we all registered our disapproval and just walked
away, leaving him standing there alone.
(III) People want to get approval through their presence and let their voice be heard, but
when it comes to all things European, they are not concerned.

(a)Both (I) and (II) is correct


(b)Both (II) and (III) is correct
(c)Both (I) and (II) are correct
(d)Only (III) is correct
(e)All are correct

Q10. To fish in troubled waters

Use Code NIMISHA for maximum Discount

For any query call or whatsapp 9953592800


230

(I) Frank is getting himself in a dangerous situation by buying more shares of that
company.
(II) We must be careful not to appear to be taking advantage of his problems.
(III) Things seem to be returning to their original position as politicians are beginning to
call for deregulation of the industry once again.

(a)Only (I) is correct


(b)Only (II) is correct
(c)Both (I) and (II) are correct
(d)Only (III) is correct
(e)All are correct

Q11. Gift of the gab


(I) Alexis really has the ability to speak persuasively, so she should be the one to address
the potential investors.
(II) Acting is not mere a manner to speak clearly and confidently, but a skill that comes
from a serious observation of life and all around.
(III) He was a pleasant little man with spiked hair, a black pipe, and a great skill to
persuade other people with his words.

(a)Only (II) is correct


(b)Only (I) is correct
(c)Both (I) and (II) are correct
(d) Both (II) and (III) are correct
(e)All are correct

Q12. Eat the humble pie


(I) He swore to me that he had left it back in my shed, but had to make a humble apology
when he discovered it was in his garage all the time.
(II) I think Ellen is a perfectionist because the thought of accepting defeat and criticism
terrifies her.
(III) It was a risk I was prepared to take, although if it didn't work out I knew I might have to
come back and accept humiliation.

(a)Only (I) is correct


(b)Only (II) is correct
(c)Both (I) and (II) are correct

Use Code NIMISHA for maximum Discount

For any query call or whatsapp 9953592800


231

(d)Both (III) and (I) is correct


(e)All are correct

Q13. Rest on one’s laurels


(I) I know your first novel was a smash success, but if you just stop putting in the efforts,
you're going to fade into obscurity.
(II) The company has stopped to innovate and advance their device design for the past
decade, and smaller companies are surpassing them with innovative stuff.
(III) It may be time-consuming to check the oil in your car every time you buy gasoline, but
you should be cautious rather than be sorry later.

(a)Only (III) is correct


(b)Only (I) and (II) is correct
(c)Both (III) and (II) are correct
(d) Both (I) and (III) are correct
(e)All are correct

Q14. Let the grass grow beneath one’s feet


(I) She'd known she was taking a risk in revealing the secret about Jacobs.
(II) I used to wait before doing something, and I missed out on a lot of opportunities.
(III) Catherine proved that she doesn't stand still and inactive as she won the Northern
Women's Championship.

(a) Both (I) and (III) are correct


(b) Both (III) and (II) are correct
(c) Both (I) and (II) are correct
(d)Only (III) is correct
(e)All are correct

Q15. Shape up or ship out


(I) I've warned you about turning up to work late for the last time—you need to improve
your behaviour or leave the job.
(II) He never used to support that political candidate, but he's changed his opinion all of a
sudden.

Use Code NIMISHA for maximum Discount

For any query call or whatsapp 9953592800


232

(III) I can't believe he went out on his motorcycle in this rain; his nonchalant attitude is
going to get him killed.

(a)Only (II) is correct


(b)Only (III) is correct
(c)Both (I) and (II) are correct
(d)Only (I) is correct
(e)All are correct

Q16. Finger on the pulse


(I) We must abstain from destructive activities so that our children can inherit a peaceful
world.
(II) Sara really has keen awareness of nightlife in the city, so I would ask her where you
should have your birthday party.
(III) Monica wouldn't be so busy and stressed if she didn't have involvement in several
activities.

(a)Only (II) is correct


(b)Only (III) is correct
(c)Both (I) and (II) are correct
(d)Only (I) is correct
(e)All are correct

Q17. A hill of beans


(I) Our hearts had a strong desire for travelling to Spain for the summer, until Lizzy broke
her leg and we had to cancel the trip.
(II) Even after so many years, the company is still comparable with the most profitable
businesses in the world.
(III) For all the congressmen's posturing about overhauling the tax system, his plan to do so
is of little importance.

(a) Both (I) and (III) are correct


(b) Both (III) and (II) are correct
(c) Both (I) and (II) are correct
(d)Only (III) is correct
(e)All are correct

Use Code NIMISHA for maximum Discount

For any query call or whatsapp 9953592800


233

Q18. Jump on the bandwagon


(I) There will always be people ready to suddenly become involved in an activity and
start classes in whatever is fashionable, with little or no training.
(II) The senator got the advantage by visiting the state three times before his opponent
managed to make an appearance.
(III) As soon as their policies became popular, all the other parties started to join them.

(a)Only (III) is correct


(b)Only (I) is correct
(c)Both (III) and (II) are correct
(d) Both (I) and (III) are correct
(e)All are correct

Q19. To keep one’s hair on


(I) My mother would always stay calm when she would argue with my father.
(II) You have to keep cool and maintain decorum when you are talking with such
important clients else we will lose a lot of business.
(III) If you really want to become a comic book artist, you have to keep trying to achieve it.

(a)Only (I) is correct


(b)Only (II) is correct
(c)Both (I) and (II) are correct
(d)Only (III) is correct
(e)All are correct

Q20. Knee jerk reaction


(I) In a spontaneous reaction after the big defeat, the coach dropped many of the players
and fielded a new look team for the next match.
(II) You had your chance, now let your brother have a chance at breaking the piñata.
(III) Her remark was probably an automatic response to your comments, which were not
very flattering.

(a)Both (I) and (III) are correct


(b)Both (II) and (III) are correct

Use Code NIMISHA for maximum Discount

For any query call or whatsapp 9953592800


234

(c)Both (I) and (II) are correct


(d)Only (III) is correct
(e)All are correct

Q21. Put the cart before the horse


(I) Don’t reverse the right method of it by investing in a new shop before selling that old
one situated in west of the city.
(II) I think you are doing it wrong by leaving your permanent job before getting new one.
(III) I'm afraid that since everyone knows about my recent diagnosis, I will be ruining
people’s enjoyment.

(a)Only (III) is correct


(b)Only (II) is correct
(c)Both (I) and (II) are correct
(d)Only (I) is correct
(e)All are correct

Q22. A cloud on the horizon


(I) Financial analysts believe that the sudden drop in oil prices points to a problematic
situation for the national economy.
(II) Although we are making good profits there is a sign of trouble — the government may
increase taxes.
(III) Unless you want to be a police suspect, you can't be seen with me, now that I have an
unfavourable relationship with you.

(a)Only (I) is correct


(b) Both (III) and (I) are correct
(c)Both (I) and (II) are correct
(d)Only (III) is correct
(e)All are correct

Q23. See eye to eye


(I) Some people think they can get away with anything and they always try to deceive
people.
(II) His mother and I don't agree with his decision to drop out of college.

Use Code NIMISHA for maximum Discount

For any query call or whatsapp 9953592800


235

(III) The ruling party and the opposition don’t have same views on most things.

(a)Both (II) and (III) are correct


(b)Only (II) is correct
(c)Both (I) and (II) are correct
(d)Only (III) is correct
(e)All are correct

Q24. Wear heart on sleeve


(I) She is a very sensitive person and openly reveals her feelings, so it very easy to hurt
her feelings.
(II) If you are open about your emotions, you will be very vulnerable and people are going
to take advantage of you.
(III) Jenny said she wasn't scared before we went into the haunted house, but terror was
evident from her face.

(a)Only (I) is correct


(b)Only (II) is correct
(c)Both (I) and (II) are correct
(d)Only (III) is correct
(e)All are correct

Q25. Wet behind the ears


(I) The heat and dust of this city is something different from the clean and cool climes of
the place where we used to live.
(II) Matthew’s inability to contribute to the topic at hand showed he was new and lacked
necessary experience when it comes to marketing.
(III) From the way the soldier handled the gun, everyone could tell that he was
inexperienced.

(a) Both (III) and (II) are correct


(b)Only (II) is correct
(c)Both (I) and (II) are correct
(d)Only (III) is correct
(e)All are correct

Use Code NIMISHA for maximum Discount

For any query call or whatsapp 9953592800


236

Q26. Stiff upper lip


(I) Throughout the funeral of her parents, Mary kept a control over her emotions.
(II) Once I learned that the trip was falling under my financial responsibility, I decided not
to go.
(III) It is hard to remain firm when everything is simply falling apart.
(a)Only (I) is correct
(b)Only (II) is correct
(c)Both (I) and (II) are correct
(d)Both (III) and I are correct
(e)All are correct

Q27. Knight in the shining armour


(I) The new law is just a misleading clue meant to draw our attention away from the issues
regarding education.
(II) John is like a well mannered man to Jane since he paid her credit card bills.
(III) Don’t try to be brave soldier you can’t save that drowning dog into the raging river.

(a)Only (I) is correct


(b)Only (II) is correct
(c)Both (I) and (III) are correct
(d)Only (III) is correct
(e)All are correct

Q28. Skating on thin ice


(I) Amanda decided to take a risk by quitting her day job to pursue her passion of wildlife
photography.
(II) Rather than engaging in a danger by investing in risky ventures, people should hire a
business and financial expert first.
(III) I feel their decision to get married after meeting just a week ago is like getting into
trouble.

(a)Only (I) is correct


(b)Only (II) is correct
(c)Both (I) and (II) are correct
(d)Only (III) is correct
(e)All are correct

Use Code NIMISHA for maximum Discount

For any query call or whatsapp 9953592800


237

Q29. Play second fiddle


(I) Martha is always in her sister’s shadow, playing subordinate role to her every time.
(II) The new boy band is distinct and determined not to be in a weak position to any other
group.
(III) After the nasty fight with her sister, Alice decided to improve her relationship by
buying her sister some presents.

(a)Only (I) is correct


(b)Only (II) is correct
(c)Both (I) and (II) are correct
(d)Only (III) is correct
(e)All are correct

Q30. Make a virtue of necessity


(I) Though I had to leave for my vacation too but I extracted something beneficial from it
and learned new technology needed for the project.
(II) The apartment they live in now is entirely different from the huge mansion they once
used to have.
(III) It’s was a long journey from Delhi to Australia so I thought I would use this situation in
a best possible way and write some articles that had to be submitted to the magazines.

(a)Only (I) is correct


(b)Only (II) is correct
(c) Only (III) is correct
(d)Both (I) and (III) are correct
(e)All are correct

ANSWER

1.Ans. (d)
Sol. Play it by ear: If you play it by ear, you deal with things as they happen, rather than
following a plan or previous arrangement.
-To act according to the circumstances; improvise
Hence, option (d) is the right answer choice.

2.Ans. (a)

Use Code NIMISHA for maximum Discount

For any query call or whatsapp 9953592800


238

Sol. Be up with the lark: To wake and get out of bed at sunrise.
- get up early in the morning:
Hence, option (a) is the right answer choice.

3.Ans. (d)
Sol. Go up for the jugular: o attack in the most aggressive way possible or where the victim
is most vulnerable. Despite the imagery, the term is almost never used to refer to physical
violence. (The jugular is a vein in the neck that transports blood between the head and the
heart, and is a common target for some predators.)
Hence, option (d) is the right answer choice.

4.Ans. (c)
Sol. Stir up a hornet’s nest: To create, provoke, or trigger a dangerous, troublesome, or
complicated situation.
-To provoke or instigate a lot of very angry or offended reactions.
Hence, option (c) is the right answer choice.

5.Ans. (a)
Sol. Come hell or high water: No matter what; in spite of any obstacle.
Hence, option (a) is the right answer choice.

6. Ans. (b)
Sol. Make a beeline for something or someone: If you make a beeline for something, you go
straight to it without any hesitation or delay.
- To head directly and quickly toward something or some place.

7. Ans. (e)
Sol. Cry over spilt milk: To be upset over something that cannot be fixed, often something
minor.
Don't be upset over something that cannot be fixed. Said especially of something trivial or
minor.
lament or make a fuss about a misfortune that has happened and that cannot be changed or
reversed.
Hence, option (e) is the right answer choice.

8. Ans. (a)
Sol. Bury the hatchet: To make peace with someone.
- settle one's differences

Use Code NIMISHA for maximum Discount

For any query call or whatsapp 9953592800


239

- When people who have argued bury the hatchet, they agree to forget their argument and
become friends again.
Hence, option (a) is the right answer choice.

9. Ans. (b)
Sol. Vote with one’s feet: To show one's approval or disapproval of something through one's
presence or absence, especially disapproval through leaving a place by walking out.
- Indicate one's disapproval by walking out or emigrating.
- To register disapproval by leaving.

10. Ans. (c)


Sol. To fish in troubled waters: to involve oneself in a difficult, confused, or dangerous
situation, especially with a view to gaining an advantage.
- If you fish in troubled waters, you try to get an advantage by getting involved in someone
else's problems.
Hence, option (c) is the right answer choice.

11.Ans. (e)
Sol. Gift of the gab: If someone has the gift of the gab, they are able to speak confidently,
clearly, and in a persuasive way.
- The ability to speak to others in a self-assured, persuasive manner.
- The ability to speak easily and to persuade other people with your words.
Hence, option (e) is the right answer choice.

12. Ans. (d)


Sol. Eat the humble pie: make a humble apology and accept humiliation.
Hence, option (d) is the right answer choice.

13. Ans. (b)


Sol. Rest on one’s laurels: To stop putting in effort, trying to innovate, or working to advance
one's career or status and instead rely on one's past achievements or accolades to remain
relevant or successful.
To stop trying because one is satisfied with one's past achievements.
Hence, option (b) is the right answer choice.

14.Ans. (b)
Sol. Let the grass grow beneath one’s feet: To be inactive; to do nothing or stand still. Often
used in the negative as an imperative ("don't let the grass grow beneath your feet").

Use Code NIMISHA for maximum Discount

For any query call or whatsapp 9953592800


240

- to wait before doing something or to do something slowly —used in negative statements


Hence, option (b) is the right answer choice.

15.Ans. (a)
Sol. Shape up or ship out: To either improve behaviour, performance, or attitude, or else (be
forced to) leave (a job, relationship, living situation, etc.).
Hence, option (a) is the right answer choice.

16. Ans. (a)


Sol. Finger on the pulse: A keen awareness of current trends and happenings.
Hence, option (a) is the right answer choice.

17. Ans. (d)


Sol. A hill of beans: Something of little or no importance, value, or worth.
Hence, option (d) is the right answer choice.

18. Ans. (d)


Sol. Jump on the bandwagon: To join or follow something once it is successful or popular.
- If someone jumps on the bandwagon, they suddenly become involved in an activity
because it is likely to succeed or it is fashionable.
- do something that others are already doing because it is successful or fashionable
Hence, option (d) is the right answer choice.

19. Ans. (c)


Sol. To keep one’s hair on: to request someone to stay calm and not get annoyed
to ask to maintain cool and decorum
to ask to not over react on a situation
Hence, option (c) is the right answer choice.

20. Ans. (a)


Sol. Knee jerk reaction:
an automatic response to something
an immediate reaction made without thinking
a reflex reaction
an instant reaction made without examining causes or facts
a spontaneous and involuntary reaction
Hence, option (a) is the right answer choice.

Use Code NIMISHA for maximum Discount

For any query call or whatsapp 9953592800


241

21.Ans. (c)
Sol. Put the cart before the horse: to do things in the wrong order or sequence
To do something in wrong manner
Reverse the right method of doing something
Hence, option (c) is the right answer choice.

22.Ans. (b)
Sol. A cloud on the horizon: An omen that something bad or problematic will happen in the
near future.
- a sign of trouble or difficulty to come
Hence, option (b) is the right answer choice.

23. Ans. (a)


Sol. See eye to eye: To agree fully with someone
Be in full agreement
To have the same views about something
Hence, option (a) is the right answer choice.

24.Ans. (c)
Sol. Wear heart on sleeve: display your emotions openly
Make your feelings apparent and obvious
Openly reveal your feelings and emotions
Unable to hide your emotions and feelings
Be emotionally transparent
Be open or forthright about your emotions.
Hence, option (c) is the right answer choice.

25.Ans. (a)
Sol. Wet behind the ears: immature or poor skill
To be inexperienced
To be new at something or somewhere and so lack the necessary experience
Hence, option (a) is the right answer choice.

26.Ans. (d)
Sol. Stiff upper lip: To be brave in the face of adversity
To keep your emotions in control and not show when you are upset
To remain firm in purpose and not let an unpleasant situation distract you

Use Code NIMISHA for maximum Discount

For any query call or whatsapp 9953592800


242

Hence, option (d) is the right answer choice.

27.Ans. (b)
Sol. Knight in the shining armour: used to describe someone who saves you from a difficult
situation
A well-mannered man who comes to save or help somebody from trouble
The term is used quite humorously as someone who comes to your aid
Hence, option (b) is the right answer choice.

28.Ans. (e)
Sol. Skating on thin ice: to do something considered quite dangerous or risky
To be in a situation that can get quite dangerous or risky
Engaged in some activity or behaviour that is very risky, dangerous, or likely to cause a lot
of trouble.
Hence, option (e) is the right answer choice.

29. Ans. (c)


Sol. Play second fiddle: to play a secondary or minor role with regards to someone else
To be regarded as less important or be in a weak position when compared to someone else
To be a subordinate to someone or take a backseat when it concerns that person
Hence, option (c) is the right answer choice.

30. Ans. (d)


Sol. Make a virtue of necessity: extract something beneficial from an unwelcome obligation
To shift the important deeds or act into a positive or useful experience
To use a difficult situation in the best possible way
To give the best possible effort one can under an uneasy situation
Hence, option (d) is the right answer choice.

Use Code NIMISHA for maximum Discount

For any query call or whatsapp 9953592800


243

CHAPTER 7 - READING COMPREHENSION

Direction(1-8): Read the given passage carefully and answer the questions that
follow.

Is India in a state of ‘undeclared emergency’? Several media organizations — the Press


Club of India, Editors Guild of India, Indian Women’s Press Corps, Indian Journalists Union
are among them — think this is, indeed, the case. Their concerns cannot be ___________.
‘Democratic’ India has developed a rash of brash and intimidating members of the
fourth estate, who still have the spine left to be objective and critical.

Legal provisions, some of which are legacies of colonial law, are being blatantly used to
impose a culture of silence. Consider the latest transgression of the slew of sedition cases
filed against several senior journalists as well as a prominent member of the opposition
across five states for their alleged ‘misleading’ tweets on the violence and the death of a
farmer during the disturbances on Republic Day. This particular incident reveals the dual
nature of the challenge that confronts Indian media today.

Use Code NIMISHA for maximum Discount

For any query call or whatsapp 9953592800


244

First, a partisan, competitive ecosystem and commercial (a)interpretations are forcing


journalists to express opinions that do not, on some (b)actualities, reflect ground
(c)imperatives - the faulty (d)occasions of the death of the farmer are a case in point.

The occasional slip of veracity has been exploited by a cynical administration to ______
serious charges, ranging from sedition to the endorsement of communal tensions, against
the accused.

Yet, the twisting of facts by a media fraternity, perceived to be the cheerleaders of the
present dispensation, is seldom scrutinized. This is unmitigated hypocrisy and intimidation.
The weaponization of sedition as well as of other draconian legislations can have only one
intent: the stifling of expression of independent, critical opinion not just by journalists but also
comedians and citizens. This warrants serious reflection on the part of institutions that claim
to be the custodians of democracy.

1. Which of the following phrasal verbs can contextually fit in the blank in the first
paragraph?

A. brushed down
B. brushed aside
C. laid aside
D. laid down
E. None of the above options

2. What does the author mean by the phrase '‘Democratic’ India has developed a rash of
brash and intimidating members of the fourth estate, who still have the spine left to
be objective and critical.' in the first paragraph?

I. There are a large number of blatantly bold members in the press, who can still be
characterised by careful evaluation, judgement and impartiality, in Democratic India.
II. Some members of the media who have the guts to be unbiased and judgemental are
still left in Democratic India.
III. Democratic India has plenty of dauntless journalists and media persons who still can
afford to be impartial and present undistorted facts, in Democratic India.

A. Only I
B. Both I and II

Use Code NIMISHA for maximum Discount

For any query call or whatsapp 9953592800


245

C. Only III
D. Both I and III
E. All I, II and III

3. In the third paragraph, four words in bold may or may not be correctly placed. Below are
some combinations of the correct sequence of those words, one of which will make the
whole sentence contextually correct. Choose the correct combination of those words. If no
replacement is required, choose option (E).

A. cdba
B. bdac
C. bcda
D. cbad
E. None of the above options

4. There is a blank and a word highlighted in bold in the fourth paragraph. Below are
some pairs of the word, that'll fit in the blank and the meaning of the highlighted word
respectively. Choose the correct pair.

A. level, language or behaviour that is intended to offend other people.


B. slap, incitement of discontent or rebellion against a government.
C. strike, conduct or speech inciting people to rebel against the authority of a monarch.
D. put, conduct, such as speech and organization, that tends to act against dutifulness
of the citizens.
E. None of the above pairs

5. What is the 'dual nature of the challenge' as mentioned in the second paragraph,
according to the author?

I. Journalists are being compelled to distort the truth sometimes and consequently their
this mistake exposed them to serious accusations by the derisive administration.
II. The present rivalrous nature of the press media obligates the journalist to hide the
true facts and this wrongdoing of them, is working in favour of the disrespectful
administration.

Use Code NIMISHA for maximum Discount

For any query call or whatsapp 9953592800


246

III. For the sake of surviving the aggressive and competing world of media, journalists
are working against their ethics and distorting facts, which is then helping the
pessimistic administration to bring accusations against the oppressors.

A. Only III
B. Both I and III
C. Only I
D. Only II
E. Both II and III

6. What does the author mean by the phrase 'cheerleaders of the present dispensation'
in the last paragraph?

A. supporters of the current chaos


B. admirers of the existing situation
C. protagonists of the contemporary government
D. followers of the present-day organisation.
E. None of the above options

7. Which of the following statements can be inferred from the last paragraph?

I. Government's unnecessarily and extremely severe laws are meant to minimise the
freedom of speech of individuals and suppress the voices of dissent.
II. The fact that media houses do distort the real facts, is rarely looked at, critically.
III. Sedition charges are the weapons of the government, which display an utterly
uncontrolled pretension.

A. Only III
B. Both I and II
C. Both II and III
D. Both I and III
E. All I, II and III

8. What's the actual tone of the author in the last line of the last paragraph?

A. Admonitory
B. Reprimanding
C. Distressing

Use Code NIMISHA for maximum Discount

For any query call or whatsapp 9953592800


247

D. Teasing
E. None of the above options

Direction(9-17): Read the given passage carefully and answer the questions that
follow.

In 2016, the Supreme Court had ruled that access to justice is a fundamental right. But the
universality of justice does not necessarily translate to its even dispensation. This
inference can be drawn from the findings of the India Justice Report 2020, a comprehensive
study of the prevalent nature and reach of the legal system. The report’s examination of
seven small and 18 major states has not yielded upbeat results. Not even the best-
performing states scored above 60% in terms of justice delivery across four crucial
institutions — the police, the judiciary, prisons and legal aid. That glaring lacunae in these
critical spheres exist even in the best-performing state, indicates the challenges that
confront India’s justice delivery mechanism.

The reasons behind the bottlenecks are manifold. One is the appalling number of
vacancies in judges’ posts. India has one judge for every 50,000 citizens; Bengal’s lower
courts have the highest number of pending cases for over a five-year period. In spite of the
presence of undertrial review committees, India’s prison occupancy rate stood at a
shocking 119 per cent at the end of 2019; two-thirds of the inmates are undertrial
detainees. Although 80 per cent of the Indian population is entitled to legal aid, a paltry 1.5
crore people have received it in the last 25 years.

Some of the other contradictions are illustrative of several deepening chasms. Maharashtra
topped states in providing legal aid, but witnessed a decline in the already-meagre number
of women police officers. Indeed, the representation of women across institutions has not
been proportionate to scale and often registers a growth in the lower ranks only.

The dispensing of justice is not merely about (p)precedents: it must also take into
consideration what constitutes the idea of justice. Problematic (q)overreach of the law and
perceived governmental (r)verdicts are setting worrying (s)readings. The recent
controversial judgments by a bench of the Bombay High Court on sexual offences against
minors as well as the incident of cases of communal violence being dropped by Karnataka’s
courts last year may lead to public confusion and even disenchantment.
The institutional deficits prevailing within the justice delivery system must be addressed. But
that will not be all. It is crucial for its stakeholders — the police, judges and the State — to
reinstate citizens’ faith in the pledge to uphold justice fairly and quickly for all.

Use Code NIMISHA for maximum Discount

For any query call or whatsapp 9953592800


248

9. What does the author mean by the phrase 'But the universality of justice does not
necessarily translate to its even dispensation.' in the first paragraph?

I. The fact, justice for all, doesn't always guarantee the even distribution of justice.
II. Justice is indeed for everyone, but it's often not served with the assurance of
credibility.
III. Not everyone gets to enjoy the fruit of justice notwithstanding its all-inclusive nature.

A. Only III
B. Both I and III
C. Both II and III
D. Only II
E. Only I

10. What does the author indicate by the term 'glaring lacunae', according to the first
paragraph?

I. The fact that justice isn't being served properly in terms of the police, the judicature,
prisons and legal assistance, even in the best performing states.
II. The fact that some of the small and major states are being faced with difficulties in
delivering justice.
III. The fact that even in the best-performing states, gaps exist in the justice delivery
system, which are the predicaments India’s justice delivery mechanism faces.

A. Only III
B. Both II and III
C. Only II
D. Both I and III
E. All I, II and III

11. What's the literal meaning of the word 'bottlenecks' as used in the second
paragraph?

A. Problems that delay progress


B. Problems that are never-ending.
C. Problems that are difficult to solve.
D. Problems that consist of many layers.
E. None of the above options

Use Code NIMISHA for maximum Discount

For any query call or whatsapp 9953592800


249

12. Which of the following statements is FALSE according to the second paragraph?

I. Two-thirds of the convicts in Indian prisons are being held in custody awaiting trial for
a crime.
II. 1.5 crore people have got legal support in the last 25 years and this number is not at
all negligible.
III. Lower courts in Bengal have the maximal number of unsettled cases, due to the fact
there is one judge for every 50,000 citizens there.

A. Only III
B. Both I and III
C. Both I and II
D. Only II
E. Both II and III

13. What does the author refer to by the term 'deepening chasms' in the third paragraph?

I. Portrayal of women all over institutions being incommensurate.


II. Diminution in the number of women police officers.
III. Growth in the number of women in only the lower ranks across institutions.

A. Only II
B. Both I and III
C. Both II and III
D. Only III
E. All I, II and III

14. In the fourth paragraph, four words in bold may or may not be correctly placed. Below
are some combinations of the correct sequence of those words, one of which will make
those sentences contextually correct. Choose the correct combination of those words. If no
replacement is required, choose option (E).

A. r s q p
B. q p r s
C. p s q r
D. q r p s
E. No replacement required

Use Code NIMISHA for maximum Discount

For any query call or whatsapp 9953592800


250

15. Choose an appropriate synonym for the word 'disenchantment' as used in the fourth
paragraph.

A. disillusionment
B. panic
C. discord
D. regret
E. None of the above options

16. What can be inferred from the fourth paragraph?

I. Judgements are not the only parameters of providing justice; the concept of justice
must be given thought to, too.
II. In the process of giving justice, questionable interpretation of laws and government's
undue interference are the things that set distressing examples and as a result, there
may be bafflement and discontent in people.
III. People expect the verdicts to come in a righteous way; but several factors like undue
governmental intervention and misinterpretations of the laws often influence the
verdicts which shatter people's hopes as the verdicts are not the ones they hoped for.

A. Both I and III


B. Only II
C. Both II and III
D. Both I and II
E. All I, II and III

17. What has the author suggested for the fair and quick delivery of justice?

I. The lacunae in the institutional level must be contended with.


II. The entities involved in the justice delivery system must reestablish people's faith in
its credibility.
III. The idea of justice amongst people must be made clear.

A. Only III
B. Both I and II
C. Only II
D. Both II and III
E. All I, II and III

Use Code NIMISHA for maximum Discount

For any query call or whatsapp 9953592800


251

Direction(18-27): Read the given passage carefully and answer the questions that
follow.

The powers that be protest too much. This is an apt description of the pushback
orchestrated by the external affairs ministry in response to a tweet by a popular international
singer on the ongoing farmers’ protest.
In an unprecedented step, the ministry of external affairs responded to the growing
international scrutiny in a lengthy statement that urged celebrities to ascertain facts before
rushing to make critical comments. This is welcome advice, but the dictum should be
applicable to all. The ministry itself, it appears, has taken certain licences with facts, stating
that Parliament had passed the contentious farm laws after “full debate and discussion”.
India and perhaps the world know that the legislations were steamrolled ignoring the
reservations of the Opposition. What is equally illustrative about the indignation of the
ministry and its admirers — celebrities all — was their comprehension of sovereignty.
The sovereignty of a robust democracy cannot be so brittle as to crumble in the face of a
single tweet. This prickliness, a dangerous kind of intolerance for criticism, has become a
signature of New India and its administration. The allergy is ________ to the health of this
democracy. What strengthens democracy is — not unilateral opinion but — a nation’s ability
to accommodate divergent, competing views.
There may be other reasons that explain the paranoid rebuttal. Could it be that the Centre is
no longer certain that its carefully curated narrative on the farmers’ protest is having its
desired effect? If that is the case, then the Narendra Modi government has itself to blame.
The sustained (m)impassivity of farmers, the periodic use of force and (n)echelons at the
sites of protest and, above all, the (o)demonization shown from the highest (p)incitation of
power have bared the ugly truths that no amount of cheerleading on television or on social
media can conceal.

18. What's the actual meaning of the phrase 'The powers that be protest too much.' in
the first paragraph?

I. The people in charge utter words of protest too much.


II. Words of dissent do come with a great amount of power.
III. The authorities do express opposition too much.
A. Only III
B. Both I and III
C. Only II
D. Only I

Use Code NIMISHA for maximum Discount

For any query call or whatsapp 9953592800


252

E. None of I, II and III

19. Which of the following statements is/are FALSE according to the first paragraph?

I. The celebrities have been recommended by the ministry of external affairs, to refrain
from making hasty comments that are intended to be fault-finding, unless they've
learnt the facts.
II. The fact that, the laws enacted by the legislative body, were made sure forcefully,
disregarding the objections of the Opposition, is a well-known fact.
III. Taking into account the full debate and discourse, the Parliament passed the
controversial farm laws.

A. Both I and II
B. Only II
C. Both II and III
D. Both I and III
E. None of the above options

20. What does the author mean by the line 'What is equally illustrative about the
indignation of the ministry and its admirers — celebrities all — was their
comprehension of sovereignty.' in the first paragraph?

I. From the discontent amongst the ministry and its admirers, their idea of supreme
power can be interpreted.
II. The ministry and its supporters do agree with each other on the very understanding of
administrative power.
III. The interpretation of supremacy by the ministry and its enthusiasts can be
demonstrated by the anger and distress among them.

A. Both I and III


B. Only II
C. Both II and III
D. Only III
E. All I, II and III

21. What does the author refer to by the term 'dictum' in the first paragraph?

A. The law, established by the ministry of external affairs.

Use Code NIMISHA for maximum Discount

For any query call or whatsapp 9953592800


253

B. The discipline, recommended by the ministry of external affairs.


C. The rule, set by the ministry of external affairs.
D. The formal pronouncement from the ministry of external affairs.
E. None of the above options

22. Which of the following words can most appropriately fit in the blank in the second
paragraph?

A. inimical
B. symbolical
C. well-disposed
D. equivocal
E. None of the above options

23. Which of the following inferences can be drawn from the second paragraph?

I. A healthy democracy should be open and responsive to ideas, impressions, or


suggestions, that are not one-sided, but rather assorted and challenging.
II. New India's tendency to show violent resistance to protesting tweets, is a sign of a
weak democracy.
III. If one single tweet can make India’s democracy fall apart then its administration can
be considered as a frail administration.

A. Both I and III


B. Both I and II
C. Only I
D. Only II
E. All I, II and III

24. What possibly is causing the government's worried denial, according to the author, in
the third paragraph?

I. Government's doubt on the effectiveness of the way it carefully and thoughtfully


presented farmers' protests.
II. Government's hesitation on establishing its masterminded response to the farmers'
protests.
III. Government's inability to provide certainty in response to the farmers' protests by
organizing a proper response.

Use Code NIMISHA for maximum Discount

For any query call or whatsapp 9953592800


254

A. Only II
B. Both I and II
C. Only III
D. Only I
E. Both II and III

25. In the last paragraph, four words in bold may or may not be correctly placed. Below are
some combinations of the correct sequence of those words, one of which will make those
sentences contextually correct. Choose the correct combination of those words. If no
replacement is required, choose option (E).

A. n m o p
B. p n m o
C. m p n o
D. o n p m
E. None of the above options

26. Which of the following words is an antonym of the word 'conceal', as used in the last
paragraph?

A. secrete
B. bury
C. ensconce
D. bare
E. None of the above options

27. What's the actual tone of the author in the last paragraph?
A. Censorious
B. Vituperative
C. Approbative
D. Plausive
E. None of the above options

Direction(28-31): Read the given passage carefully and answer the questions that
follow.

Use Code NIMISHA for maximum Discount

For any query call or whatsapp 9953592800


255

Justices K M Joseph and Hrishikesh Roy of the Supreme Court have rightly flagged what
has become a disquieting routine on the airwaves not only at prime time but almost all the
time: Talking heads spewing hate, nutcases passed off as experts as long as they can froth
on sensitive issues, each one locked in a race to the bottom, and all this while, anchors
provoking, cheering from the studio. Of course, switching off the TV is the best answer to
this trash but the bench has a point when it suggests this erodes the social compact,
deepens the divide.

Unfortunately, the court’s prescription is problematic. Not only does it overlook the structural
forces that enable this hate speech ecosystem, its solution could become more of a
problem. Hearing a bunch of petitions that seeks directions from the Court to the
government to curb hate speeches on TV channels, the judges have proposed that
guidelines along the lines of Vishaka, the SC judgment on sexual harassment at workplace,
can be put in place until the state brings in a law to regulate hate content on television. The
anguish of the court is understandable but its proposal is fraught. Hate speech has to be
read within the purview of Article 19, which safeguards the freedom of speech.

In numerous cases, the Supreme Court has upheld the primacy of Article 19 and warned
against state overreach (Romesh Thappar vs. The State of Madras,1950 and Shreya
Singhal vs. Union of India, 2015 among others). It has also defined what constitutes hate
speech. Any overarching law or guidelines to regulate speech stands the risk of violating the
letter and spirit of Article 19. In a polarised discourse marked by imbalances of power, who
will define hate speech?

The court said the “visual media” in India is the “chief medium of hate speech” and felt that
the government was “standing by as a mute witness when all this is happening”. The
silence, in many cases, is strategic and deliberate. That’s the nub.

At the heart of the problem is the political economy of TV news which thrives on hate speech
today more than ever. Justice Roy minced few words: “Hate drives TRPs, drives profit.” And
when politics fuels, legitimises that hate, judicial interventions are unlikely to work. A new
law on hate speech, as the court has suggested, runs the risk of being challenged — and
violated — every second given the ceaseless cycle of news and social media. Try enforcing
guidelines on what is hate speech or what is not on Twitter or YouTube. Indeed, there is a
formidable record of evidence to show how several IPC provisions meant to check hate
speech — including Section 153 (A) or 295 (A) — end up being weaponised by the state
and its agencies to curb dissent. The court must judge every instance of hate speech in its
own context. Nothing is a stronger deterrent against hate speech than those in power

Use Code NIMISHA for maximum Discount

For any query call or whatsapp 9953592800


256

speaking up against it, calling it out every time, without fail. Not much will be achieved by
lecturing TV channels on the power of love, when they have discovered the profit in hate.

28. The author has said - The silence, in many cases, is strategic and deliberate.
That’s the nub.
Which of the following statements can justify this?

a. People who actually experience ‘hate speech’ understand it as systemic; it builds up


slowly but steadily, and has a cumulative effect on the way a community begins to be
commonly understood.
b. The leaching of the poison in the form of hate speech would be rendered effective if
the government keeps mum and doesn't protest against it.
c. If we take any single ‘hate speech’, it doesn’t necessarily cover all presumed
grievances against one particular community.

A. Only a B. Only a and b C. Only b D. Only c E. All a, b and c

29. Why possibly does the author think that at the heart of the problem is the political
economy of TV news which thrives on hate speech today more than ever?

a. There is little cost to pay for hate speech, there are few incentives for TV at prime
time to be fair and accurate.
b. Indeed, most anchors are paid employees of their channels and they know they can
get away with peddling hate because someone in the boardroom has taken a call in
its favour.
c. Returning to TRPs, it's found that these ratings are important mainly to those who
pump money into commercial advertisements on TV as they need to ascertain who
and how many are likely to be watching the various channels.

A. Only a and b B. Only b C. Only b and c D. All a, b and c E. None of a, b and c

30. If the author is asked - What constitutes hate speech?


Which of the following will be his answer(s)?

Use Code NIMISHA for maximum Discount

For any query call or whatsapp 9953592800


257

a. The tendency towards hate mongering is mired in the oldest, most archaic ‘bullying
for power’.
b. Hate speech is in itself violent in its persistent stigmatising and calls towards
inclusion.
c. It contains expressions that expose a certain group to hatred, also, hate speech
seeks to delegitimize group members in the eyes of the majority, reducing their social
standing and acceptance within society.

A. Only a and b B. Only a and c C. Only b and c D. All a, b and c E. None of a, b


and c

31. Hate speech has to be read within the purview of Article 19, which safeguards the
freedom of speech.

Which of the following statements can be made from the above statement?

a. Hate speech is considered a reasonable restriction on freedom of speech and


expression.
b. Under article 19, the constitution provides for reasonable restrictions against
derogatory speech in the interests of sovereignty and integrity of India.
c. Hate speech is the offence of promoting communal disharmony or feelings of hatred
between different religious, racial, language or regional groups or castes or
communities.

A. Only a and b B. Only a and c C. Only b and c D. All a, b and c E. None of a, b


and c

Direction(32-33): Read the given passage carefully and answer the questions that
follow.

It has been almost 30 years since I petitioned the SC regarding the criminal negligence of
the CBI in a militancy and corruption related matter. The case is popularly known as the Jain
Hawala case. My expectation was that under the monitoring of the apex court, the CBI, DRI
and income tax agencies would do their duty, ensure proper investigations and take the
case to its logical conclusion. It was a difficult fight against the 115 most powerful people in

Use Code NIMISHA for maximum Discount

For any query call or whatsapp 9953592800


258

the country, who had allegedly taken money in a clandestine manner from the same source,
which was funding Hizbul Mujahideen in J&K. Due to the initiative of the SC the matter did
progress to some extent but for various reasons, could not reach its logical conclusion.

In 2013, the SC once again made strong observations against the functioning of the CBI and
referring to the “Vineet Narain” judgment, said that nothing had improved since 1997, when
this judgment was delivered. Encouraged by this remark, I again petitioned the SC,
reminding them that 22 names mentioned in the Jain Diary had not been
decoded/deciphered by the CBI till that date.

Despite the hype around the Jain Hawala case, the CBI acted in an irresponsible manner.
During the course of that case, I had filed two affidavits on April 8, 1995 and January 9,
1996, in which I had given the numbers of the case files that were not presented by the CBI
to the apex court. These files would have exposed the chronology of the suppression of this
case and those in the CBI responsible for the said suppression. The evidence regarding the
dilution of investigation and how the beneficiaries mentioned in the diary were spared is
available but not a single delinquent CBI official, responsible for the inaction from June 1991
to March 1995, was even asked to explain his conduct. Despite the intervention of the SC,
the CBI kept the investigation in cold storage for more than 40 months and all the accused
persons were spared.

32. Which of the following statements can be inferred from the first paragraph,
according to the author's idea of the CBI?

a. CBI was no more a "caged parrot" but was truly performing its duty.
b. CBI, the country's premier institution, faced the risk of losing its credibility.
c. CBI has done little or nothing to prove the perception that it's a caged parrot wrong.

A. Only a and b B. Only a and c C. Only b and c D. All a, b and c E. None of a, b


and c

33. Which of the following questions can arise after going through the second
paragraph?

a. Who knows whether these names were hardened militants, narcotic operators or anti-
national people?
b. Doesn’t it prove that there was criminal neglect on the part of the CBI?

Use Code NIMISHA for maximum Discount

For any query call or whatsapp 9953592800


259

c. Isn't it very unfair to consider CBI's action as negligent and insincere?

A. Only a and b B. Only a and c C. Only b and c D. All a, b and c E. None of a, b


and c

ANSWER

1.(Answer - B. brushed aside


brush aside - to treat (something) as not important : to ignore or dismiss (something).
The context has justified the concerns of the media organisations later in the
passage. That's why, their concerns can't be ignored or dismissed.
brush down - to clean someone or something using your hands or a brush.
lay aside - to keep (something) for special or future use : to reserve or save (something).
lay down - i) to put away or stop using something.
ii) to state something plainly esp. as an official rule.)

2.(Answer - D. Both I and III


'‘Democratic’ India has developed a new rash of brash and intimidating members of
the fourth estate, who still have the spine left to be objective and critical.'
a rash of - a large number of.
brash - blatantly bold and brave.
fourth estate - it refers to the press and news media or the public press.
The three branches of government -
Legislative, Executive and Judicial. The fourth one is the press.
have the spine - have the guts.
objective - impartial or unbiased.
critical - judgmental.
I - Correct. 'characterised by careful evaluation, judgement and impartiality' means - critical
and objective.
II - Incorrect. It has a different sense. It says, some brave members of the press are still left.
But, according to the context, a large number of members are in the media, who still have
the guts or courage to be unbiased.
III - Correct.
plenty of - many.
dauntless - brave.
present undistorted facts - being objective and unbiased.
undistorted - something that has not been changed from its usual or original form.)

Use Code NIMISHA for maximum Discount

For any query call or whatsapp 9953592800


260

3.(Answer - A. c d b a
First, a partisan, competitive ecosystem and commercial imperatives are forcing journalists
to express opinions that do not, on some occasions, reflect ground actualities - the faulty
interpretations of the death of the farmer are a case in point.
imperative - an essential or urgent thing.
actuality - reality.
interpretation - understanding or conclusion.
on some occasions - sometimes, but not often.
partisan - biased or discriminatory.
a case in point - an example of the situation or behaviour that you are talking about.)

4. (Answer - B. slap, incitement of discontent or rebellion against a government.


slap - put or apply (something) somewhere quickly or forcefully.
'slap serious charges' is absolutely correct.
sedition - incitement or provocation of discontent(dissatisfaction) or rebellion(civil
disobedience) against a government or against the authority of a monarch(ruler).
A - Incorrect. 'level' is correct. But, the meaning is wrong, as it's talking about offending
other people.
C - Incorrect. 'strike' is an incorrect verb here. But, the meaning is correct.
D - Incorrect. 'put' is correct. But, the meaning is wrong, as it's talking about acting against
the dutifulness of the citizens.
dutifulness - obedience by virtue of devotion to duty.)

5.(Answer - C. Only I
'The dual nature of the challenge' has been expressed by the author in two scenarios.
The first nature is -
Refer to -
First, a partisan, competitive ecosystem and commercial imperatives are forcing journalists
to express opinions that do not, on some occasions, reflect ground
actualities - the faulty interpretations of the death of the farmer are a case in point.
competitive ecosystem and commercial imperatives - media's aggressive nature and
competitive needs.
express opinions that do not, on some occasions, reflect ground actualities - express
distorted facts/hide the true facts.
And, the second nature is -
Refer to -

Use Code NIMISHA for maximum Discount

For any query call or whatsapp 9953592800


261

The occasional slip of veracity has been exploited by a cynical administration to slap serious
charges, ranging from sedition to the endorsement of communal tensions, against the
accused.
The occasional slip of veracity - the occasional failure to present the truth or remain
truthful.
veracity - truthfulness.
So, journalist's failure to stay truthful is exploited by the cynical government to bring serious
accusations against them.
I - Correct.
accusation - charge.
derisive - cynical.
II - Incorrect. It's not clear. 'working in favour of the disrespectful administration' is
ambiguous. It hasn't specified anything particular.
wrongdoing - mistake.
III - Incorrect. It says, the accusations are brought against the oppressors. But, the
paragraph says, the accusations are brought against the accused, who are the journalists
here.
oppressor means dictators.)

6.(Answer - C. protagonists of the contemporary government


protagonist - supporter or flagbearer.
contemporary - present-day or existing.
cheerleader - an enthusiastic and vocal supporter of someone or something.
dispensation - a political, religious, or social system prevailing at a particular time;
government.
D - looks correct but 'organisation' is nonspecific.)

7.(Answer - B. Both I and II


I - Refer to - The weaponization of sedition as well as of other draconian legislations can
have only one intent: the stifling of expression of independent, critical opinion not just by
journalists but also comedians and citizens.
stifle - to suppress or to silence.
draconian legislation - unnecessarily and extremely severe laws.
II - Refer to - Yet, the twisting of facts by a media fraternity, perceived to be the
cheerleaders of the present dispensation, is seldom scrutinized.
twisting of facts - distorting the real facts.
seldom - rarely.
scrutinize - look at carefully.

Use Code NIMISHA for maximum Discount

For any query call or whatsapp 9953592800


262

III - Incorrect. According to the context, the fact that the government rarely scrutinizes
media's wrongdoings, is hypocritical, not the sedition charges.
pretension - hypocrisy.)

8.(Answer - B. Reprimanding
Reprimand - censuring or criticizing.
This warrants serious reflection on the part of institutions that claim to be the custodians of
democracy.
It means - This(stifling of expression of independent, critical opinion not just by journalists
but also comedians and citizens) provides adequate grounds to justify the fact that, the
part of institutions that claim to be the protectors(custodians) of democracy, need to reflect
on themselves, because by suppressing voices, they're not protecting democracy;
they're destroying it.
Admonitory - warning.
Distressing - causing anxiety.
Teasing - mocking or making fun of.)

9.(Answer - B. Only I and II


'the universality of justice' means - justice for all; the all-inclusive(including all) nature of
justice.
dispensation - Here it's been used an uncountable noun.
It means - issuing of something, especially from a position of authority.
even dispensation - even distribution.
I - Correct. It's talking about exactly the same thing.
III - Correct. 'enjoy the fruit of justice' means - 'get the good results that justice comes
with.' In another way - not everyone is evenly/equally served by justice.
II - Incorrect. It's talking about the assurance of credibility or believability.)

10.(Answer - D. Both I and III


glaring lacunae - obvious gaps.
(lacunae is the plural form of lacuna which means gap.)
Refer to - Not even the best-performing states scored above 60% in terms of justice
delivery across four crucial institutions — the police, the judiciary, prisons and legal
aid. That glaring lacunae in these critical spheres exist even in the best-performing state,
indicates the challenges that confront India’s justice delivery mechanism.
I - Correct. It's talking about exactly the same thing.
judicature - judiciary.
aid - assistance.

Use Code NIMISHA for maximum Discount

For any query call or whatsapp 9953592800


263

III - Correct. Refer to - That glaring lacunae in these critical spheres exist even in the best-
performing state, indicates the challenges that confront India’s justice delivery
mechanism.
The author has called the glaring lacunae as the challenges that confront India’s justice
delivery mechanism.
predicament - challenge or difficulty.
confront - face.
II - Incorrect. It's nowhere written in the context that small and major states are being faced
with difficulties in delivering justice. It's written that these statements have performed
poorly in justice delivery.)

11.(Answer - A. Problems that delay progress.


Even if one doesn't know the literal meaning of this word, he/she can guess the meaning by
reading the subsequent lines. The author is saying that the reasons behind the bottlenecks
are many(manifold) and one of the reasons is the shocking(appalling) number of
vacancies in judges' posts. That means, due to the low number of judges the justice
process is being delayed.)

12.(Answer - E. Both II and III


II - False. 1.5 crore people have got legal support in the last 25 years and this number is
negligible.
Refer to - 'Although 80 per cent of the Indian population is entitled to legal aid, a paltry 1.5
crore people have received it in the last 25 years.'
a paltry 1.5 crore - a meagre/negligible 1.5 crore.
III - False. According to the context, there is one judge for every 50,000 citizens in India,
not in Bengal.
Refer to - 'India has one judge for every 50,000 citizens; Bengal’s lower courts have the
highest number of pending cases for over a five-year period.'
maximal - maximum/highest.
I - True. Refer to - 'two-thirds of the inmates are undertrial detainees.'
convicts being held in custody awaiting trial for a crime - undertrial detainees.
detainee - a person held in custody.)

13.(Answer - E. All I, II and III


chasm - a marked division, separation, or difference.
deepening - becoming darker.
The author is talking about the differences between the involvements of men and women
across institutions. Maharashtra witnessed a diminution(decline) in the number of women
police officers. Women are not portrayed or represented equally as men are and often

Use Code NIMISHA for maximum Discount

For any query call or whatsapp 9953592800


264

we get to see a growth in the lower ranks only. (Indeed, the representation of women
across institutions has not been proportionate to scale and often registers a growth in
the lower ranks only.…)
Statement I, II and III - all are stating that difference.
has not been proportionate to scale - has not been equivalent.
incommensurate - disproportionate.)

14.(Answer - A. r s p q
The dispensing of justice is not merely about verdicts: it must also take into consideration
what constitutes the idea of justice. Problematic readings of the law and perceived
governmental overreach are setting worrying precedents.
verdict - judgement.
reading - a particular interpretation.
overreach - the act of doing more than your authority allows.
precedent - model; example; standard.

15.(Answer - A. disillusionment

disillusionment/disenchantment - a feeling of disappointment resulting from the discovery


that something is not as good as one believed it to be.

discord - lack of agreement.)

16.(Answer - E. All I, II and III

Refer to - The dispensing of justice is not merely about verdicts: it must also take into
consideration what constitutes the idea of justice. Problematic readings of the law and
perceived governmental overreach are setting worrying precedents.

dispensing of justice - providing justice.


must be given thought to - must be taken into consideration.

problematic readings - questionable interpretations.


governmental overreach - government's undue or unauthorised intervention or
interference.

worrying precedents - distressing examples.

Use Code NIMISHA for maximum Discount

For any query call or whatsapp 9953592800


265

In this context, distressing examples may often lead to public confusion and
discontent. The paragraph also gave some examples of such unfortunate events.

III - The paragraph has talked about public disenchantment or disillusionment. People are
disillusioned when they hope for something, but get something else.
This statement is talking about exactly the same thing.)

17.(Answer - B. Both I and II

I - Refer to - 'The institutional deficits prevailing within the justice delivery system must be
addressed.'

addressed / contended with - dealt with.


lacunae - deficits.

II - Refer to - 'But that will not be all. It is crucial for its stakeholders — the police, judges and
the State — to reinstate citizens’ faith in the pledge to uphold justice fairly and quickly for all.'

entities involved in the justice delivery system - stakeholders of the justice delivery
system.

reinstate - reestablish.
credibility - believability or reliability.

III - No such thing has been mentioned in the passage.)

18.(Answer - B. Both I and III

The powers that be - It's a phrase. It means - important people who have authority over
others or the authorities or the administration or the people in charge.

to utter words of protest / to express opposition - to protest.

words of dissent - protests.)

18.(Answer - E. None of the above options

Use Code NIMISHA for maximum Discount

For any query call or whatsapp 9953592800


266

Only III is FALSE.

Refer to - The ministry itself, it appears, has taken certain licences with facts, stating that
Parliament had passed the contentious farm laws after “full debate and discussion”.
It is the ministry that claims that the Parliament passed the controversial farm laws, taking
into account the full debate and discourse. It's just a statement. In reality it's not true..

contentious - controversial.

I - TRUE. Refer to - In an unprecedented step, the ministry of external affairs responded to


the growing international scrutiny in a lengthy statement that urged celebrities to ascertain
facts before rushing to make critical comments.
ascertain - to learn or to confirm.

rushing to make critical comments - making hasty comments that are intended to be
fault-finding.

refrain from - stop oneself from doing something.

II - TRUE. Refer to - India and perhaps the world know that the legislations were steamrolled
ignoring the reservations of the Opposition.

legislation - laws enacted by the legislative body.


steamroll - to make sure by using all power/force.
reservation - here it means - objection.)

20.(Answer - A. Both I and III

'What is equally illustrative about the indignation of the ministry and its admirers —
celebrities all — was their comprehension of sovereignty.'

illustrative - interpretative or explaining or demonstrative.


indignation - anger; discontent; distress.
comprehension of sovereignty - idea/understanding/interpretation of
supremacy/administrative power/supreme power.

Use Code NIMISHA for maximum Discount

For any query call or whatsapp 9953592800


267

I & III are talking about exactly the same thing.


enthusiast - ardent supporter.

II - Incorrect. It's incomplete. It hasn't mentioned the anger or indignation amongst the
ministry and its admirers.)

21.
(Answer - D. The formal pronouncement from the ministry of external affairs.

pronouncement/dictum - declaration or statement.


Refer to - '….ministry of external affairs responded to the growing international scrutiny in a
lengthy statement that urged celebrities to ascertain facts before rushing to make critical
comments. This is welcome advice, but the dictum should be applicable to all.' )

22.(Answer - A. inimical

inimical - harmful or detrimental.

symbolical - Serving as a symbol.


well-disposed - friendly.
equivocal - ambiguous.)
23.(Answer - C. Only I can be inferred.

Refer to the line - What strengthens democracy is — not unilateral opinion but — a nation’s
ability to accommodate divergent, competing views.
This single line is explaining the whole paragraph.
Inference means a conclusion reached on the basis of evidence and reasoning and
statement I is certainly a conclusion. Hence, it is the inference as well.

unilateral - one-sided or biased.


divergent - differing; varying; assorted.
competing - challenging.
accommodate - accept or be open and responsive to.

Statement II and III are not inferences. They are just stating the facts mentioned in the
paragraph.

Use Code NIMISHA for maximum Discount

For any query call or whatsapp 9953592800


268

prickliness - here it means violent resistance.


crumble - to fall apart.
frail - weak.)

24.(Answer - D. Only I

paranoid rebuttal - worried denial.

Refer to - Could it be that the Centre is no longer certain that its carefully curated narrative
on the farmers’ protest is having its desired effect?

carefully curated - carefully chosen and thoughtfully organized or presented.


narrative - a particular way of explaining or understanding events.

Statement I is talking about exactly the same thing..

II - Out of the context. It's talking about the government's hesitation.

III - Out of the context. It's talking about providing certainty or confidence in response to the
farmers' protests.)

25.(Answer - E. None of the above options

The correct combination is -

o p m n

The sustained demonization of farmers, the periodic use of force and incitation at the sites
of protest and, above all, the impassivity shown from the highest echelons of power have
bared the ugly truths that no amount of cheerleading on television or on social media can
conceal.

demonization - the portrayal of something as wicked and threatening.


incitation - provocation.
impassivity - apathy or insensitivity.
echelon - a rank or position within an organization.)

Use Code NIMISHA for maximum Discount

For any query call or whatsapp 9953592800


269

26.(Answer - D. bare

bare - to disclose or reveal.

conceal - to hide.
secrete - a) to deposit or conceal in a hiding place.
b) to form and give off.
ensconce - establish or settle (someone) in a comfortable, safe place.
bury - a) put or hide underground.
b) cover (someone or something) completely.
All the remaining words are synonymous with 'conceal'.)

27.(Answer - A. Censorious

Censorious - condemning and criticising.

Vituperative - marked by harshly abusive criticism.


Approbative / Plausive - affirmative.)

28.(Answer - C. Only b

leaching - spreading.
Of course, this statement justifies the statement - The silence, in many cases, is
strategic and deliberate. That’s the nub.

nub - essence.

a - Incorrect. It talks about how people perceive hate speech. Not the government.

c - Incorrect. It's absolutely out of context.)

29.(Answer - E. None of a, b and c

No statement justifies the fact that at the heart of the problem is the political economy
of TV news which thrives on hate speech today more than ever.

Things about political economy are absent in all the three statements.)

Use Code NIMISHA for maximum Discount

For any query call or whatsapp 9953592800


270

30.(Answer - B. Only a and c

The author is absolutely against hate speech and throughout the passage he has
criticized it.

Hence, a and c are absolutely correct.

b - Incorrect. The word 'inclusion' is contradictory.)

31.(Answer - A. Only a and b

Article 19 is about free speech.

Hence, these two statements correctly justify the fact that Hate speech has to be read
within the purview of Article 19, which safeguards the freedom of speech, as both are
connected in some ways.

c - Incorrect. This definition is not related to Article 19.)

32.(Answer - C. Only b and c

The author is disappointed with CBI.

a - Incorrect. It's contradictory.)

33.(Answer - A. Only a and b

c - Incorrect. It's absolutely contradictory.

These names - 22 names)

Use Code NIMISHA for maximum Discount

For any query call or whatsapp 9953592800


271

CHAPTER 8 - PARAGRAPH BASED QUESTION

Direction(1-20): Each question has one small paragraph. It's then followed by
two/three questions, based on the paragraph. You've to answer the questions that
follow.

Paragraph - 1

Evidence is mounting that women have a unique experience of hunger. Biologically, it’s true
that women’s nutritional needs vary on average from those of men: for example, differences
in muscle and fat distribution across the body affect how energy is metabolised, which in
turn contributes to the biological drive to eat. Naturally cycling hormones such as oestrogen

Use Code NIMISHA for maximum Discount

For any query call or whatsapp 9953592800


272

taken through ingestion, injection or via the skin can also affect women’s nutritional needs
and appetite.

1. Describe women's appetite in one single word, according to the context.

A. susceptible
B. refractory
C. fractious
D. perlious
E. None of the above options

2. Choose an appropriate synonym for the word 'ingestion', as used in the passage.

A. digestion
B. uptake
C. inhalation
D. feeding
E. None of the above options

Paragraph - 2

Facebook’s massive reach and its ability to influence the choices — economic or political —
of its users have transformed it into a behemoth that can condition and even alter political
realities. This makes Facebook attractive as an ally for the political fraternity. Such
bonhomie can lead to patronage, which, in turn, can undermine the principles of neutrality
and ethics.
3. How can you describe 'Facebook'?

a. mighty
b. puissant
c. intriguing
d. irksome

A. Only a
B. Both a and c
C. All b, c and d
D. All a, b and c
E. Both c and d

Use Code NIMISHA for maximum Discount

For any query call or whatsapp 9953592800


273

4. Choose an appropriate synonym for the word 'behemoth', as used in the passage.

A. colossus
B. heavyweight
C. contender
D. antagonist
E. None of the above options

5. What's the actual meaning of the phrase 'Such bonhomie can lead to patronage',
according to the context?

I. Such cordiality can lead to a great bond.


II. Such affability can lead to sponsorship.
III. Such friendship can lead to complications.

A. Only III
B. Both I and II
C. Only II
D. Only I
E. None of the above options

Paragraph - 3
Many see the ending of the Supreme Court’s case of criminal contempt against Mr. Bhushan
as a moral victory for him, for all he had to do was pay a fine of one rupee. Yet the Supreme
Court convicted him of criminal contempt of court, which includes scandalizing the institution
and its officers, _________________________ This definition of criminal contempt is
usually linked to words or acts that are intended to obstruct the delivery of justice or the
administration of the judicial system. The court decides whether lowering its dignity is also
affecting its functioning. This power is bequeathed to it by the Constitution.

6. Describe the decision of the Supreme Court against Mr. Bhushan in one single word.

A. Righteous
B. Unhurried
C. Deliberate
D. Inequitable
E. None of the above options

7. Choose a suitable line that can be contextually fit in the blank in the paragraph.

Use Code NIMISHA for maximum Discount

For any query call or whatsapp 9953592800


274

I. thus letting down its dignity in the eyes of the people.


II. hence lowering its baseness in the society.
III. thus fostering its self-worth to the public.

A. Only III
B. Both I and III
C. Only I
D. Both I and II
E. All I, II and III

8. Choose an alternative for 'bequeathed to' as used in the passage.

A. conferred on
B. passed on
C. entrusted to
D. bestowed on
E. All of the above options

Paragraph - 4

Sometimes the real picture emerges even if the smoke has not quite cleared. That is what
seems to have happened with regard to the findings of a study by the National
Environmental Engineering Research Institute, which showed that goods vehicles are
responsible for the greater part of automobile pollution in Calcutta and Howrah. These
revelations are to be taken seriously, as they dispel a number of myths in the public
discourse regarding the dominant role played by modes of public transportation — buses,
taxis and autorickshaws — in growing pollution levels.

9. How would you describe the study by the National Environmental Engineering Research
Institute in one single word?

a. telltale
b. momentous
c. trifling
d. denotative

A. Only a
B. Both a and c

Use Code NIMISHA for maximum Discount

For any query call or whatsapp 9953592800


275

C. All a, b and c
D. All a, b and d
E. All of the above words

10. What's the actual meaning of the phrase 'as they dispel a number of myths' according
to the context?

I. as they make a number of fallacies disappear.


II. as they clear a number of confusions.
III. as they disseminate a number of mistaken beliefs.

A. Only III
B. Both II and III
C. Only I
D. Both I and III
E. All I, II and III

11. Choose the correct synonym of 'dominant', as used in the context.

A. rudimentary
B. premiere
C. preponderant
D. superficial
E. None of the above options

Paragraph - 5

Before the National Statistical Office came out with its macroeconomic data for the first
quarter ended June 30, there was only an ______________ in the pundits’ minds. How bad
could it possibly be? At minus 23.9 per cent, the contraction in real GDP is the worst ever
and the first since 1980. The greater concern is that the nominal GDP, calculated at current
prices, has also contracted by 22.6 per cent in the first quarter. The worry now is that if this
trend continues, Narendra Modi will achieve the unthinkable: he will end up shrinking India’s
economy this fiscal, a feat that nobody has managed to do.

12. If you are to fill up the blank in the line - The Indian economy is ____________, which
of the following words would you choose for the blank according to the context?

A. floundering

Use Code NIMISHA for maximum Discount

For any query call or whatsapp 9953592800


276

B. unfolding
C. rearing
D. frittering away
E. None of the above options

13. What's the actual tone of the author in the above context?

A. deprecative
B. apprehensive
C. acerbic
D. approbative
E. None of the above options

14. Choose an appropriate phrase that will fit contextually in the blank in the paragraph.

A. academic curiosity
B. theoretical outcome
C. semipolitical aspects
D. scholarly upshot
E. All of the above options

Paragraph - 6

There is cause for concern when the numbers do not add up. The recent list for the annual
sports awards approved by the Union sports ministry has, unfortunately, led to the
aggravation of such concern. On the face of it, the list sings paeans to India’s sporting
talents: on it are five Khel Ratnas and 27 Arjuna awardees. Matters get curious, though,
when the possible motives that led to the creation of the list are put under scrutiny. The
men’s national basketball team captain, Vishesh Bhriguvanshi, has bagged an Arjuna even
though in the last-recorded performance, the men’s team finished at the 14th place among
16 nations in the Asian Championship in Beirut. The assessment is thus peculiar. The
composition of the list seems to indicate that factors other than ________ and fairness may
have been given precedence by the powers that be.

15. How would you describe the list for the annual sports awards approved by the Union
sports ministry, according to the context?

a. queer
b. fishy

Use Code NIMISHA for maximum Discount

For any query call or whatsapp 9953592800


277

c. presumptive
d. flimsy

A. Both a and d
B. All a, b and d
C. All b, c and d
D. Only b
E. All of the above words

16. What's the actual meaning of the phrase 'the list sings paeans to'?

A. the list snubs


B. the list assuages
C. the list confuses
D. the list extols
E. None of the above options

17. Choose an appropriate word that will fit contextually in the blank in the paragraph.

A. dereliction
B. foible
C. distinction
D. blemish
E. None of the above options

Paragraph - 7

The policymakers on Mint Street are starting to feel the heat as the Narendra Modi
government starts to ratchet up pressure on the Reserve Bank of India to bail it out of every
crisis that it stumbles into. The finance minister, Nirmala Sitharaman, dropped a bombshell
when she asked the states to borrow up to Rs 2.35 trillion from a special window that the
RBI would open to help them recover compensatory dues under the goods and services tax
regime because of poor tax collections in a Covid-wracked business environment. The
announcement is as much a shock for the central bank as it is for the states as there was no
prior consultation with either. This is just another instance of how the Centre has tried to
browbeat the banking regulator to do its bidding — and then sort out the resultant mess.

18. How would you describe the Narendra Modi government's attitude towards the Reserve
Bank of India?

Use Code NIMISHA for maximum Discount

For any query call or whatsapp 9953592800


278

a. imperious
b. insolent
c. deferent
d. conscientious

A. Both a and c
B. Both b and d
C. All a, b and c
D. Both a and b
E. All a, c and d

19. Choose the correct synonym of 'browbeat', as used in the context.

A. pressurize
B. intimidate
C. compel
D. subjugate
E. All of the above options

20. Which of the following statements is/are TRUE according to the context?

I. Nirmala Sitharaman has asked the states to borrow up to Rs 2.35 trillion from a
special window of the RBI in consultation with the states only.
II. The policymakers on Mint Street are in an uncomfortable situation as the government
has started to pressurise the RBI to follow its orders.
III. Nirmala Sitharaman's decision indicates the way the government intimidates the RBI
to help them sort their problems out.

A. Only III
B. Both I and II
C. Only II
D. Both II and III
E. None of I, II and III

ANSWER

1.(Answer - A. susceptible

Use Code NIMISHA for maximum Discount

For any query call or whatsapp 9953592800


279

susceptible - likely or liable to be influenced by a particular thing.

Women's hunger is influenced by the muscle and fat distribution across the body affect how
energy is metabolised and, naturally cycling hormones such as oestrogen taken through
ingestion, injection or via the skin.

refractory - resistant to a process or stimulus.


fractious - easily upset or annoyed, and often complaining.
pelious - bringing or involving the chance of loss or injury.)

2.(Answer - B. uptake

ingestion / uptake - the process of taking food, drink, or another substance into the body by
swallowing or absorbing it.)

3.(Answer - D. All a, b and c

mighty / puissant - having great power or influence.

intriguing - arousing one's curiosity or interest; fascinating.

irksome - irritating; annoying.)

4.(Answer - A. colossus

colossus - a thing of enormous size, importance, or ability.

heavyweight - a person who is important or serious and that other people notice.

contender - a person or group competing with others to achieve something.


antagonist - one who offers opposition.)

5.(Answer - C. Only II

Use Code NIMISHA for maximum Discount

For any query call or whatsapp 9953592800


280

bonhomie - cheerful friendliness; geniality; affability.


patronage - sponsorship; backing.

cordiality - sincere affection and kindness.)

6.(Answer - C. Deliberate

Deliberate - intentional.

Refer to - The court decides whether lowering its dignity is also affecting its functioning.
It's solely the court's decision. Whether it's righteous, inequitable or unhurried, isn't
clear in the context.

Righteous - moral.
Inequitable - unfair.
Unhurried - careful and patient.)

7.
(Answer - C. Only I
let down - lower.
There should be a negative statement in the blank because of its previous line.
dignity - self-worth.
baseness - lack of moral principles; bad character.
'lowering its baseness' doesn't make any sense.

foster - to grow.)

8.(Answer - E. All of the above options


bequeathed to someone - left in one's will or presented to someone.)

9.(Answer - D. All a, b and d

telltale - revealing or suggestive.


momentous - of great importance.
denotative - Indicative.
trifling - unimportant.)

10.(Answer - C. Only I

Use Code NIMISHA for maximum Discount

For any query call or whatsapp 9953592800


281

dispel here means - to eliminate or to make something disappear.

myth - (here it means) a widely held but false belief or idea.


fallacy - a mistaken belief, especially one based on unsound arguments.
myth can't necessarily be confusion.
disseminate - spread (something, especially information) widely.)

11.
(Answer - C. preponderant

preponderant - predominant in influence, number, or importance.

rudimentary - basic.
premiere - preceding all others in time.
superficial - unimportant.)

12.(Answer - A. floundering

flounder - to struggle.
unfold - to evolve.
rear - to lift or raise.
fritter away - to waste money or time.
('Indian economy is wasting money/time' doesn't make any sense.)

13.(Answer - B. apprehensive

apprehensive - anxious or fearful that something bad or unpleasant will happen.

deprecative - belittling.
acerbic - harsh in tone.
approbative - affirmative.)

14.(Answer - A. academic curiosity


The sentence is followed by a question - 'How bad could it possibly be?'
So, 'curiosity' is the correct word here.

semipolitical - partially political nature.


upshot - result.)

Use Code NIMISHA for maximum Discount

For any query call or whatsapp 9953592800


282

15.(Answer - B. All a, b and d

queer - strange.
fishy - dubious or doubtful.
flimsy - unconvincing.

presumptive - believed to be something, or likely to be true.)

16.(Answer - D. the list extols

paean - a song of praise.


extol - to praise.
snub - refuse to acknowledge.
assuage - to satisfy (an appetite or desire) or to appease.)

17.(Answer - C. distinction

distinction - excellence that sets someone or something apart from others; merit.

dereliction - the shameful failure to fulfil one's obligations.


foible - weakness.
blemish - imperfection.)

18.(Answer - D. Both a and b

imperious - arrogant and domineering.


insolent - showing a rude and arrogant lack of respect.

deferent - respectful.
conscientious - feeling a moral responsibility to do your work carefully and to be fair to
others.)

19.(Answer - E. All of the above options

browbeat - intimidate or force (someone), typically into doing something, with stern or
abusive words.

pressurize / compel - attempt to persuade or coerce (someone) into doing something.

Use Code NIMISHA for maximum Discount

For any query call or whatsapp 9953592800


283

subjugate - to treat yourself, your wishes, or your beliefs as being less important than other
people or their wishes or beliefs.)

20.(Answer - E. None of I, II and III

All are FALSE.

I - FALSE. Refer to - The announcement is as much a shock for the central bank as it is for
the states as there was no prior consultation with either.
The states also didn't have any idea about it.

II - FALSE. It may be true in sense, but it's not directly mentioned in the paragraph.
III - FALSE. Refer to - This is just another instance of how the Centre has tried to browbeat
the banking regulator to do its bidding — and then sort out the resultant mess.
'resultant mess' isn't the problems the government is facing. It's the aftermath that is
created due to pressurizing the RBI.)

CHAPTER 9 - MISCELLANEOUS PATTERN

Direction(1-4): Four statements are given below, labelled A, B, C and D, among these,
three statements are logically connected. From the given options, choose the option
that does not fit into the theme of the passage.
You're now given three statements. You've to replace the odd statement with a
statement / statements from the three statements given, that is/are in line with the
other three statements.

1.
A. In the summer we enjoy eating mangoes; during the monsoon we relish custard apples
and cherries and in the winter it's time to eat strawberries, grapes and oranges.
Use Code NIMISHA for maximum Discount

For any query call or whatsapp 9953592800


284

B. The French enjoy longevity because they only eat seasonal produce; a fruit that is 'in
season' has nutritional properties that overcome the imbalances that a change of weather
brings along.

C. Fruit and vegetables are grown and harvested in different locations and in different
seasons throughout the year; for example, oranges are climate-sensitive plants and grow
better in places with hot dry summers such as Spain, Italy, and Greece.

D. Do you remember the last time you visited a grocery store and didn’t find bananas?
They’re pretty ubiquitous, and understandably so; we can enjoy fresh, luscious bananas in
the dead of January, as well as in July.

A. A B. B C. C D. D E. No statement is ODD

2. Which of the following statements can replace the odd statement in the above
context?

a. It’s funny to think of grapefruit as a happy accident, but that’s exactly how this species
came about; pomelos and sweet oranges were brought to Barbados from Asia in the
1600s, and boom!
b. Tomatoes are always in season because there’s such an incredible demand for them
around the globe; that’s why people cultivate them greenhouses and hydroponic labs
from Iceland to New Zealand, rain or shine, sun or snow.
c. You’ve probably seen many culinary dishes that include the Kiwi as an essential
ingredient; it is because it is one of the healthiest winter fruits in India, thanks to the
presence of several vitamins and minerals that will ensure your body’s proper
functioning at all times.

A. Only a and b B. Only c C. Only b D. Only a E. None of a, b and c

3.

Use Code NIMISHA for maximum Discount

For any query call or whatsapp 9953592800


285

A. Miniatures demonstrate the most elevated type of craftsmanship; from the inside to the
outside, engineering these precious objects took time and patience; upon opening a vanity
case, you can see there is place for everything - a lipstick holder, a pair of glasses, mirror,
comb and cigarette lighter.

B. The materials used for vanity cases presented jewellers with infinite possibilities: precious
stones, onyx, jade, tortoise-shell – using craftsmanship and ingenuity, the greatest jewellery
houses created cases of all designs, textures and colours.

C. Bejewelled, elaborate and delicate; the vanity case is the essence of elegance; an
embellished case that serves many different roles, the vanity case is both clutch and
compact case allowing for ultimate yet discrete storage for one’s accessories.

D. Vanity box is an exceptionally valuable and minimized storage box; this fits well in a
suitcase, holds bounty, and can hang in a wide range of spots; also you can structure your
cosmetics in the inner compartments by the movable cushioned dividers.

A. A B. B C. C D. D E. No statement is ODD

4. Which of the following statements can replace the odd statement in the above
context?

a. Pride huge light pink aluminum surrounded adornments vanity case with a fake
calfskin impact covering and four extendable compartments - ideal for amassing up,
adornments, makeup, and toiletries with its hearty structure and simple grasp convey
handle.
b. There’s something quite precious about carrying a vanity case as a handbag, as if the
wearer must have their essentials in perfect order at all times; a vanity case is perfect
for the one who prizes useful pockets and pouches over space.
c. Structure is part of the appeal, but so is the sleekness of the leather vanity case;
perfectly boxy with pristine hardware, the top-handle bags can lend polish to looks –
particularly travel wear.

A. Only a and b B. Only a and c C. Only b and c D. All a, b and c E. None of a, b


and c

Use Code NIMISHA for maximum Discount

For any query call or whatsapp 9953592800


286

SBI PO PRE MEMORY-BASED PAPER 2021

Direction(5-7): In the following questions two columns are given containing three
Sentences/phrases each. A sentence/phrase from the first column may or may not
connect with another sentence/phrase from the second column to make a
grammatically and contextually correct sentence. Choose the correct pair(s).

5.
Column I
A. If the monsoon does turn out normal and farmers respond to high prices by planting
aggressively,
B. Even as economy-watchers are primed for a further hike in the Reserve Bank of
India’s benchmark interest rates,
C. Inflationary pressures remain elevated and so does the threat of inflation expectations
becoming self-fulfilling —

Column II
D. one can expect a bumper domestic kharif crop as well.
E. which is what prompted the MPC’s decision to increase the policy repo rate by 40
basis points to 4.4 per cent.
F. there are perhaps three reasons for its monetary policy committee (MPC) to be less
alarmist than in last month’s “off-cycle” meeting.

A. AD, BF and CE
B. AE, BD and CF
C. AD, BE and CF
D. AF, BD and CE
E. None of the above options

6.
Column I
A. With rampant lockdowns amidst the pandemic, a virtual office was set up on

Use Code NIMISHA for maximum Discount

For any query call or whatsapp 9953592800


287

B. In a crisis, when all resources are stretched to the limit, a robust system could help
optimise
C. CoWIN, or the Covid-19 Vaccine Intelligence Network, was developed in a record
time, with

Column II
D. consideration given to scalability, modularity, and interoperability.
E. resource utilisation for maximum coverage and efficiency.
F. unnecessary stress for the population or the medical system.

A. Only AF and BD
B. Only BE and CD
C. AF, BE and CD
D. AD, BF and CE
E. None of the above options

7.
Column I
A. The GReAT report draws upon surveys conducted in 20 countries and these indicate
that the proportion of the population currently using
B. Universal assistive technology coverage implies that everyone, everywhere receives
the Assistive Technology
C. Disadvantaged groups and communities face hardships in their search for affordable
quality healthcare in India and this is more so

Column II
D. that they need without financial or any other hardship.
E. with respect to obtaining Assistive Technologies and associated services — the
estimated unmet need is about 70 per cent.
F. at least one assistive product ranges from less than 3 per cent to about 70 per cent.

A. AD, BF and CE
B. AE, BD and CF
C. AD, BE and CF
D. AF, BD and CE
E. None of the above options

Use Code NIMISHA for maximum Discount

For any query call or whatsapp 9953592800


288

Direction(8-11): In the following question, three statements are given. One or more
than one statement may have grammatical or contextual errors in it/them. You've to
choose the grammatically and contextually INCORRECT statement(s).

8.
a. The health system’s responsibility in providing equitable access to ATs, just as for
essential medicines and vaccines, is increasingly being recognised and country-level
plans are drawing up with an aim to fund and provide Assistive Technologies under
the Universal Health Coverage.
b. By sending a delegation of officials of appropriate seniority, with no political fronting,
India has also conveyed the approach cannot be read as a recognition of the Taliban
as legitimate rulers.
c. For now, the engagement is likely to remain functional, for liaising on India’s
humanitarian supplies to Afghanistan, and for trade, which has remarkably continued
even after the Taliban takeover, and despite the absence of diplomatic relations
between the two sides.

A. Only a and b B. Only a and c C. Only b and c D. All a, b and c E. None of a, b


and c

9.
a. India must be prepared for the Afghan people’s expectation that it will back them on
issues such as media freedom and women’s rights, which the Taliban are rolling
back, promulgated various restrictions.
b. We had the most sophisticated software available at that point in time, but I still
remember the warning he raised about the danger in the large-scale digitisation of
archival recordings of Indian classical music without listening to the nuances.
c. A premature and punishing summer makes life miserable for many across India,
especially the cities, because the soaring temperatures adversely affect health, cause
a dip in agricultural production, and also dry up rivers.

A. Only a and b B. Only a and c C. Only b and c D. All a, b and c E. None of a, b


and c

10.

Use Code NIMISHA for maximum Discount

For any query call or whatsapp 9953592800


289

a. Cities in India are also feeling the heat due to a combination of factors, including the
urban heat island effect, but largely on account for ill-conceived urbanisation.
b. Even a cursory look at the situation in Myanmar, post the February 1, 2021, military
takeover, conjuring up a picture of a country that is spiralling downwards.
c. Favourable bilateral relations with Bangladesh, under Sheikh Hasina, offer an
opportunity for opening a new axis of land-sea connectivity for promoting trade and
commerce with Southeast Asia.

A. Only a and b B. Only a and c C. Only b and c D. All a, b and c E. None of a, b


and c

11.
a. But Indian films have ably demonstrated over the years that there is a market for both
what are colloquially known as ‘mainstream’ and ‘alternative’ cinema.
b. The Cannes Film Festival last week delivered a vote of confidence for films produced
by the world’s most densely populated region.
c. While India’s economy has changed discernibly after the market-friendly economic
reforms of 1991, all is well with some features of the economy.

A. Only a and b B. Only a and c C. Only b and c D. All a, b and c E. None of a, b


and c

New Pattern Odd-One Out Questions

Directions(12-20): Four small paragraphs are given below, labelled A, B, C and D,


among these, three paragraphs are logically connected. From the given options,
choose the option that does not fit into the theme of the passage.

There will then be a QUESTION based on the THREE CONTEXTUALLY CONNECTED


PARAGRAPHS and a QUESTION based on the ODD PARAGRAPH.

Question 1

12.

Use Code NIMISHA for maximum Discount

For any query call or whatsapp 9953592800


290

A. Despite sharing certain values and religious beliefs – as well as living in the same
country, under the same constitution – members of India’s major religious communities often
don’t feel they have much in common with one another.

B. Indians see religious tolerance as a central part of who they are as a nation. Across the
major religious groups, most people say it is very important to respect all religions to be
“truly Indian.”

C. More than 70 years after India became free from colonial rule, Indians generally feel their
country has lived up to one of its post-independence ideals: a society where followers of
many religions can live and practice freely.

D. India’s massive population is diverse as well as devout. Not only do most of the world’s
Hindus, Jains and Sikhs live in India, but it also is home to one of the world’s largest Muslim
populations and to millions of Christians and Buddhists.

A. A B. B C. C D. D E. No paragraph is ODD

13. Which of the following statements can be inferred from PARAGRAPHS B, C and
D?

a. Religious diversity has been a defining characteristic of India’s population for


centuries.
b. Indians usually express enthusiasm for religious tolerance.
c. In other words, Indians’ concept of religious tolerance does not necessarily involve
the mixing of religious communities.

A. Only a and b B. Only a and c C. Only b and c D. All a, b and c E. None of a, b


and c

14. Which of the following things can possibly be said according to paragraph A?

a. The majority of Hindus see themselves as very different from Muslims (66%), and
most Muslims return the sentiment, saying they are very different from Hindus (64%).

Use Code NIMISHA for maximum Discount

For any query call or whatsapp 9953592800


291

b. Indians generally stick to their own religious group when it comes to their friends for
the similarities in cultural and traditional values.
c. Indians express enthusiasm for a consistent preference for keeping their religious
communities in segregated spheres.

A. Only a and b B. Only a and c C. Only b and c D. All a, b and c E. None of a, b


and c

Question 2

15.
A. When asked to name the biggest risks of employees making serious security mistakes,
78% of respondents cited poor password hygiene and 81% pointed to misuse of personal e-
mail. When asked what they expected their biggest security challenges to be in the coming
year, 41% of local respondents said one of their biggest concerns was employee naivety.

B. Phishing is the biggest culprit: 96% of companies have been the target of an e-mail-
related phishing attempt, with 65% reporting an increase in such attacks. Ransomware is
also running amok.

C. Among the South African SOES respondents, nine out of 10 reported an increase in e-
mail volume at their organisations. At the same time, three-quarters of the respondents said
the number of e-mail-based threats had also risen, with 55% citing concerns over the
sophistication of e-mail-based attacks.

D. Over a dozen airport websites are regularly impacted by the "denial of service" attack.
That type of attack essentially overloads sites by jamming them with artificial users.

A. A B. B C. C D. D E. No paragraph is ODD

16. Which of the following paragraphs can be inferred from PARAGRAPHS B, C and
D?

Use Code NIMISHA for maximum Discount

For any query call or whatsapp 9953592800


292

a. Cyber security threats are on the rise in many countries and small-and medium-sized
businesses have become a soft target due to their lack of preparedness and
resources.
b. Cyber attacks have been rated a top rated risk and become the new norm across
public and private sectors. This risky industry continues to grow.
c. Cyber attacks have been on the rise, in sync with the digitization of business that has
become more and more popular in recent years; there are dozens of different types of
attacks.

A. Only a and b B. Only a and c C. Only b and c D. All a, b and c E. None of a, b


and c

17. What are the possible suggestions or warnings that can be made, in line with
paragraph A?

a. Re-work your current passwords, using numbers and capital letters intermittently
along with uncommon symbols. If you have newyork as your password, try changing
it to something like N3wY0rK!
b. Check the context of the email message. If anything looks suspicious or the person is
asking you something that they wouldn’t usually ask you to do, it’s more than likely
spam.
c. Most ads are harmless, but the ones that are too good to be true – how to instantly
make $1 million dollars or you WON’T believe what Hillary/Trump/Kimye just did are
often clickbait advertisements that could lead you to sites with malware.

A. Only a and b B. Only a and c C. Only b and c D. All a, b and c E. None of a, b


and c

Question 3

18.
A. Cyber insurance, also known as cyber risk insurance and cyber liability insurance,
protects businesses from the impact of cyber crime. Hand-in-glove with this, good cyber
security can help with cyber insurance in multiple ways.

Use Code NIMISHA for maximum Discount

For any query call or whatsapp 9953592800


293

B. Data breach insurance or Cyber insurance costs vary depending on the size of your
company and how much coverage your business needs. If you have a smaller company,
you may not need as much coverage, and thus, your premiums will be lower than a larger
company with many areas that need protection.

C. Cyber insurance generally covers your business' liability for a data breach involving
sensitive customer information, such as Social Security numbers, credit card numbers,
account numbers, driver's license numbers and health records.

D. Cyber insurance has media liability coverage that protects you from intellectual property
infringement, except for patent infringement. This coverage typically applies to both printed
advertising and online advertising, including your company’s social media posts.

A. A B. B C. C D. D E. No paragraph is ODD

19. Which of the following CONCLUSIONS can be made after going through
PARAGRAPHS A, C and D?

a. Cybersecurity insurance, while at first viewed as a niche tool, is now considered a


requirement for every company’s risk management system.
b. Though cybersecurity insurance coverage is now essential, many businesses remain
unaware that cyber risk is insurable, let alone what exactly cybersecurity insurance
covers.
c. Cyber risk is capable of being insured and the coverage options available today are
comprehensive enough to meet the needs of your company.

A. Only a and b B. Only a and c C. Only b and c D. All a, b and c E. None of a, b


and c

20. Which of the following things can possibly be said according to paragraph B?

a. No matter what type of insurance policy you’re purchasing, there are certain
characteristics of your business that are considered the main drivers behind cyber
insurance cost.

Use Code NIMISHA for maximum Discount

For any query call or whatsapp 9953592800


294

b. The size of your company is important because the more employees you have, the
greater the risk of phishing and social engineering attacks you face.
c. Low-risk companies, such as local businesses with a limited customer base, will pay
less for their cyber insurance than, for example, a retail store that receives and stores
customer credit card numbers in their store and through their website or ecommerce
shop.

A. Only a and b B. Only a and c C. Only b and c D. All a, b and c E. None of a, b


and c

Various Types of Questions on a Passage

Types - Phrase Filler, Idioms, Comprehensive Ability(3Q).

Direction(21-25): The passage below has SOME QUESTIONS OF DIFFERENT


PATTERNS. Detailed instructions are given below. Answer those questions by
marking the correct options.

The authors of the Global Hunger Index (GHI), released on October 15, write that "the level
of hunger in India is serious''. The country is ranked 107 of the 121 countries they studied.
With a score of 29.1 (0 means no hunger), India is behind its South Asian neighbors —
Nepal, Bangladesh, Sri Lanka and Pakistan. Close to 20% of children in the country below
the age of five suffer from the most visible and life-threatening form of malnutrition —
wasting. About 35% of such children are not as tall as they should be. These sobering
factoids could have served as inputs for government programmes such as Poshan 2.0 and
the Mid-Day Meal scheme. However the Ministry of Women and Child Development not only
dismissed the GHI but also questioned the intent of its authors and the veracity of the index.

Part of the government’s critique pertains to one of the major takeaways of the GHI — the
pandemic seems to have aggravated India’s malnutrition crisis. It contends that the report
ignores the food security efforts undertaken during the crisis, especially the provision of 5 kg
foodgrain to all beneficiaries of the National Food Security Act in addition to their regular
ration. There can be little doubt about the efficacy of the PDS as a safety net during troubled

Use Code NIMISHA for maximum Discount

For any query call or whatsapp 9953592800


295

times. However, as several scholars have pointed out, the nutrition deficit of the country’s
children is, in large measure, a function of their poor diets.

The government has argued that the index relied on an opinion poll. The authors of the
report have clarified that the GHI draws on data reported by member countries, including
India. The debate on methodology isn’t settled and such conversations could enrich the
understanding of a longstanding predicament faced by the country — one that the National
Family Health Surveys have long underscored. The NFHS-5, conducted just before the
pandemic, showed that more than 20% of children below the age of five did not weigh as
much as they should. The discourse on nutrition could acquire greater nuance with more
data — on household consumption of food items, for instance. But the country has not had
any official estimate on per capita household spending since 2011-12. Work on filling this
vacuum has, reportedly, begun. The government would be well-advised to
_____________________ that can help it address malnutrition effectively, and
_____________________.

Type - Comprehensive Ability

21. Which of the following worrying facts about children in the country is/are TRUE,
according to the above passage?

a. Around 20% of children in the country are too thin and their immune systems are
weak, leaving them vulnerable to developmental delays, disease and death.
b. About 35% of children suffering from undernourishment and wasting are suffering
from stunting.
c. Around 35% of children in the country are not adequately tall.

A. Only a and b B. Only a and c C. Only a D. All a, b and c E. None of a, b and c

Type - Comprehensive Ability

22. What does the Ministry of Women and Child Development probably say about the
Global Hunger Index (GHI), as expressed in the above passage?

a. The index is an erroneous measure of hunger and suffers from serious


methodological issues.

Use Code NIMISHA for maximum Discount

For any query call or whatsapp 9953592800


296

b. The ranking is a part of a consistent effort to taint India's image as a nation that does
not fulfill the food security and nutritional requirements of its population.
c. The ground reality is much different from what is being portrayed in the international
data and that misinformation seems to be the hallmark of the annually released
Global Hunger Index.

A. Only a and b B. Only a and c C. Only b and c D. All a, b and c E. None of a, b


and c

Type - Comprehensive Ability

23. Which of the following statements can possibly be inferred from what several
scholars have pointed out regarding the nutrition deficit in children in the country?

a. Many children in the country do not consume adequate amounts of fruits, vegetables
and non-cereal proteins.
b. Eggs do not figure in the mid-day meal schemes in a majority of states.
c. Children have a greater need for essential nutrients per kilogram of body weight, but
their disinclination to take proper food is creating all the problems.

A. Only a and b B. Only a and c C. Only b and c D. All a, b and c E. None of a, b


and c

Type - Phrase Filler

The government would be well-advised to _____________________ that can help it


address malnutrition effectively, and _____________________.

24. Which of the following pairs of phrases can most appropriately fit in the blanks in
the above portion of the passage respectively?

a. bolster its information repositories, not be prickly about GHI rankings


b. strengthen its food distribution system, not be complacent about GHI rankings
c. hold up its information depositaries, not be making a fuss about GHI rankings

Use Code NIMISHA for maximum Discount

For any query call or whatsapp 9953592800


297

A. Only a and b B. Only a and c C. Only b and c D. All a, b and c E. None of a, b


and c

Type - Deriving Idioms

25. How would you describe the performance of India in the GHI report and India's
attitude to the GHI report, using IDIOMS?

a. weak sauce, gnashing of teeth


b. of the first water, raising a rumpus
c. not up to snuff, kicking up a stink

A. Only a and b B. Only a and c C. Only b and c D. All a, b and c E. None of a, b


and c

Type - Comprehensive Ability

26. What can be inferred from what has been said in the SECOND PARAGRAPH of the
above passage?

a. All the time, we are taking sides.


b. All the time we are seeing someone as an ally, someone as an opponent.
c. We believe, staying non-discriminatory and even-handed is the best way to live.

A. Only a and b B. Only a and c C. Only b and c D. All a, b and c E. None of a, b


and c

Type - Comprehensive Ability

27. Why do you think we are called upon to be in a constant judgment mode?

a. Because, as we live more like a consumer than a citizen, this makes the market
happy.
b. Because, the market feeds and loves this state of constant restlessness and
judgment.

Use Code NIMISHA for maximum Discount

For any query call or whatsapp 9953592800


298

c. Because, we are comforted and vindicated by the illusion that our voting is being
counted every moment, 24 by 7.

A. Only a and b B. Only a and c C. Only b and c D. All a, b and c E. None of a, b


and c

Type - Comprehensive Ability

28. Why has the author criticized the voting mindset so much, in the entire passage?

a. Because he believes, because we reduce our arguments into yes or no, like or hate,
for or against, we become the champions of dumbing down, of mediocrity day by day.
b. Because, voting in its broader meaning and understanding, has been a part of
contemporary political philosophy and other social choice theories.
c. Because he thinks we are always in canvassing mode, and it makes us unmerciful
and disparaging.

A. Only a and b B. Only a and c C. Only b and c D. All a, b and c E. None of a, b


and c

Type - Phrase Filler

Democracy endures not only because the majority is heard but the majority doesn’t
weaponise the verdict to target the other.
That is the gift we can give to each other and to ourselves this Diwali.
___________________________________. That’s the purest essence of democracy —
and a good way to say Happy Diwali.

29. Which of the following phrases can most appropriately and contextually fit in the
blank in the above statement?

a. The realization that inflicting injury on others is causing severe injury on ourselves.
b. The realization that healing others is healing ourselves.
c. The realization that being selfless is laying a finger on yourself.

Use Code NIMISHA for maximum Discount

For any query call or whatsapp 9953592800


299

A. Only a and b B. Only a and c C. Only b and c D. All a, b and c E. None of a, b


and c

Type - Comprehensive Ability

However, this constant voting mindset has affected our moral reasoning. Human reasoning,
simplicity and empathy for others are values that are the core principles of humanity but
today they are wonderful to preach but not to practice.

30. Which of the following QUESTIONS can arise in the END of the above paragraph
of the above passage?

a. Because, won an election by praising the rival candidate?


b. Because, who will be responsible for practicing humanity?
c. Because, who understands the actual value of commiseration and righteousness?

A. Only a and b B. Only a and c C. Only b and c D. All a, b and c E. None of a, b


and c
(f) Yes, it is Diwali, and beyond those twinkling lights, there hides the well-known secret.

31. Which of the following IDIOMS can contextually be derived from the above
statement?

a. out of sight
b. as plain as a pikestaff
c. hole-and-corner

A. Only a and b B. Only a and c C. Only b and c D. All a, b and c E. None of a, b


and c

(g) Who else will feel the hurt when your son turns around and asks, “You didn’t make
sweets again this year?”

Use Code NIMISHA for maximum Discount

For any query call or whatsapp 9953592800


300

32. Which of the following WORDS can contextually be derived from the above
statement?

a. attachment
b. affinity
c. propinquity

A. Only a and b B. Only a and c C. Only b and c D. All a, b and c E. None of a, b


and c

(h) As per the survey, nearly half of all women in the US experienced heightened stress
during the holidays, at risk to the health of their minds, not to speak of bodies.

33. Which of the following WORDS and IDIOMS can contextually be derived from the
above statement?

a. put on the line


b. hassle
c. exacerbated

A. Only a and b B. Only a and c C. Only b and c D. All a, b and c E. None of a, b


and c

(i) And finally no one expects us to, but when all the photos pop up on your WhatsApp
groups of well-dressed, smiling families, you better have cajoled, pleaded, run around, made
the effort with your own for that perfect shot.

34. Which of the following IDIOMS can be derived from different WORDS in the above
statement?

a. twist someone's arm

Use Code NIMISHA for maximum Discount

For any query call or whatsapp 9953592800


301

b. have to one's name


c. grin from ear to ear

A. Only a and b B. Only a and c C. Only b and c D. All a, b and c E. None of a, b


and c

(j) A survey by the American Psychological Association tells us women bear the brunt of
festivals.

35. Which of the following PHRASES can contextually be derived from the above
statement?

a. be subjected to
b. live through
c. take stock of

A. Only a and b B. Only a and c C. Only b and c D. All a, b and c E. None of a, b


and c

36. Which of the following combinations is the combination of FOUR CONSECUTIVE


STATEMENTS after the correct rearrangement?

A. b a c d
B. e d h g
C. a d g i
D. c e f d
E. None of the above options

Various Types of Questions on Short Passages

Types - Phrase Filler, Word Filler, Comprehensive Ability(2Q), Idiom.

Use Code NIMISHA for maximum Discount

For any query call or whatsapp 9953592800


302

Direction(37-42): The short passage below has SOME QUESTIONS OF DIFFERENT


PATTERNS. Detailed instructions are given below. Answer those questions by
marking the correct options.

If not for a last minute decision to skip the Sunday mass in 2019, Sri Lanka’s captain Dasun
Shanaka might well not be alive today. St Sebastian Church, in his home town Negombo,
was one of the six churches and hotels attacked by terrorists. Shanaka was too tired to
make it to church and when he heard the blast, he rushed to find it completely destroyed. A
few months later, the nation would slip into economic ________ and political crisis, with
people rising in protest against those in power. Inflation shot up over 50 per cent, people
struggled with daily power cuts and shortages of fuel, food, and medicines,
____________________________.

Cricket continued amidst this crisis. When the Australian cricket team toured Sri Lanka in
June this year, they were met with lines of people _________ all around the city of Colombo
against the government. But inside the stadium people _________________________.

Chamika Karunaratne, the allrounder who did the Naagin dance after the team won against
Bangladesh at the just concluded Asia Cup, once had to miss practice as he was stuck in
the line for petrol. Several former cricketers such as Roshan Mahanama participated in the
peaceful protests and numerous others including Mahela Jayawardene and Kumar
Sangakkara, two modern greats of the game, voiced their support to the people.

If the chaos around wasn’t enough, Sri Lankan cricket went through a musical chairs of
captaincy and coaches before settling with Shanaka as the captain and Chris Silverwood,
who was unceremoniously sacked from England after the Ashes defeat, as the coach. Two
years ago, they had dropped many senior players. In 2021, after a heavy defeat to England,
Shanaka was made captain. The irony is that Sri Lanka is yet to qualify for the T20 World
Cup, and have to play qualification rounds. From then on, to reunite the team, infuse them
with energy and purpose, it has been an unenviable task. On Sunday, Team Sri Lanka was
crowned the Asian champions, a win they will _______ for a long time.

Type - Phrase Filler

Use Code NIMISHA for maximum Discount

For any query call or whatsapp 9953592800


303

Inflation shot up over 50 per cent, people struggled with daily power cuts and shortages of
fuel, food, and medicines, ____________________________.

Cricket continued amidst this crisis. When the Australian cricket team toured Sri Lanka in
June this year, they were met with lines of people protesting all around the city of Colombo
against the government. But inside the stadium people _________________________.

37. Which of the following pairs of PHRASES can most appropriately fit in the blanks
in the above passage respectively?

a. as the country defaulted on its debt repayments, turned out in large numbers to watch
the game
b. since the country cut the mustard in footing the bill for its debt repayments, showed
up in abundance to enjoy the game
c. as the country foundered in shelling out for its debt repayments, came along in huge
numbers to relish the game

A. Only a and b B. Only a and c C. Only b and c D. All a, b and c E. None of a, b


and c

Type - Word Filler

A few months later, the nation would slip into economic ________ and political crisis, with
people rising in protest against those in power.

Cricket continued amidst this crisis. When the Australian cricket team toured Sri Lanka in
June this year, they were met with lines of people _________ all around the city of Colombo
against the government.

On Sunday, Team Sri Lanka was crowned the Asian champions, a win they will _______ for
a long time.

38. Which of the following combinations of WORDS can most appropriately fit in the
blanks in the above portions respectively?

a. disarray, dissenting, cherish


b. chaos, standing by, treasure
c. mayhem, crying out, hold dear

Use Code NIMISHA for maximum Discount

For any query call or whatsapp 9953592800


304

A. Only a and b B. Only a and c C. Only b and c D. All a, b and c E. None of a, b


and c

Type - Comprehension

39. Which of the following statements can be made on Sri Lanka's cricket, based on
your own conscience?

a. It is the fighting spirit of the Sri Lankan cricketers that continues to sail them through
everything; the success at the Asia cup has come at a time when the country’s pitch
is bouncy, patchy and full of uncertainties.
b. It's wonderful how a bunch of men coming from a country that’s facing political turmoil
and deep economic crisis marshalled their way into the finals of the Asia Cup 2022
and beat one of the tournament favourites.
c. Sports can hardly console those who have lost homes or jobs in Sri Lankan crisis; Sri
Lanka’s masterly triumph will perhaps act as a much-needed pinprick for the
distressed nation.

A. Only a and b B. Only a and c C. Only b and c D. All a, b and c E. None of a, b


and c

Type - Comprehension

40. If you have to justify the phrase 'power of sports', which is aptly appropriate for
the above context, which of the following statements can be made contextually?

a. Sports can temporarily alleviate clouds of darkness and despair, it can put a smile on
the faces of people; at a time when worry, tension and uncertainty are constant
companions, that is simply priceless.
b. Against the disturbing backdrop for Sri Lanka's ongoing crisis, to celebrate a title run
in cricket might appear incongruous.
c. This Asia Cup triumph will not change their lives, it won’t make the hardships
disappear in a trice or immediately return them to their glory days when tourism

Use Code NIMISHA for maximum Discount

For any query call or whatsapp 9953592800


305

thrived and the masses came a-thronging, but it instills the hope that it is darkest
before dawn, and that a turnaround is never too far away.

A. Only a and b B. Only a and c C. Only b and c D. All a, b and c E. None of a, b


and c

Type - Idioms

41. How would you describe the character of Sri Lankan cricket, using an appropriate
idiom?

a. rock-ribbed
b. stout-hearted
c. of the first water

A. Only a and b B. Only a and c C. Only b and c D. All a, b and c E. None of a, b


and c

(f) It is the subject of one of the oldest known works of literature, Gilgamesh; it has also
inspired such playwrights, poets and novelists as Shakespeare, Shelley, Poe and Conrad.

42. How would you describe 'It', as expressed in the above statement?

a. vernacular
b. a shot in the arm
c. formal

A. Only a and b B. Only a and c C. Only b and c D. All a, b and c E. None of a, b


and c

Use Code NIMISHA for maximum Discount

For any query call or whatsapp 9953592800


306

(g) The answer, science says, lies in their DNA; a new study published in the journal, Cell
Reports, claims that unrelated look-alikes may share more than just facial features.

43. Which of the following statements can be made from the above statement?

a. Human resemblance is more than just skin deep.


b. Each person's microbiome, for instance, is heavily influenced by environmental
factors like nutrition, exercise, and smoking.
c. Disparate people with super similar faces also may have similar heights, weights,
habits, and behaviors.

A. Only a and b B. Only a and c C. Only b and c D. All a, b and c E. None of a, b


and c

(h) The doppelgängers not only shared similarities in DNA but were genetically predisposed
to certain lifestyle traits, such as smoking habits as well as education levels.

44. Which of the following statements can be suggested by the above statement?

a. Shared genetic variation relates not only to physical appearance but may also
influence common habits and behaviour.
b. There are greater differences in DNA modifications and microbiome between
doppelgangers.
c. The doppelgangers who looked most similar, “not only shared the face, but also other
features beyond, such as anthropometric characters and personality traits.

A. Only a and b B. Only a and c C. Only b and c D. All a, b and c E. None of a, b


and c

Use Code NIMISHA for maximum Discount

For any query call or whatsapp 9953592800


307

(i) The study used facial recognition algorithms, genetic testing and questionnaires on daily
life and behaviour to identify similarities between 32 pairs of look-alikes.

45. How would you describe the study, as expressed in the above statement?

a. methodical
b. arbitrary
c. empirical

A. Only a and b B. Only a and c C. Only b and c D. All a, b and c E. None of a, b


and c

(j) The researchers ran the facial profiles of the participants through three different facial
recognition algorithms; each algorithm recognised the same 16 pairs from this set as
‘identical twins’.

46. How would you describe the process described in the above statement?

a. tidy
b. hands-on
c. heuristic

A. Only a and b B. Only a and c C. Only b and c D. All a, b and c E. None of a, b


and c

47. Which of the following combinations is the combination of FOUR CONSECUTIVE


STATEMENTS after the correct rearrangement?

A. b a c d
B. e d h g
C. a d g i
D. c e f d
E. None of the above options

Use Code NIMISHA for maximum Discount

For any query call or whatsapp 9953592800


308

(g) It is estimated that if a minimum-wage earner were to aspire to earn the remuneration of
a senior CEO in the garment industry, it would take the person 941 years to do so; there
cannot be starker evidence of embedded inequality in the country.

48. Which of the following statements can be TRUE according to the above
statement?

a. The patterns of wealth inequality that we see in society are deeply embedded in the
way our economic system works.
b. In creating a more egalitarian system, truly radical changes to how we own wealth will
not be imperative.
c. Three types of economic inequality do exist in India - Income Inequality, Pay
Inequality and Wealth Inequality.

A. Only a and b B. Only a and c C. Only b and c D. All a, b and c E. None of a, b


and c

(h) India has recently overtaken the United Kingdom to become the fifth largest economy of
the world; this higher rank is considered by the Government of India as a grand success.

49. How would you describe the attitude of the Government of India, as expressed in
the above statement?

a. browned off
b. gratified
c. full of oneself

Use Code NIMISHA for maximum Discount

For any query call or whatsapp 9953592800


309

A. Only a and b B. Only a and c C. Only b and c D. All a, b and c E. None of a, b


and c

(i) According to the International Monetary Fund, India’s per capita income at current prices
stood at $2,191 in 2021 and was expected to increase to $2,358 in 2022; if the degree of
inequality is taken into account and the incomes of the top 10% excluded, then the per
capita income would fall to a little less than half the total.

50. Which of the following attributes has the International Monetary Fund measured?

a. A measure of the amount of money earned per person in a nation or geographic


region.
b. Total income measures the average income earned per person in a given area (city,
region, country, etc.) in a specified year.
c. The figure that is derived when the national income is divided by the population size.

A. Only a and b B. Only a and c C. Only b and c D. All a, b and c E. None of a, b


and c

51. Which of the following combinations is the combination of THREE CONSECUTIVE


statements after the correct rearrangement?

A. b a d
B. e d f
C. a e c
D. d c g
E. None of the above options

Direction(52-55): Given below the sentences each of which has been divided into five
parts out of which the first part has been marked bold. Each of the questions is then

Use Code NIMISHA for maximum Discount

For any query call or whatsapp 9953592800


310

followed by the five options which give the sequence of the rearranged parts. You
must choose the option which gives the correct sequence of the parts. If the sentence
is already arranged or the correct sequence doesn't match any of the given sequence,
mark E.

52. (A) The switch to online classes / (B) A time warp after the pandemic and the anxieties
of children are increasing because of / (C) The pivot back to regular schooling / (D)
Teachers seem to be caught in / (E) During the public health emergency and then

A. DBAEC
B. ECBDA
C. DAEBC
D. BDACE
E. None of the above options

53. (A) With rainwater harvesting systems and will enable / (B) The schools will be energy-
efficient / (C) The study of / (D) With natural farming patches, they will be equipped / (E)
Traditional environment-friendly practices

A. DBAEC
B. ECBDA
C. DAEBC
D. BDACE
E. None of the above options

54. (A) An entirely surprising standoff between / (B) The promulgation of Nepal’s
Constitution has been marred by an unforeseen, though not / (C) The seventh anniversary
of / (D) The constitutional head, the executive and the judiciary / (E) It is unfortunate that

A. DBAEC
B. ECBDA

Use Code NIMISHA for maximum Discount

For any query call or whatsapp 9953592800


311

C. DAEBC
D. BDACE
E. None of the above options

55. (A) The capture of Moirang in Manipur under / (B) A wave of enthusiasm among the
masses of India, who were disappointed by / (C) The failure of the Quit India Movement / (D)
INA’s adventurous battles and / (E) The leadership of Lt Col Shaukat Ali on April 14, 1944,
had triggered

A. DBAEC
B. ECBDA
C. DAEBC
D. BDACE
E. None of the above options

Direction(56-60): In the following question, a table is given with two columns. In


column I, three sentences are given with one blank in each of them. In column II, three
IDIOMS(PHRASES) are given. You've to choose the correct pairs of the statement and
the IDIOM(PHRASE) that will fit in that particular sentence correctly and make that
sentence contextually correct.

56.
Column I
A. The advocacy group should _____________ in the criminal justice system to ensure
that all people are treated to a fair trial and equal representation.
B. The President laid out a new economic plan to try to ______________ by increasing
taxes on the wealthy.
C. Tiffany is always gossiping about me, so I wanted to _____________ by starting a
rumor about her.
Column II
D. balance the accounts
E. balance the scale
F. balance the books

Use Code NIMISHA for maximum Discount

For any query call or whatsapp 9953592800


312

A. AE, BF and CD B. AD, BE and CF C. AF, BD and CE D. AE, BD and CF E.


None of the above options

57.
Column I
A. My patient's lung cancer reports ____________ because my uncle recently died of
the same thing.
B. I don't want to ________________ in this new film, with a performance that is dull at
best and downright amateurish at worst.
C. After days of struggling to come up with a solution, one finally
___________________ this morning.
Column II
D. hit below my weight
E. hit me right between the eyes
F. hit me close to home

A. AE, BF and CD B. AD, BE and CF C. AF, BD and CE D. AE, BD and CF E.


None of the above options

58.
Column I
A. The former president of marketing at Floogle Inc. has _____________ as
Spacebook's new chief financial officer.
B. I'm glad you've finally _____________ and chosen a candidate for the job.
C. If the boss approves this initiative, it could _______________ as early as next month.
Column II
D. come on stream
E. come off the fence
F. come on board

Use Code NIMISHA for maximum Discount

For any query call or whatsapp 9953592800


313

A. AE, BF and CD B. AD, BE and CF C. AF, BD and CE D. AE, BD and CF E.


None of the above options

59.
Column I
A. I really _____________ when my exam was postponed to next week, as I hadn't
studied for it at all!
B. I knew you were stressed out about being laid off, but you were so skilled that you
______________.
C. The president's popularity among the more extreme members of his party was
diminished because he ______________ with some bipartisan policies.
Column II
D. landed on both feet
E. dodged the bullet
F. took a middle course

A. AE, BF and CD B. AD, BE and CF C. AF, BD and CE D. AE, BD and CF E.


None of the above options

60.
Column I
A. The young political candidate waited like ______________ for his rival's sordid
history to come to public attention.
B. Watching that game was so stressful—I was like ________________ the whole time!
C. I thought I was getting a great deal buying my car from that online seller, but as soon
as I drove it home, I realized I'd bought _____________.
Column II
D. a cat in a meal-tub
E. a cat on a hot tin roof
F. a cat in the sack

Use Code NIMISHA for maximum Discount

For any query call or whatsapp 9953592800


314

A. AE, BF and CD B. AD, BE and CF C. AF, BD and CE D. AE, BD and CF E.


None of the above options

New Pattern on Short Passage

Based on - Phrasal Verbs, Idioms and Inference.

Direction(61-63): In the following question, there is a small passage. In it, there are
FOUR PHRASAL VERBS highlighted. Those PHRASAL VERBS may not be in their
correct places. You've to arrange them in a contextually correct way.
The passage has also THREE BLANKS indicating THREE MISSING IDIOMS. You've to
choose the correct pair(s) of IDIOMS.
Lasky, the passage has an INFERENCE-BASED QUESTION. You've to choose the
correct answer(s) for this.

Passage 1

Mulayam Singh Yadav or “Netaji” was a political leader and a ______________ in politics
whose career (m)looked up to four decades in the key state of UP. Hailing from Saifai
village in Etawah district, his political journey from the 1970s to 2012 was during a time of
intense ______________ in UP, which he navigated with considerable success. Elected to
the state assembly eight times and Parliament seven times, Yadav, a wrestler, was thrice
chief minister of UP. (n)Arched over and liked by politicians across the spectrum, he was a
member of several parties before forming his own outfit.

A seminal feature of Yadav’s career was that it paralleled and (o)took up UP politics. He
shrewdly deployed three ideologies-cum-strategies — socialism, backward caste
mobilisation and social justice for the ones who were _______________. During his college
days, he was influenced by Ram Manohar Lohia’s socialism, an ideology he (p)acted upon
throughout his life.

61. Which of the following arrangements of the PHRASAL VERBS is CORRECT? If


they're correctly placed, choose E as your answer.

A. p n m o
B. o n m p

Use Code NIMISHA for maximum Discount

For any query call or whatsapp 9953592800


315

C. n m p o
D. m o n p
E. No replacement required

Mulayam Singh Yadav or “Netaji” was a political leader and a ______________ in politics
whose career arched over four decades in the key state of UP. Hailing from Saifai village in
Etawah district, his political journey from the 1970s to 2012 was during a time of intense
_____________ in UP, which he navigated with considerable success. Elected to the state
assembly eight times and Parliament seven times, Yadav, a wrestler, was thrice chief
minister of UP. Looked up to and liked by politicians across the spectrum, he was a
member of several parties before forming his own outfit.
A seminal feature of Yadav’s career was that it paralleled and acted upon UP politics. He
shrewdly deployed three ideologies-cum-strategies — socialism, backward caste
mobilisation and social justice for the ones who were _______________. During his college
days, he was influenced by Ram Manohar Lohia’s socialism, an ideology he took up
throughout his life.

62. Which of the following orders of the IDIOMS is/are correct?

a. big cheese, hurly-burly, on their uppers


b. big shot, argy-bargy, on their beam-ends
c. top dog, grey matter, silk-stocking

A. Only a and b B. Only a and c C. Only b and c D. All a, b and c E. None of a, b


and c

63. Keeping Mulayam Singh Yadav's three ideologies-cum-strategies, which of the


following facts can be CORRECT?

a. During his three stints as CM, Yadav introduced several welfare schemes for the
empowerment of backwards and poor Muslims.
b. An analysis of Yadav’s legacy points to his central role in UP and the limitations and
disappointments of his politics.

Use Code NIMISHA for maximum Discount

For any query call or whatsapp 9953592800


316

c. His biggest challenge was overcoming the elitism of the Congress party, which was in
power and led by Indira Gandhi; he pushed anti-Congressism forward and constantly
encountered challenges from elite landlordism and feudal oppression of the weaker
sections.

A. Only a and b B. Only a and c C. Only b and c D. All a, b and c E. None of a, b


and c

New Pattern on Short Passage

Based on - Phrasal Verbs, Idioms and Inference.

Direction(64-66): In the following question, there is a small passage. In it, there are
FOUR PHRASAL VERBS highlighted. Those PHRASAL VERBS may not be in their
correct places. You've to arrange them in a contextually correct way.
The passage has also THREE BLANKS indicating THREE MISSING IDIOMS. You've to
choose the correct pair(s) of IDIOMS.
Lasky, the passage has an INFERENCE-BASED QUESTION. You've to choose the
correct answer(s) for this.

Passage 2

Worldwide, Covid-19 cases (m)pass on to average approximately 5 lakh daily, while India
has about 3,000 daily cases. On average, over 1,500 people (n)make out daily due to
Covid. Survivors run the risk of long Covid symptoms, which we are just beginning to (o)run
into.
Research has __________ that about half of those infected with SAR-CoV-2 have long
Covid symptoms up to four months after the initial diagnosis. Based on these estimates,
nearly 300 million people are suffering from Long Covid. More conservative estimates place
the figure at 10 to 20 per cent. Around 80 per cent of adults with symptoms of long Covid
(p)keep up limitations in their daily activities compared to before they __________ the virus.
Research has ___________ more than 200 different symptoms of long Covid so far.

Use Code NIMISHA for maximum Discount

For any query call or whatsapp 9953592800


317

64. Which of the following arrangements of the PHRASAL VERBS is CORRECT? If


they're correctly placed, choose E as your answer.

A. p m n o
B. o n m p
C. n m p o
D. m o n p
E. No replacement required

Worldwide, Covid-19 cases keep up to average approximately 5 lakh daily, while India has
about 3,000 daily cases. On average, over 1,500 people pass on daily due to Covid.
Survivors run the risk of long Covid symptoms, which we are just beginning to make out.
Research has _____________ that about half of those infected with SAR-CoV-2 have long
Covid symptoms up to four months after the initial diagnosis. Based on these estimates,
nearly 300 million people are suffering from Long Covid. More conservative estimates place
the figure at 10 to 20 per cent. Around 80 per cent of adults with symptoms of long Covid
run into limitations in their daily activities compared to before they ____________ the virus.
Research has ____________ more than 200 different symptoms of long Covid so far.

65. Which of the following orders of the IDIOMS is/are correct?

a. borne witness to the fact, came down with, put its finger on
b. been a monument to the fact, went down with, put a name to
c. make up for the fact, did away with, known by sight

A. Only a and b B. Only a and c C. Only b and c D. All a, b and c E. None of a, b


and c

66. Which of the following statements can be inferred from the entire context?

a. People with asymptomatic infections are unlikely to suffer long-term effects.


b. Never mind pandemic fatigue, we aren’t free of Covid yet.

Use Code NIMISHA for maximum Discount

For any query call or whatsapp 9953592800


318

c. Many people do not fully recover months after being infected with the coronavirus.

A. Only a and b B. Only a and c C. Only b and c D. All a, b and c E. None of a, b


and c

Direction(67-70): In the following question, a THEME followed by three passages is


given. Determine which passage(s) is/are based on the given theme.

67. Theme - Breaking the Impasse

a. One of the claimed achievements of Prime Minister Sheikh Hasina’s visit to India last
week was a memorandum of understanding on the withdrawal of water from the
Kushiyara river, a branch of the Barak on the border between Assam’s Karimganj and
Bangladesh’s Sylhet district; the agreement will enable the operation of the Rahimpur
canal, which is expected to irrigate significant farmland areas in Bangladesh.
b. The pact entails checks on goods entering Northern Ireland from mainland Britain,
prompting the DUP to fret that the territory is being gradually hived off from the UK.
c. The first chief minister-level talks to settle fisherfolk’s agitation against the Vizhinjam
project ended in a stalemate on Thursday with CM Pinarayi Vijayan rejecting the Latin
Church’s demand to stop the work.

A. Only a and c B. Only a C. Only b D. All a, b and c E. None of a, b and c

68. Theme - The Anti-defection Law

a. The election of both the Prime Minister and Chief Ministers of some of its states and
territories had experienced instances of perceived uncertainty resulting from
legislators changing their political allegiance; the law was sought to limit such
instances in India.
b. It states that any member, after being elected as a representative of a certain political
party, shall be disqualified if he/she joins any other political party after the election.

Use Code NIMISHA for maximum Discount

For any query call or whatsapp 9953592800


319

c. Whoever by words either spoken or intended to be read, or by signs or by visible


representations, makes or publishes any imputation concerning any person intending
to harm, or knowing or having reason to believe that such imputation will harm, the
reputation of such person is punishable by the law.

A. Only a and b B. Only a and c C. Only b and c D. All a, b and c E. None of a, b


and c

69. Theme - India’s Great Anaemia Mystery

a. Observing National Dietetics Day, dietetics department along with the blood bank,
GMSH-16, organised an event, titled ‘Anemia Mukt Bharat’ on Monday morning. A
blood donation camp to mark the occasion was held which saw huge participation by
blood donors; the event also became a platform to create awareness about anemia in
the society.
b. Over half of all women and children in India are anaemic, and that number has
increased in the last three years. Between 2005 and 2015, anaemia declined in India,
albeit marginally. But the recent National Family Health Survey (NFHS-5) data shows
a reversal of those gains — anaemia rates increased from 53% to 57% in women and
58% to 67% in children in 2019-21.
c. It is estimated that about 20% of maternal deaths are directly related to anemia and
another 50% of maternal deaths are associated with it in India. The question,
therefore, is why, despite being the first country to launch the National Nutritional
Anemia Prophylaxis Programme in 1970, the problem of Anemia remains so
widespread.

A. Only a and b B. Only a and c C. Only b and c D. All a, b and c E. None of a, b


and c

70. Theme - Normalisation of India-China Relations Is A Long Way Off

Use Code NIMISHA for maximum Discount

For any query call or whatsapp 9953592800


320

a. India’s challenges on the border are daunting, amidst the growing military power gap
with China; there is no easy return to the border that was once peaceful.
b. India imposed a series of economic measures against China and matched the PLA
deployments on the border. India insisted that it is no longer business as usual with
China and restoring the status quo ante on the border was precondition for the
normalisation of the relationship; China, however, argued that India should not
overstate the conflict on the border and focus on broadening the bilateral relationship.
But Delhi repeatedly reaffirmed that the “state of the border” reflects the “state of the
relationship”.
c. Last week’s agreement between the Indian armed forces and the People’s Liberation
Army on disengagement from the Gogra-Hot Springs area has not come a day too
soon. This area is the last of the friction points created by deliberate Chinese
aggression in the Ladakh sector of the Indo-Tibetan border in the summer of 2020.
Since then sustained talks at the military level has seen the two armies step back a
bit from the face-off points.

A. Only a and b B. Only a and c C. Only b and c D. All a, b and c E. None of a, b


and c

Direction(71-73): In the following question, a table is given with two columns. In


column I, three sentences are given with one blank in each of them. In column II, three
PHRASAL VERBS are given. You've to choose the correct pairs of the statement and
the PHRASAL VERB that will fit in that particular sentence correctly and make that
sentence contextually correct.

71.
Column I
A. I need to ________ the office to pick up some documents.
B. Please ________ each step as you do it.
C. I would try not to _________ the boss today if you can help it—the quarterly sales
figures just came in, and she is not in a good mood.
Column II
D. cheese off
E. pop in
F. check off

Use Code NIMISHA for maximum Discount

For any query call or whatsapp 9953592800


321

A. AE, BF and CD B. AD, BE and CF C. AF, BD and CE D. AE, BD and CF E.


None of the above options

72.
Column I
A. If the school board members __________ funding for the arts, then what will become
of the theater program?
B. The team was down nearly 20 points going into the final quarter of the game, but they
somehow managed to ____________ a stunning victory.
C. I didn't feel sick until after the ceremony had started, so I had to ___________ vomit
the entire time.
Column II
D. choke back
E. cut back
F. claw back

A. AE, BF and CD B. AD, BE and CF C. AF, BD and CE D. AE, BD and CF E.


None of the above options

73.
Column I
A. I was expecting a slap on the wrist or something for making such an inappropriate
joke, but then they decided to __________ and fire me!
B. Hold that container still so those marbles don't ___________.
C. I was working for a special effects studio for a while, and it was my job to
___________ the various creatures and props before they were put into their silicone
molds.
Column II
D. clatter around
E. clay up
F. turn around

A. AE, BF and CD B. AD, BE and CF C. AF, BD and CE D. AE, BD and CF E.


None of the above options

Use Code NIMISHA for maximum Discount

For any query call or whatsapp 9953592800


322

Direction(74-76): In the following question, a table is given with two columns. In


column I, three sentences are given with one blank in each of them. In column II, three
phrasal verbs are given. You've to choose the correct pairs of the statement and the
phrasal verb that will fit in that particular sentence correctly and make that sentence
contextually correct.

74.
Column I
A. After a long day at work, I came home, _________, and watched some TV.
B. Sarah nearly __________ when she gave me the tour of her city.
C. I ___________ at the various nightclubs around town last Saturday.
Column II
D. walked my feet off
E. kicked my heels up
F. put up my feet

A. AE, BF and CD B. AD, BE and CF C. AF, BD and CE D. AE, BD and CF E.


None of the above options

75.
Column I
A. The police suspect that someone tried to ________ the evidence with arson.
B. His sarcastic remarks will really _________ my self-confidence over the years.
C. I need to ________ these old clothes that haven't fit me in years.
Column II
D. chuck away
E. blaze away
F. chisel away

A. AE, BF and CD B. AD, BE and CF C. AF, BD and CE D. AE, BD and CF E.


None of the above options

76.
Column I

Use Code NIMISHA for maximum Discount

For any query call or whatsapp 9953592800


323

A. Arijit Singh is everyone's favorite and he is always able to _______________ during


each performance.
B. Mom has tried to ______________ on her son's death, but I know she's still very
upset about it.
C. Make sure you have researched your position thoroughly and comprehensively
before the debate. You don't want to _______________.
Column II
D. bring a knife to a gunfight
E. bring down the house
F. bring down the shutters

A. AE, BF and CD B. AD, BE and CF C. AF, BD and CE D. AE, BD and CF E.


None of the above options

Directions (77-): The given sentence has been broken up into four different parts. The
error, if any, will be in one or more parts of the sentence. Select the option which contains
the part/parts of the sentence which has/have an error (spelling, grammatical or contextual).
If there is no error, choose option 5.

77. A US judge ordered Starbucks to/ reinstated seven workers in Tennessee/ who say they
were sacked in/ retaliation for pro-union activity/ no error.

A. A US judge ordered Starbucks to


B. reinstated seven workers in Tennessee
C. who say they were sacked in
D. retaliation for pro-union activity
E. No error in any part

78. Over half a million tonnes of wheat/ and other types of food have been shipped/ from
Ukraine's ports in the/ past month, say the United Nations/ no error.

A. Over half a million tonnes of wheat


B. and other types of food have been shipped
C. from Ukraine's ports in the
Use Code NIMISHA for maximum Discount

For any query call or whatsapp 9953592800


324

D. past month, say the United Nations.


E. No error in any part

79. A year after the Taliban takeover, BBC correspondent/ Secunder Kermani visits the
group heartlands/ in southern Afghanistan to discover/ that peace comes at a price/ no error.

A. A year after the Taliban takeover, BBC correspondent


B. Secunder Kermani visits the group heartlands
C. in southern Afghanistan to discover
D. that peace comes at a price.
E. No error in any part

80. Losing a job is devastating for most people,/ and layoffs are still an gruelling/ ordeal, no
matter how warm a reception/ they may get on social media/ no error.

A. Losing a job is devastating for most people,


B. and layoffs are still an gruelling
C. ordeal, no matter how warm a reception
D. they may get on social media
E. No error in any part

Directions (81-83): Identify the words that are similar in meaning to the phrase/idiom in
bold. If none of the options conveys the correct meaning, mark (5) as your answer. The
options do not necessarily need to be grammatically correct.

81. He has taken more responsibilities as he couldn’t say "no" to his boss. I think he has
bitten more than he can chew, and he’ll struggle to handle them all.

A. To take on more responsibility than one can handle


B. To take on less responsibility than one can handle
C. To take more responsibility unwillingly
D. To take responsibility to show your ability
Use Code NIMISHA for maximum Discount

For any query call or whatsapp 9953592800


325

E. None of the above options

82. Do you think these accusations are credible or just an attempt to force the mayor out
of office?

A. To pressure one to hold office despite difficulties


B. To pressure one to resign
C. drive someone out of office
D. Both 2 and 3
E. None of the above options

83. We need to change the locks fore and aft, I'm afraid.

A. The point of completion or conclusion


B. final or most critical extremity
C. At the front and back of something
D. Both 1 and 2
E. None of the above options

84. I can't believe you told your teacher to suck an egg after she yelled at you.

A. An expression of anger
B. An expression of scorn
C. An expression of disdain
D. All of the above
E. None of the above options

Directions (85-91): Read the given passage carefully and answer the questions that
follow.

Policy intervention without knowledge of the ground realities often ends up as an exercise in
Use Code NIMISHA for maximum Discount

For any query call or whatsapp 9953592800


326

self-gratification for those in authority and results in little or no benefit for the intended target
group. Wanting to do good must be matched by knowing the right thing to do in the
circumstance, and in the case of children, be guided by child-centric policies. Whether the
recent recommendation of a parliamentary panel to bring more abandoned children into the
adoption process will fulfil these parameters is an issue that warrants further discussion. A
recent report, “Review of Guardianship and Adoption Laws”, by the Parliamentary Standing
Committee on Personnel, Public Grievances and Law and Justice has pointed to the huge
mismatch between the number of people wanting to adopt children and the number of
children legally available for adoption, and suggested that the way to remedy that would be
to ensure that “orphan and abandoned children found begging on the streets… are made
available for adoption at the earliest”. To do so, it has suggested periodic district surveys to
identify children who are orphaned/abandoned.
The report argued that in a country with millions of orphans, only 2,430 children were
available for adoption. It is true that there are always more people wanting to adopt children
than the number of children actually available for adoption; it has been so historically, but
the increasing chasm, as the report indicates, will undoubtedly have to be addressed.
According to the report, there were 27,939 prospective parents registered with the Central
Adoption Resource Authority (CARA) as on December 2021, from nearly 18,000 in 2017.
There were 6,996 orphaned, abandoned and surrendered children residing in childcare
institutions considered adoptable, but only 2,430 were declared legally free for adoption by
Child Welfare Committees. It claimed that the waiting time for adoption had increased to
three years from one year, in the past five years. The total number of children adopted in
2021-22 was only 3,175.
But the process of adoption in the country was tightened — procedurally and legally — in
response to rampant malpractices and inter-country adoption rackets. CARA was installed
as the nodal body for in-country and inter-country adoptions, to monitor and regulate the
process, ensuring through stringent rules that the adoption is in the best interests of the
child, and no illegality is involved.
While the parliamentary committee has interpreted that there is automatic happiness when a
child in an institution is placed in a home, it is important to exercise caution. No doubt, the
country should take care of its children orphaned due to circumstances, but even as it
acknowledges that institutionalisation may be detrimental over the long term, it should pay
equal attention to the finer aspects of child care, and allow itself to be guided by a child-
centric philosophy. There are no shortcuts in ensuring orphaned children come to no harm.

85. Which of the following are the key findings of the recent report, “Review of Guardianship
and Adoption Laws”?

1. It is recommended to accord equal treatment to both the mother and father as natural
Use Code NIMISHA for maximum Discount

For any query call or whatsapp 9953592800


327

guardians.
2. In India, there are only 2,430 children available for adoption while the number of parents
desiring to bring home a child is growing rapidly.
3. The waiting time for adoption has increased to three years from one year in the past five
years.

A. Both 1 and 2
B. Only 2 and 3
C. Both 1 and 3
D. Only 2
E. All 1, 2, and 3

Policy intervention without knowledge of the ground realities often ends up as an


exercise in self-gratification for those in authority and results in little or no benefit for
the intended target group.

86. What does the author want to convey with these lines?
1. Authorities should first have to understand the ground reality and then indulge in course
correction.
2. Grandiose announcements regarding schemes are made perhaps because it is
considered to be a political necessity.
3. Authorities revel in coming up with ideas and schemes without even caring to understand
the ground realities.

A. Both 1 and 2
B. Only 3
C. Both 1 and 3
D. Only 2
E. All 1, 2, and 3

87. What is CARA, according to the above article?

1. It is authorised to regulate and monitor inter-country and in-country adoptions. ·


2. Central Adoption Resource Authority (CARA) is a statutory body of the Ministry of Women
Use Code NIMISHA for maximum Discount

For any query call or whatsapp 9953592800


328

& Child Development, Government of India.


3. CARA is designated as the Central Authority to deal with inter-country adoptions in
accordance with the provisions of the Hague Convention on Inter-country Adoption, 1993,
ratified by the Government of India.

A. Both 1 and 2
B. Only 2 and 3
C. Both 1 and 3
D. Only 1
E. All 1, 2, and 3

88. Why did the author say that the process of adoption should be tightened?

1. to avoid improper adoption


2. to stop malpractices
3. to deter inter-country adoption rackets.

A. Both 1 and 2
B. Only 2 and 3
C. Both 1 and 3
D. Only 1
E. All 1, 2, and 3

The total number of children adopted in 2021-22 was only 3,175.


But the process of adoption in the country was tightened — procedurally and legally — in
response to rampant malpractices and inter-country adoption rackets.

89. What is the synonym of the highlighted word "rampant"?


1. sparse
2. pervasive
3. widespread

A. Both 1 and 2

Use Code NIMISHA for maximum Discount

For any query call or whatsapp 9953592800


329

B. Only 2 and 3
C. Both 1 and 3
D. Only 1
E. All 1, 2, and 3

90. Which of the following recommendations are deduced from the given article?

1. The point should not be to track more children and put them into adoption, but to not
leave children out of the safety net.
2. In order to link children to nurturing families there is a need for a paradigm shift that looks
beyond “custodial” needs such as food and shelter and focuses on their rights.
3. Orphan and abandoned children found begging in the streets are produced before the
Child Welfare Committee and are made available for adoption at the earliest.

A. Both 1 and 2
B. Only 2 and 3
C. Both 1 and 3
D. Only 1
E. All 1, 2, and 3

It is true that there are always more people wanting to adopt children than the number of
children actually available for adoption; it has been so historically, but the increasing chasm,
as the report indicates, will undoubtedly have to be addressed.

91. What is/are the antonym/s of the highlighted word "chasm"?


1. differences
2. harmony
3. Attachment

A. Both 1 and 2
B. Only 2 and 3
C. Both 1 and 3
D. Only 1
Use Code NIMISHA for maximum Discount

For any query call or whatsapp 9953592800


330

E. All 1, 2, and 3

ANSWER

1.(Answer - D. D

If you read all the statements carefully, you'll see that statements A, B and C are
talking about the SEASONAL FRUITS.
However, statement D is talking about an all-season fruit.

Refer to - we can enjoy fresh, luscious bananas in the dead of January, as well as in July.
ubiquitous - present, appearing, or found everywhere, every time.)

2.(Answer - B. Only c

Kiwi is a SEASONAL FRUIT.


Refer to - it is because it is one of the healthiest winter fruits in India…..

a - Incorrect. It's not talking anything about seasons.

b - Incorrect. Tomatoes are all-season fruits.)

3.(Answer - B. B

If you read all the statements carefully, you'll see that statements A, C and D are
talking about the UTILITIES OF VANITY CASES OR BOXES.
However, statement B is talking about the design aesthetics of vanity cases and
materials used in vanity cases.)

4.(Answer - A. Only a and b

Statements a and b are talking about the UTILITIES OF VANITY CASES.

Use Code NIMISHA for maximum Discount

For any query call or whatsapp 9953592800


331

c - Incorrect. This is talking about the design aesthetics.)

5.(Answer - A. AD, BF and CE)

6.(Answer - B. Only BE and CD)

7.(Answer - D. AF, BD and CE)

8.(Answer - A. Only a and b

a - Incorrect.
The phrase - and country-level plans are drawing up - must be replaced with -

and country-level plans are being drawn up.


draw up - to compose or frame.

b - Incorrect.
The correct phrase should be - India has also conveyed that the approach cannot be read
as a recognition of the Taliban as legitimate rulers.)

9.(Answer - B. Only a and c

a - Incorrect.

'promulgated various restrictions' must be replaced with 'promulgating various


restrictions'.
promulgate - put (a law or decree) into effect by official proclamation.

c - Incorrect.
'especially the cities' must be replaced with 'especially in the cities', as the actual
object is - 'life'.)

10. (Answer - A. Only a and b

a - Incorrect.

The correct phrase is - on account of.


That mean - because of.

Use Code NIMISHA for maximum Discount

For any query call or whatsapp 9953592800


332

b - Incorrect.

'conjuring up' must be replaced with 'conjures up', as the subject is - a cursory look at
the situation in Myanmar.

conjure up - to present to the mind; evoke or imagine.)

11.(Answer - B. Only a and c

a - Incorrect.

The correct phrase is - for both of what are colloquially known

c - Incorrect.

As the sentence starts with 'While', the correct phrase should be -


not all is well with some features of the economy.)

12.(Answer - A. A

If you read all the statements carefully, you'll see that paragraphs B, C and D are
talking about India's religious tolerance and diversity.

But, paragraph A is talking about dissimilarity.


India’s major religious communities often don’t feel they have much in common with
one another. Hence, there is no sense of tolerance clearly displayed.)

13.(Answer - A. Only a and b

Absolutely!
Since paragraphs B, C and D are talking about India's religious tolerance and
diversity, these two statements can absolutely be inferred.

c - Incorrect. This may be true but can't be inferred from these three statements.)

14.(Answer - D. All a, b and c

Use Code NIMISHA for maximum Discount

For any query call or whatsapp 9953592800


333

All these statements are saying that - India’s major religious communities often don’t
feel they have much in common with one another.
And that is the very idea of paragraph A.)

15.(Answer - A. A

If you read all the statements carefully, you'll see that paragraphs B, C and D are
talking about different cyber attacks and threats.
But, paragraph A is talking about the risks of employees making serious security
mistakes.)

16.(Answer - C. Only b and c

a - Incorrect. It's nowhere mentioned that small-and medium-sized businesses have


become a soft target.

b and c can correctly be inferred.)

17.(Answer - D. All a, b and c

a and b - Correct.
They are talking about misuse of emails and poor password management.

c - Correct. This is talking about lack of experience(naivety).

18.(Answer - B. B

If you read all the statements carefully, you'll see that paragraphs A, C and D are
talking about the things that cyber insurance provides or covers.

But, paragraph B is talking about the cost of cyber insurance.)

19.(Answer - B. Only a and c

b - Incorrect. It can never be a CONCLUSION. It's a fact.

Use Code NIMISHA for maximum Discount

For any query call or whatsapp 9953592800


334

a and c can well be the conclusions.

niche - denoting products, services, or interests that appeal to a small, specialized section
of the population.)

20. (Answer - D. All a, b and c

Data breach insurance or Cyber insurance costs vary depending on the size of your
company and how much coverage your business needs. If you have a smaller
company, you may not need as much coverage, and thus, your premiums will be
lower than a larger company with many areas that need protection.

All these statements can certainly be inferred from PARAGRAPH B.)

21.(Answer - A. Only a and b

Refer to - Close to 20% of children in the country below the age of five suffer from the most
visible and life-threatening form of malnutrition — wasting. About 35% of such children
are not as tall as they should be.

stunting - Stunting is the impaired growth and development that children experience from
poor nutrition.

wasting - children suffering from wasting are too thin and their immune systems are weak,
leaving them vulnerable to developmental delays, disease and death.

c - Incorrect. The phrase - '35% of children in the country' - is absolutely wrong.)

22.(Answer - D. All a, b and c

Refer to - However the Ministry of Women and Child Development not only dismissed the
GHI but also questioned the intent of its authors and the veracity of the index.

b - Correct. It's talking about questioning the intent of the authors.

c - Correct. It's talking about the fact that the Ministry of Women and Child
Development dismissed the GHI.

Use Code NIMISHA for maximum Discount

For any query call or whatsapp 9953592800


335

a - Correct. It's talking about questioning the veracity or accuracy of the report.)

23.(Answer - A. Only a and b

Refer to - However, as several scholars have pointed out, the nutrition deficit of the
country’s children is, in large measure, a function of their poor diets.

c - Incorrect.
It's nowhere mentioned and also absolutely absurd that children are disinclined or
unwilling to take proper food.
They don't get it. That doesn't mean they don't want it.)

24.(Answer - B. Only a and c

bolster / hold up - strengthen.

repository / depository - a place where or receptacle in which things are or may be stored.

prickly - irritable or complaining.


make a fuss - to complain.

complacent - self-satisfied.

India can't be self-satisfied with the GHI report, as the report isn't in favor of India.)

25.(Answer - B. Only a and c

weak sauce / not up to snuff - substandard or poor.

of the first water - superior.

gnashing of teeth - complaint and unhappy mental state.

kick up a stink - to make a strong public complaint.

raise a rumpus - to make a forceful complaint.)

26.(Answer - A. Only a and b

Use Code NIMISHA for maximum Discount

For any query call or whatsapp 9953592800


336

a and b are basically saying the same thing.

c - Incorrect. It's contradictory. Staying non-discriminatory and even-handed means


staying neutral. But, in reality, we don't do that.)

27.(Answer - A. Only a and b

Absolutely! The more we become more like a consumer, the more benefit the market
will get.

c - Incorrect. It's not the reason. It might be a possible outcome or aftereffect or a


motivation.)

28.(Answer - B. Only a and c

Certainly!

These two points are the reasons the author has criticized the voting mindset so
much.

disparaging - judgemental or fault-finding.

b - Incorrect. It's not a negative aspect.)

29.(Answer - A. Only a and b

Absolutely!
Both have the same tone.

c - Incorrect. It's absolutely contradictory.

lay a finger on - touch someone, especially with the intention of harming them.
Being selfless never means harming yourself.)

30. (Answer - B. Only a and c

b - Incorrect. It doesn't make sense.

Use Code NIMISHA for maximum Discount

For any query call or whatsapp 9953592800


337

commiseration - sympathy.
righteousness - morality.)

31.(Answer - B. Only a and c

a - Correct. It means - hidden.

c - Correct. It means - secret or not known to the public.

b - Incorrect. It means - clearly visible.)

32.(Answer - D. All a, b and c

The statement is talking about a mother's unconditional love.

affinity - deep affection or empathy.


propinquity - the state of being close to someone.)

33.(Answer - D. All a, b and c

put on the line - put at risk.


hassle - stress.
exacerbated - increased.)

34.(Answer - D. All a, b and c

a - Correct.
It means - persuade (someone) to do something by sustained coaxing or flattery.
It has been derived from 'cajoled'.

b - Correct.
It means - have in one's possession.
It has been derived from 'own'.

c - Correct.

Use Code NIMISHA for maximum Discount

For any query call or whatsapp 9953592800


338

It means - to smile.
It has been derived from 'smiling'.)

35.(Answer - D. All a, b and c

be subjected to - to suffer.
live through - to go through pain.

take stock of - make an overall assessment of a particular situation.


It has been derived from 'survey'.)

36.(Answer - C. a d g i

The correct arrangement is -

b a f e c d g i j h

37.(Answer - B. Only a and c

default on - To fail to repay a loan on something.

cut the mustard - come up to expectations; reach the required standard.


foot the bill for - to pay debt.

founder - to fail.
shell out (for) - pay a specified amount of money.

show up / turn up / come along - to arrive.


relish - to enjoy.)

38.(Answer - B. Only a and c

disarray / chaos / mayhem - a state of disorganization.

stand by - be present while something bad is happening but fail to take any action to stop it.

dissent / cry out - protest.

Use Code NIMISHA for maximum Discount

For any query call or whatsapp 9953592800


339

cherish / hold dear - to treasure.)

39.(Answer - A. Only a and b

These two statements explain the actual situation in a very positive tone, in which the
passage has been framed.
Sri Lanka's win at Asia cup has been deemed very significant by the author.
Hence, these two statements are absolutely correct.

c - Incorrect.
It's incorrect due to one wrong word.
That word is - pinprick.

pinprick - sth that causes irritation.

This word should be replaced with 'balm'.)

40. (Answer - B. Only a and c

b can easily be eliminated.

The phrase - cricket might appear incongruous - is contradictory to the idea of power
of sports.

incongruous - not in harmony or keeping with the surroundings or other aspects of


something.

a and c are absolutely positive and reassuring.)

41.(Answer - D. All a, b and c

rock-ribbed - high-spirited.

stout-hearted - determined.

of the first water - highly admirable.)

Use Code NIMISHA for maximum Discount

For any query call or whatsapp 9953592800


340

42.(Answer - C. Only b and c

a shot in the arm - an encouraging stimulus.


Refer to - it has also inspired such playwrights, poets and novelists as Shakespeare,
Shelley, Poe and Conrad.

formal - (of language) associated with literary works or other formal writing; having a
marked style intended to create a particular emotional effect; literary.

vernacular - the language or dialect spoken by the ordinary people in a particular country or
region.
No such thing is mentioned.)

43.(Answer - B. Only a and c

Unrelated look-alikes may share more than just facial features.

Hence, a and c are absolutely correct.

disparate - unlinked or unrelated.

b - Incorrect. It's out of context.)

44.(Answer - B. Only a and c

The doppelgängers not only shared similarities in DNA but were genetically
predisposed to certain lifestyle traits, such as smoking habits as well as education
levels.

That means, shared genetic variation relates not only to physical appearance but may
also influence common habits and behaviour.
And also, the doppelgangers who looked most similar, “not only shared the face, but
also other features beyond, such as anthropometric characters and personality traits.

anthropometric characters - height, weight, etc.

b - Incorrect. It's contradictory.)

Use Code NIMISHA for maximum Discount

For any query call or whatsapp 9953592800


341

45.(Answer - B. Only a and c

methodical - scientific.

empirical - based on, concerned with, or verifiable by observation or experience rather than
theory or pure logic.
Refer to - facial recognition algorithms, genetic testing and questionnaires on daily life and
behaviour

arbitrary - based on random choice or personal whim, rather than any reason or system.)

46.(Answer - D. All a, b and c

tidy - organized or methodical.

hands-on - involving direct involvement or intervention.

heuristic - enabling someone to discover or learn something.)

47.(Answer - C. a d g i

The correct arrangement is -

b c e f a d g i j h

48.(Answer - B. Only a and c

The fact - If a minimum-wage earner were to aspire to earn the remuneration of a


senior CEO in the garment industry, it would take the person 941 years to do so.
This fact shows how unequal our society is.
It's deeply embedded in the way our economic system works.

From this fact, it can also be said that three types of economic inequality do exist in
India - Income Inequality, Pay Inequality and Wealth Inequality.

Use Code NIMISHA for maximum Discount

For any query call or whatsapp 9953592800


342

b - Incorrect. The phrase - will not be imperative - is contradictory and incorrect.)

49.(Answer - C. Only b and c

Of course, the Government of India is very satisfied and complacent.

full of oneself - highly complacent and boastful.

gratified - highly pleased.

browned off - dissatisfied.)

50. (Answer - D. All a, b and c

All these attributes are the definition of 'Per Capita Income' which has been measured
by the IMF.)

51.(Answer - B. e d f

The correct arrangement is -

h b a e d f c i g)

52.(Answer - A. DBAEC

Teachers seem to be caught in a time warp after the pandemic and the anxieties of children
are increasing because of the switch to online classes during the public health emergency
and then the pivot back to regular schooling.)

53.(Answer - D. BDACE

The schools will be energy-efficient with natural farming patches, they will be equipped with
rainwater harvesting systems and will enable the study of traditional environment-friendly
practices.)

Use Code NIMISHA for maximum Discount

For any query call or whatsapp 9953592800


343

54.(Answer - E. None of the above options

It's ECBAD

It is unfortunate that the seventh anniversary of the promulgation of Nepal’s Constitution has
been marred by an unforeseen, though not an entirely surprising standoff between the
constitutional head, the executive and the judiciary.)

55.(Answer - C. DAEBC

INA’s adventurous battles and the capture of Moirang in Manipur under the leadership of Lt
Col Shaukat Ali on April 14, 1944, had triggered a wave of enthusiasm among the masses of
India, who were disappointed by the failure of the Quit India Movement.)

56.(Answer - A. AE, BF and CD

balance the scale - To achieve justness, fairness, or equality in some situation.

balance the books - By extension, to try and make up the deficit in a given budget by
increasing revenue or income.

balance the accounts - To get revenge for a prior misdeed.)

57.(Answer - C. AF, BD and CE

hit one close to home - To affect one deeply and emotionally because one can relate very
closely.

hit below one's weight - To perform, achieve, or do something at a level that is considered
to be lower than one's abilities, talents, or personal attributes.

hit one (right) between the eyes - To suddenly become obvious or apparent to one.)

58.(Answer - E. None of the above options

Use Code NIMISHA for maximum Discount

For any query call or whatsapp 9953592800


344

The correct option is - AF, BE and CD

come on board - To join a company, organization, or some other group.

come off the fence - To make a decision or take a side when presented with two options or
possibilities, usually after a period of indecision or delay.

come on stream - To become functional.)

59.(Answer - D. AE, BD and CF

dodge a bullet - To narrowly avoid something or some situation that turns out to be
undesirable, disastrous, dangerous, or otherwise harmful.

land on both feet - To come through or survive a tough or uncertain situation successfully
or gracefully.

take a middle course - To compromise between two extreme or polarizing alternatives; to


find a solution, policy, or course of action that is acceptable or agreeable to two different or
opposing sides.)

60. (Answer - B. AD, BE and CF

cat in a meal-tub - A surprise attack or ambush; a hidden or unseen danger.

cat on a hot tin roof - Someone who is anxious and unable to sit still or relax.

cat in the sack - Something to be suspicious of.)

61.(Answer - C. n m p o

arched over - spanned over.


Looked up to - respected.
acted upon - influence.
take up - espouse; adopt or support (a cause, belief, or way of life).

62.(Answer - A. Only a and b

Use Code NIMISHA for maximum Discount

For any query call or whatsapp 9953592800


345

big cheese / big shot / top dog - a very powerful, influential and important person.

hurly-burly / argy-bargy - ferment; agitation and excitement among a group of people,


typically concerning major change and leading to trouble or violence.

on one's uppers / on one's beam ends - in a very poor condition or underprivileged.

grey matter - intelligence.


silk-stocking - wealthy or rich.)

63.(Answer - B. Only a and c

a - Correct.
Refer to - He shrewdly deployed three ideologies-cum-strategies — socialism, backward
caste mobilisation and social justice for the ones who were underprivileged.

Hence, a can absolutely be correct. It's talking about backward caste mobilisation.

c - Correct.
It's talking about socialism and social justice for the ones who were underprivileged.

landlordism - the system whereby land (or property) is owned by landlords to whom tenants
pay a fixed rent.
feudal - absurdly outdated or old-fashioned.
oppression - prolonged cruel or unjust treatment or exercise of authority.
elitism - the belief that a society or system should be led by an elite.

b - Incorrect. It may be true but it's negative in tone, which can't be derived from his
three ideologies-cum-strategies.)

64.(Answer - A. p m n o

keep up - continue.
pass on - die.
make out - understand.
run into - encounter or experience.)

65.(Answer - A. Only a and b

Use Code NIMISHA for maximum Discount

For any query call or whatsapp 9953592800


346

bear witness to / be a monument to - indicate.

come down with / go down with - catch or develop (a disease or infectious agent).

put one's finger on / put a name to / know by sight - to identify.

make up for - to take the place of something lost or damaged or to compensate for
something bad with something good.
do away with - to remove or put an end to something.)

66.(Answer - C. Only b and c

a - Incorrect. It may be true but there is no substantial evidence in the passage to


back this.

b and c - Correct.

Refer to - More conservative estimates place the figure at 10 to 20 per cent. Around 80 per
cent of adults with symptoms of long Covid run into limitations in their daily activities
compared to before they contracted the virus.

And,

Research has indicated that about half of those infected with SAR-CoV-2 have long Covid
symptoms up to four months after the initial diagnosis.)

67.(Answer - B. Only a

impasse / stalemate / deadlock - a situation in which no progress is possible, especially


because of disagreement.
Statement a is certainly talking about breaking a deadlock.

b - Incorrect. It's contradictory.

hived off - removed.


fret - to worry.

c - Incorrect. It's contradictory.

Use Code NIMISHA for maximum Discount

For any query call or whatsapp 9953592800


347

Refer to - talks to settle fisherfolk’s agitation against the Vizhinjam project ended in a
stalemate on Thursday….)

68.(Answer - A. Only a and b

defection - the act of leaving a country, political party, etc. to go to another one.

Hence, a and b are correct as they talk about this very thing.

c - Incorrect. It's talking about Anti-defamation Law.)

69.(Answer - C. Only b and c

Both these statements are talking about India's great Anemia Mystery or Problem.

a - Incorrect. It's not a negative statement. It's just stating facts.)

dietetics - the branch of knowledge concerned with the diet and its effects on health,
especially with the practical application of a scientific understanding of nutrition.)

70.(Answer - A. Only a and b

way off - remote.

These two statements express the fact that the normalisation of India-China relations
is a remote thing.

c - Incorrect. Slightly contradictory.

Refer to - Since then sustained talks at the military level has seen the two armies step back
a bit from the face-off points.

has not come a day too soon - used to say that something didn't happen when it was
almost too late.)
71.(Answer - A. AE, BF and CD

pop in - To come or go somewhere for a brief visit or purpose.

Use Code NIMISHA for maximum Discount

For any query call or whatsapp 9953592800


348

check off - To mark someone or something as present, verified, or completed, as on a list.

cheese off - To anger or irritate someone.)

72.(Answer - A. AE, BF and CD

cut back - To reduce or decrease something.

claw back - To regain or recover something with great difficulty and effort.

choke back - To try to keep from expelling something, such as words or tears.)

73.(Answer - C. AF, BD and CE

turn around - To do something suddenly that is unexpected and often undesired.

clatter around - To make loud rattling noise or disturbing commotion while moving around
in a space or container.

clay up - To create a sculpture made of clay to be used as the model for a mold. A noun or
pronoun can be used between "clay" and "up.")

74.(Answer - C. AF, BD and CE

put up one's feet - To relax, as or as if by lounging with one's feet elevated.

walk one's feet off - To cause, direct, or force one to walk for so long that one's feet
become very tired and sore.

kick one's heel up - To engage or partake in jovial festivities, especially by dancing.)

75.(Answer - A. AE, BF and CD

blaze away - To destroy or clear away something in a great fire.

chisel away - To erode or diminish (something) incrementally but continuously.

Use Code NIMISHA for maximum Discount

For any query call or whatsapp 9953592800


349

chuck away - To dispose of something. A noun or pronoun can be used between "chuck"
and "away.")

76.(Answer - A. AE, BF and CD

bring down the house - To perform or entertain so successfully as to cause the audience
to erupt in applause, laughter, or cheers for a long stretch of time.

bring down the shutters - To stop thinking or talking about something.

bring a knife to a gunfight - To come poorly prepared or equipped for some task, goal,
competition, or confrontation. Often used in the negative as a forewarning or piece of
advice.)

77.B

Solution : “Reinstated” must be changed to “reinstate”.

The correct formation: “to+v1”

78.D

Solution : “say” must be replaced with “says”.

79.B

Solution : “Group” must be replaced with “group's”.

80.B

Solution : “an” must be replaced by “a”.

81.A
Solution : bite off more than (one) can chew: To take on more responsibility than one can
handle.

82.D

Solution : force (one) out of office: To pressure one to resign or otherwise force one to
leave or be removed from an authoritative position that one has been elected to.

83.C

Use Code NIMISHA for maximum Discount

For any query call or whatsapp 9953592800


350

Solution : fore and aft: At the front and back of something

84.D

Solution : (go) suck an egg: An expression of anger, scorn, or disdain.

85.B

Solution : Statement 1: not mentioned in the above article.

Refer to: A recent report, “Review of Guardianship and Adoption Laws”, by the
Parliamentary Standing Committee on Personnel, Public Grievances and Law and Justice
has pointed to the huge mismatch between the number of people wanting to adopt
children and the number of children legally available for adoption… It is true that there
are always more people wanting to adopt children than the number of children
actually available for adoption

It claimed that the waiting time for adoption had increased to three years from one
year, in the past five years.

86.C

Solution : Statements 1 and 3: correct.

Statement 2: out of context.

87. D

Solution : Refer to: CARA was installed as the nodal body for in-country and inter-country
adoptions to monitor and regulate the process, ensuring through stringent rules that the
adoption is in the best interests of the child and no illegality is involved.

88. E

Solution : Refer to: But the process of adoption in the country was tightened—procedurally
and legally—in response to rampant malpractices and inter-country adoption rackets.

89. B

Solution : Rampant: existing or spreading everywhere in a way that is very difficult to


control; pervasive.

"sparse" is an antonym of the highlighted word.

Use Code NIMISHA for maximum Discount

For any query call or whatsapp 9953592800


351

90. E

Solution : Refer to: and suggested that the way to remedy that would be to ensure that
“orphan and abandoned children found begging on the streets… are made available for
adoption at the earliest”.

While the parliamentary committee has interpreted that there is automatic happiness when a
child in an institution is placed in a home, it is important to exercise caution. No doubt, the
country should take care of its children orphaned due to circumstances, but even as it
acknowledges that institutionalisation may be detrimental over the long term, it should pay
equal attention to the finer aspects of child care, and allow itself to be guided by a child-
centric philosophy. There are no shortcuts in ensuring orphaned children come to no harm.

91. B

Solution : Chasm: a wide difference of feelings, interests, etc. between two people or
groups.

“Harmony” and “attachment” are the antonyms of the highlighted word.

Use Code NIMISHA for maximum Discount

For any query call or whatsapp 9953592800

You might also like